You are on page 1of 132

Introduction to Topology

Tomoo Matsumura
November 30, 2010

Contents
1

Topological spaces
1.1 Basis of a Topology . . . . . . . . . . . . . . . . . . . . . . . . . . . . . . . . . . . . . . . . .
1.2 Comparing Topologies . . . . . . . . . . . . . . . . . . . . . . . . . . . . . . . . . . . . . . .

3
3
5

Product topology, Subspace topology, Closed sets, and Limit Points


2.1 The Product Topology on X Y . . . . . . . . . . . . . . . . . .
2.2 The Subspace Topology . . . . . . . . . . . . . . . . . . . . . . .
2.3 Closed Sets, Closure, Interior, and limit points . . . . . . . . . . .
2.4 Subspace topology and closed sets/closure . . . . . . . . . . . . .

.
.
.
.

.
.
.
.

.
.
.
.

.
.
.
.

.
.
.
.

.
.
.
.

.
.
.
.

.
.
.
.

.
.
.
.

.
.
.
.

.
.
.
.

.
.
.
.

.
.
.
.

.
.
.
.

.
.
.
.

.
.
.
.

6
6
7
7
9

Hausdorff Spaces, Continuous Functions and Quotient Topology


3.1 Hausdorff Spaces . . . . . . . . . . . . . . . . . . . . . . . .
3.2 Continuous Maps . . . . . . . . . . . . . . . . . . . . . . . .
3.3 Homeomorphisms . . . . . . . . . . . . . . . . . . . . . . . .
3.4 Properties of continuous functions . . . . . . . . . . . . . . .
3.5 Quotient Topology . . . . . . . . . . . . . . . . . . . . . . .

.
.
.
.
.

.
.
.
.
.

.
.
.
.
.

.
.
.
.
.

.
.
.
.
.

.
.
.
.
.

.
.
.
.
.

.
.
.
.
.

.
.
.
.
.

.
.
.
.
.

.
.
.
.
.

.
.
.
.
.

.
.
.
.
.

.
.
.
.
.

.
.
.
.
.

.
.
.
.
.

.
.
.
.
.

.
.
.
.
.

11
11
12
13
14
15

Compactness, Metric, and Universality


4.1 Definition of compactness and the compact subspaces of R . .
4.2 Useful Theorems about compactness . . . . . . . . . . . . . .
4.3 Metric . . . . . . . . . . . . . . . . . . . . . . . . . . . . . .
4.4 Limit Point Compactness and sequentially compact 28 [Mun]
4.5 Completeness of metric and Theorem 4.11 . . . . . . . . . . .
4.6 Universal properties of maps and induced topologies . . . . .

.
.
.
.
.
.

.
.
.
.
.
.

.
.
.
.
.
.

.
.
.
.
.
.

.
.
.
.
.
.

.
.
.
.
.
.

.
.
.
.
.
.

.
.
.
.
.
.

.
.
.
.
.
.

.
.
.
.
.
.

.
.
.
.
.
.

.
.
.
.
.
.

.
.
.
.
.
.

.
.
.
.
.
.

.
.
.
.
.
.

.
.
.
.
.
.

.
.
.
.
.
.

.
.
.
.
.
.

17
17
18
19
20
22
22

Connected spaces
5.1 Connected spaces: the concept . . . . . . . . . . . . . . .
5.2 Connected spaces: basic propeties . . . . . . . . . . . . .
5.3 R is connected . . . . . . . . . . . . . . . . . . . . . . . .
5.4 Path connected . . . . . . . . . . . . . . . . . . . . . . .
5.5 Components, locally connected and locally path-connected

.
.
.
.
.

.
.
.
.
.

.
.
.
.
.

.
.
.
.
.

.
.
.
.
.

.
.
.
.
.

.
.
.
.
.

.
.
.
.
.

.
.
.
.
.

.
.
.
.
.

.
.
.
.
.

.
.
.
.
.

.
.
.
.
.

.
.
.
.
.

.
.
.
.
.

.
.
.
.
.

.
.
.
.
.

.
.
.
.
.

24
24
24
25
26
27

Topological manifolds and embedding into RN


6.1 Topological Manifolds 7, 30, 32, 33, 36 [Mun] . . . . . . . . . . . . . . . . . . . . . . . . . .

29
29

.
.
.
.
.

.
.
.
.
.

Group theory
7.1 Definition of groups and homomorphisms
7.2 Examples . . . . . . . . . . . . . . . . .
7.3 More definitions . . . . . . . . . . . . . .
7.4 Groups given by generators and relations .
7.5 Topological groups . . . . . . . . . . . .
7.6 Group actions on topological spaces . . .

.
.
.
.
.
.

32
32
32
33
34
35
35

Homotopy of Paths 51 [Mun]


8.1 Homotopy and Path Homotopy equivalence . . . . . . . . . . . . . . . . . . . . . . . . . . . .
8.2 Definition of a product among paths and the homotopy invariance . . . . . . . . . . . . . . . .

37
37
38

Fundamental Group and Covering Spaces


9.1 Non-canonical uniqueness of the fundamental group of a path-connected space. . . . . . . . . .
9.2 Functoriality of fundamental groups . . . . . . . . . . . . . . . . . . . . . . . . . . . . . . . .
9.3 Covering spaces and the example . . . . . . . . . . . . . . . . . . . . . . . . . . . . . . . . . .

40
40
41
42

.
.
.
.
.
.

.
.
.
.
.
.

.
.
.
.
.
.

.
.
.
.
.
.

.
.
.
.
.
.

.
.
.
.
.
.

.
.
.
.
.
.

.
.
.
.
.
.

.
.
.
.
.
.

.
.
.
.
.
.

.
.
.
.
.
.

.
.
.
.
.
.

.
.
.
.
.
.

.
.
.
.
.
.

.
.
.
.
.
.

.
.
.
.
.
.

.
.
.
.
.
.

.
.
.
.
.
.

.
.
.
.
.
.

.
.
.
.
.
.

.
.
.
.
.
.

.
.
.
.
.
.

.
.
.
.
.
.

.
.
.
.
.
.

.
.
.
.
.
.

.
.
.
.
.
.

.
.
.
.
.
.

.
.
.
.
.
.

10 Fundamental Groups and Covering Spaces, 54 [Mun]


10.1 A lifting of a map along another map and liftings of paths along a covering map
10.2 Lifting Path-Homotopy . . . . . . . . . . . . . . . . . . . . . . . . . . . . . .
10.3 Surjectivity and bijectivity of e0 . . . . . . . . . . . . . . . . . . . . . . . . .
10.4 1 (S 1 , b0 )  Z . . . . . . . . . . . . . . . . . . . . . . . . . . . . . . . . . . .
10.5 Retraction and fixed points theorem . . . . . . . . . . . . . . . . . . . . . . .
10.6 Deformation retract 58 . . . . . . . . . . . . . . . . . . . . . . . . . . . . . .

.
.
.
.
.
.

.
.
.
.
.
.

.
.
.
.
.
.

.
.
.
.
.
.

.
.
.
.
.
.

.
.
.
.
.
.

.
.
.
.
.
.

.
.
.
.
.
.

.
.
.
.
.
.

44
44
45
46
46
46
47

11 Application and more computations of 1


11.1 Homotopy invariance of fundamental groups .
11.2 Fundamental group of S n . . . . . . . . . . .
11.3 Fundamental theorem of algebra . . . . . . .
11.4 Fundamental group of torus . . . . . . . . . .

.
.
.
.

.
.
.
.

.
.
.
.

.
.
.
.

.
.
.
.

.
.
.
.

.
.
.
.

.
.
.
.

.
.
.
.

.
.
.
.

.
.
.
.

.
.
.
.

.
.
.
.

48
48
49
50
52

12 Fundamental groups of surfaces


12.1 Fundamental groups of a double torus . . . . . . . . . . . . . . . . . . .
12.2 Constructing various surfaces by identifying edges of polytopes . . . . .
12.2.1 Fundamental group of surfaces constructed from polytopes . . . .
12.2.2 Torus, n-fold torus, RP2 , Klein Bottle . . . . . . . . . . . . . . .
12.3 Properly discontinuous actions, covering spaces, and fundamental groups

.
.
.
.
.

.
.
.
.
.

.
.
.
.
.

.
.
.
.
.

.
.
.
.
.

.
.
.
.
.

.
.
.
.
.

.
.
.
.
.

.
.
.
.
.

.
.
.
.
.

.
.
.
.
.

.
.
.
.
.

53
53
53
53
54
54

13 Cauchy Integral Formula, Jordan Curve Theorem and Winding Number Theorem
13.1 Cauchy Integral Formula 66 [Mun] . . . . . . . . . . . . . . . . . . . . . . . . .
13.1.1 Residue Theorem . . . . . . . . . . . . . . . . . . . . . . . . . . . . . . .
13.2 Jordan Curve Theorem . . . . . . . . . . . . . . . . . . . . . . . . . . . . . . . .
13.3 Winding Number Theorem . . . . . . . . . . . . . . . . . . . . . . . . . . . . . .

.
.
.
.

.
.
.
.

.
.
.
.

.
.
.
.

.
.
.
.

.
.
.
.

.
.
.
.

56
56
58
58
60

14 Classification of compact topological surfaces.


14.1 Triangulation . . . . . . . . . . . . . . . .
14.2 Classification of polygon quotients . . . . .
14.3 Euler characteristic . . . . . . . . . . . . .
14.4 What happen to the Klein bottle? . . . . . .

.
.
.
.

.
.
.
.

.
.
.
.

.
.
.
.

.
.
.
.

.
.
.
.

.
.
.
.

61
61
61
63
63

.
.
.
.

.
.
.
.

.
.
.
.

.
.
.
.

.
.
.
.

.
.
.
.

.
.
.
.

.
.
.
.

.
.
.
.

.
.
.
.

.
.
.
.

.
.
.
.

.
.
.
.

.
.
.
.

.
.
.
.

.
.
.
.

.
.
.
.

.
.
.
.

.
.
.
.

.
.
.
.

.
.
.
.

.
.
.
.

.
.
.
.

.
.
.
.

.
.
.
.

.
.
.
.

.
.
.
.

.
.
.
.

.
.
.
.

.
.
.
.

.
.
.
.

.
.
.
.

.
.
.
.

.
.
.
.

.
.
.
.

Topological spaces

A topology is a geometric structure defined on a set. Basically it is given by declaring which subsets are open sets. Thus
the axioms are the abstraction of the properties that open sets have.

Definition 1.1 (12 [Mun]). A topology on a set X is a collection T of subsets of X such that
(T1) and X are in T ;
(T2) Any union of subsets in T is in T ;
(T3) The finite intersection of subsets in T is in T .
A set X with a topology T is called a topological space. An element of T is called an open set.
Example 1.2. Example 1, 2, 3 on page 76,77 of [Mun]
Example 1.3. Let X be a set.
(Discrete topology) The topology defined by T := P(X) is called the discrete topology on X.
(Finite complement topology) Define T to be the collection of all subsets U of X such that X U either
is finite or is all of X. Then T defines a topology on X, called finite complement topology of X.

1.1

Basis of a Topology

Once we define a structure on a set, often we try to understand what the minimum data you need to specify the structure. In
many cases, this minimum data is called a basis and we say that the basis generate the structure. The notion of a basis of the
structure will help us to describe examples more systematically.

Definition 1.4 (13 [Mun]). Let X be a set. A basis of a topology on X is a collection B of subsets in X such
that
(B1) For every x X, there is an element B in B such that x U.
(B2) If x B1 B2 where B1 , B2 are in B, then there is B3 in B such that x B3 B1 B2 .
Lemma 1.5 (Generating of a topology). Let B be a basis of a topology on X. Define TB to be the collection of
subsets U X satisfting
(G1) For every x U, there is B B such that x B U.
Then TB defines a topology on X. Here we assume that trivially satisfies the condition, so that TB .
Proof. We need to check the three axioms:
(T1) TB as we assumed. X TB by (B1).
(T2) Consider a collection of subsets U TB , J. We need to show
[
U :=
U TB .
J

By the definition of the union, for each x U, there is U such that x U . Since U TB , there is B B
such that x B U . Since U U, we found B B such that x B U. Thus U TB .
(T3) Consider a finite number of subsets U1 , , Un TB . We need to show that
U :=

n
\
i=1

Ui

TB .

Lets just check for two subsets U1 , U2 first. For each x U1 U2 , there are B1 , B2 B such that
x B1 U1 and x B2 U2 . This is because U1 , U2 TB and x U1 , x U2 . By (B2), there is
B3 B such that x B3 B1 B2 . Now we found B3 B such that x B3 U.
We can generalize the above proof to n subsets, but lets use induction to prove it. This is going to
be the induction on the number of subsets.
When n = 1, the claim is trivial.
Suppose that the claim is true when we have n 1 subsets, i.e. U1 Un1 TB . Since
U = U1 Un = (U1 Un1 ) Un
and regarding U 0 := U1 Un1 , we have two subsets case U = U 0 Un . By the first
arguments, U TB .
Definition 1.6. TB is called the topology generated by a basis B. On the other hand, if (X, T ) is a topological
space and B is a basis of a topology such that TB = T , then we say B is a basis of T . Note that T itself is a
basis of the topology T . So there is always a basis for a given topology.
Example 1.7.
(Standard Topology of R) Let R be the set of all real numbers. Let B be the collection of all open
intervals:
(a, b) := {x R | a < x < b}.
Then B is a basis of a topology and the topology generated by B is called the standard topology of R.
Let R2 be the set of all ordered pairs of real numbers, i.e. R2 := R R (cartesian product). Let B be the
collection of cartesian product of open intervals, (a, b) (c, d). Then B is a basis of a topology and the
topology generated by B is called the standard topology of R2 .
(Lower limit topology of R) Consider the collection B of subsets in R:

(
)

B := [a, b) := {x R | a x < b}
a, b R .

This is a basis for a topology on R. This topology is called the lower limit topology.
The following two lemmata are useful to determine whehter a collection B of open sets in T is a basis for T or
not.
Remark 1.8. Let T be a topology on X. If B T and B satisfies (B1) and (B2), it is easy to see that TB T .
This is just because of (G1). If U TB , (G1) is satisfied for U so that x U, Bx B s.t. x Bx U.
Therefore U = xU Bx . By (T2), U T .
Lemma 1.9 (13.1 [Mun]). Let (X, T ) be a topological space. Let B T . Then B is a basis and TB = T if and
only if T is the set of all unions of subsets in B.
Proof.

() Let T 0 be the set of all unions of open sets in B. If U T , then U satisfies (G1), i.e.
x U, Bx B s.t. x Bx U. Thus U = xU Bx . Therefore U T 0 . We proved T T 0 . It follows
from (T2) that T 0 T .

() Since X T , X = B some union of sets in B. Thus x X, B s.t. x B . This proves (B1) for
B. If B1 , B2 B, then B1 B2 T by (T2). Thus B1 B2 = B , B B. So x B1 B2 , B B s.t.
x B . This B plays the role of B3 in (B2). Thus B is a basis. Now it makes sense to consider TB and
we need to show TB = T . By the remark, we already know that TB T . On the other hand, if U T ,
then U = B , B B. Hence, x U, B such that x B U. Thus (G1) is satisfied for U. Thus
U TB . This proves TB T .

4

Lemma 1.10 (13.2 [Mun]). Let (X, T ) be a topological space. Let B T . Then B is a basis and TB = T if and
if any U T satisfies (G1), i.e. x U, Bx B s.t. x Bx U.
Proof.
Trivial by the definition of TB .
X satisfies (G1) so B satisfies (B1). Let B1 , B2 B T . By (T3), B1 B2 T . Thus B1 B2 satisfies
(G1). This means (B2) holds for B. Thus B is a basis. Now the assumption can be rephrased as T TB . By the
remark above, we already know T TB .


1.2

Comparing Topologies

Definition 1.11. Let T , T 0 be two topologies for a set X. We say T 0 is finer than T or T is coarser than T 0 if
T T 0 . The intuition for this notion is (X, T 0 ) has more open subsets to separate two points in X than (X, T ).
Lemma 1.12 (13.3). Let B, B0 be bases of topologies T , T 0 on X respectively. Then T 0 is finer than T
B B and x B, B0 B0 s.t. x B0 B.
Proof. Since B T T 0 , all subsets in B satisfies (G1) for T 0 , which is exactly the statement we wanted to
prove. The LHS says B T 0 . We need to show that it implies that any U T satisfies (G1) for T 0 too.
U T , x U, B B s.t. x B U
But
B B, x B, B0 B0 s.t. x B0 B.
Combining those two,
U T , x U, B0 B0 s.t. x B0 B U.


Product topology, Subspace topology, Closed sets, and Limit Points

This week, we explore various way to construct new topological spaces. And then we go on to study limit points.
For this end, it is convenient to introduce closed sets and closure of a subset in a given topology.

2.1

The Product Topology on X Y

The cartesian product of two topological spaces has an induced topology called the product topology. There is also an
induced basis for it. Here is the example to keep in mind:

Example 2.1. Recall that the standard topology of R2 is given by the basis
B := {(a, b) (c, d) R2 | a < b, c < d}.
Check the axioms (B1) and (B2)!
Definition 2.2 (15 [Mun]). If (X, TX ) and (Y, TY ) are topological spaces, then the collection B of subsets of the
form U V X Y, U TX , V TY forms a basis of a topology. The topology generated by B is called product
topology on X Y.
Proof.
(B1) Let (x, y) X Y be an arbitrary element. We need to find a subset in B containing (x, y), but since
X Y B, it is obvious.
(B2) For any U1 V1 , U2 V2 B, the intersection is (U1 V1 ) (U2 V2 ) = (U1 U2 ) (V1 V2 ) B. So
it is obvious again.

Example 2.3. The above definition gives a topology on R2 . The following theorem identify this topology with
the standard one!
Theorem 2.4 (15.1). If BX is a basis of (X, TX ) and BY is a basis of (Y, TY ), then
BXY := {B C | B BX , C BY }
is a basis of the product topology on X Y.
Proof. To check BXY , lets use Lemma 1.10 which state that B is a basis for T iff for any U T and any x U,
there is B B such that x B U. Let W T and (x, y) W. By the definition of product topology, there are
U TX and V TY such that (x, y) U V W. Since BX and BY are bases, there are B BX and C BY
such that x B U and y C V. Thus we found B C BXY such that (x, y) B C W.

Example 2.5. The standard topology of R2 is the product topology of two copies of R with the standard topology.
Example 2.6. The standard topology of Rn is given by the basis
B := {(a1 , b1 ) (an , bn ) Rn | ai < bi }.
Example 2.7. For any p = (x0 , y0 ) R2 , let B,x be the open disk of radius  > 0 centered at p. We can define a
topology of R2 by
BD := {D,x | x R2 ,  R>0 }.
The topology defined by B coincides with the standard topology on R2 .

2.2

The Subspace Topology

A subset of a topological space has a naturally induced topology, called the subspace topology. In geometry, the subspace
topology is the source of all funky topologies.

Definition 2.8. Let (X, T ) be a topological space. Let Y be a subset of X. The collection
TY := {Y U | U T }
is a topology on Y, called the subspace topology.
Lemma 2.9. If B is a basis for T , then
BY := {Y B | B B}
is a basis of the subspace topology TY for Y.
Proof. Use Lemma 1.9. Let V TY , i.e. V = Y U for some U T . For every x V, there is B B such that
x B U since B is a basis of T (Lemma 1.9). Now we found Y B BY such that x Y B V.  

2.3

Closed Sets, Closure, Interior, and limit points

Closed sets are nothing but complement of open sets. On the other hand, we can also say that open sets are nothing but
complement of closed sets. Thus we can actually use closed sets to define topology, although mathematicians usually use
open sets to define topology.

Definition 2.10. Let A be a subset of a topological space (X, T ).


A is a closed set of X if X A is an open set.
The closure A of A in X is the intersection of all closed sets of X, containing A.
[
A =
C
CA
closed

The interior Int A of A in X is the union of all open sets of X, contained in A.


\
Int A =
O
OA
open

x X is a limit point of A if x A {x}.


Remark 2.11. It is not so difficult to see from the definition that
A = A A : closed, and

Int A = A A : open.

Example 2.12.
In the standard topology for R, a set of a single element (we say a point) is a closed set, because R {a} is
an open set. Any finite set is also closed, since X {a1 , , an } = ni=1 (R {ai }) is a finite intersection of
open sets.
In the discrete topology of a set X, every point is a closed set but also an open set.
In the lower limit topology, a point is a closed set.
In the finite complement topology of any set X, a point is a closed set. But any infinite set is not closed
by definition of finite complement topology, except X itself. For example, Z is a closed set in R in the
standard topology but not in the finite complement topoloty.
7

Lemma 2.13 (Interior in terms of closure).


Int A = X (X A).
Proof. We need to show that X Int A = X A. Let U denote open sets and C denote closed sets. By definition
S
Int A = UA U. Therefore we can do the set theoretic computation:
[
\
\
X Int A = X
U=
(X U) =
C = X A.
UA

UA

C(XA)


Remark 2.14 (Defining topology by closed sets). A topology on a set X is given by defining open sets of X.
Since closed sets are just exactly complement of open sets, it is possible to define topology by giving a collection
of closed sets. Let K be a collection of subsets of X satisfying
(C1) , X P.
(C2) Any intersection of subsets in K is also in K.
(C3) Any finite union of subsets in K is also in K.
Then define T by
T := {X C | C K}
is a topology, i.e. it satisfies (T 1, 2, 3). On the other hand, if T is a topology, i.e. the collection of open sets, then
K := {X U | U T }
satisfies (C1, 2, 3).
Exercise 2.15. Prove the above claims.
Theorem 2.16. Let A be a subset of the topological space (X, T ). Let B be a basis of T .
(a) x A if and only if every neighborhood U of x intersects with A non-trivially, i.e. U A , .
(b) x A if and only if every neighborhood B B of x intersects with A non-trivially.
Terminology: U is a neighborhood of x if U T and x U.
Proof. It is easier to prove the contrapositive statements of the theorem. We will prove
x < A a U T , s.t. x U and U A = b B B, s.t. x B and B A = .
then C a closed set such that C A and x < C. Then x < C implies that x is in X C which is an
a If x < A,
open set. C A implies that (X C) A = . Let U = X C and we are done for the middle statement.
b For any U T such that x U and U A = , by the definition of a basis, there exists B such that
x B U (G1). This B clearly satisfies the last statement.
b If there is B B such that x B and B A, then this B also plays the role of U in the middle statement.
x < A.
a If U satisfies the middle statement, then C := X U is closed and x < C. Thus by definition of A,


Remark 2.17.
If it is a bird, then it is an animal.
The contrapositive statement of the above statement is
If it is not an animal, then it is not a bird.
It is the theorem that those two statements are equivalent.
Example 2.18. The subset A := {1/n | n = 1, 2, 3, } R is not closed in the standard topology. To see this,
we can appy Theorem 2.16 and Remark 2.11. A is closed if and only if A = A. So we will show A , A. Observe
0 < A and let (a, b) be an arbitrary neighborhood of 0 where a < 0 < b. Then, no matter how small b is there is

n such that 1/n < b. Thus (a, b) intersects with A non-trivially. Therefore by the theorem, 0 A.
Exercise 2.19. The boundary A of a subset A of a topological space X is defined by
A := A Int A.
From this definition, it follows that A is the disjoint union of A and Int A, i.e. A = A Int A and Int A.
(a) Find the boundary, the closure and the interior of (0, 1] in R with the standard topology.
(b) Find the boundary, the closure and the interior of (0, 1] in R with the finite complement topology.
(c) Find the boundary, the closure and the interior of of Q in R with the standard topology.
(d) Prove that A = A X A.
Exercise 2.20. Consider standard topology T st , finite complement topology T f.c. and the discrete topology Tdsct
on R. We have
= R, T st : Q
= R, Tdsct : Q
= Q.
T f.c. : Q
T f.c. : Z = R, T st : Z = Z, Tdsct : Z = Z.
Observe that T f.c. T st Tdsct . Now consider two topologies T T 0 on X, i.e. T 0 is finer than T . Let A X
T

T0

a subset. Let A and A

be the closures in the corresponding topologies. Prove that


T0

A A .

2.4

Subspace topology and closed sets/closure

Theorem 2.21. Let (X, T ) be a topological space and let Y X be a supspace of (X, T ), i.e. a subset with the
subspace topology. Then a subset A of Y is closed in Y if and only if A is an intersection of Y and a closed subset
in X.
Proof. Since an open set in Y is an intersection of Y and an open set in X by definition of subspace topology,
this theorem is rather trivial in the perspective of Remark 2.14. Here is another way to prove:
A : closed in Y

Y A : open in Y by def of closed sets


Y A = Y U U is some open set in X, by def of subspace

A = Y (Y U) = Y (X U)

A = Y C C is a closed set in X


Exercise 2.22. Let Y be a subspace of a topological space (X, T ). Prove that, if A is a closed subset of Y and Y
is a closed subset in X, then A is a closed subset of X.
9

Theorem 2.23. Let Y be a subspace of (X, T ) and let A be a subset of Y. The closure of A in Y is A Y where
A is the closure of A in X.
Proof. Let A X and A Y be the closures of A in the corresponding spaces.
\
\
\
\
A Y =1
CY =2
(Y C X ) =3
(Y C X ) =4 Y
C X = Y A X .
CY A

(YC X )A

C X A

C X A

=1 is by def of closures in Y. =2 is by the previous theorem. =3 follows since Y A. =4 is just the set theoretic
computation. =5 is the definition of closures in X.


10

Hausdorff Spaces, Continuous Functions and Quotient Topology

3.1

Hausdorff Spaces

Definition 3.1. A topological space (X, T ) is called a Hausdorff space if


(H1) x, y X such that x , y, U x , Uy T such that x U x , y Uy , and U x Uy =
i.e. for every pair of distinct points x, y in X, there are disjoint neighborhoods U x and Uy of x and y respectively.
Example 3.2.
(a) Rn with the standard topology is a Hausdorff space.
(b) R with the finite complement topology is NOT a Hausdoff space. Suppose that there are disjoint neighborhoods U x and Uy of distinct two points x and y. Observe that U x must be an infinite set, since R U x
is finite and R is an infinite set (see the definition of finite complement topoloty). It is the same for Uy . By
the disjointness, Uy X U x . Therefore it contradicts with the finiteness of X U x . Thus U x and Uy can
not be disjoint. This proves that R with the finite complement topology is not Hausdorff.
(c) Any infinite set X with the finite complement topology is not a Hausdorff. This is because every non-empty
open sets intersect non-trivially. (b) is just a special case of (c).
Proof. Let U1 := X F1 and U2 := X F2 be open sets (Fi is finite). Then U1 U2 = X (F1 F2 ).
Since X is infinite and F1 F2 is finite, this can not be empty.

The following exercise is a nice one to relate the Hausdorff condision and the product topology.
Exercise 3.3. Show that X is a Hausdorff space if and only if the diagonal := {(x, x) | x X} X X is closed
with respect to the product topology.
Theorem 3.4. Every finite set in a Hausdorff space X is closed.
Proof. A point {x} X is closed set because we can show that X {x} is open: y X {x}, by the Hausdorff
condition, there is an open set Uy such that y Uy but x < Uy (we are not using the whole condition) so that
Uy X {x}. Now, since a finite set is a finite union of single points, it is closed.

Remark 3.5. The opposite statement of Theorem 3.4 is not true. The counter example is Example 3.2 (b).
Definition 3.6. Let {xn | n N} be a sequence of points in a topological space. The sequence {xn | n N}
converges to a point x X if, for every neighborhood U of x, there is a positive integer N N such that xn U
for all n N. In other words, U contains all but finitely many points of {xn }.
Lemma 3.7. Let B be a basis of a Hausdorff space X. Then {xn } converges to x iff every B B containing x
contains all but finitely many points of {xn }.
Exercise 3.8. Find all points that the sequence {xn = 1/n | Z>0 } converges to with respect to the following
topology of R. Justify your answer.
(a) Standard Topology
(b) Finite Complement Topology
(c) Discrete Topology
(d) Lower Limit Topology
Are (c) and (d) Hausdorff?
Theorem 3.9. If X is a Hausdorff space, then every sequence of points in X converges to at most one point of X.
11

Proof. We prove by deriving a contradiction. Suppose that {xn } converges to x and y and that x , y. Then by
(H1), there are U x and Uy in TX such that x U x and y Uy and U x Uy = . Since U x contains all but finitely
many points of {xn }, it is not possible that Uy contains all but finitely many points of {xn }. Thus {xn } can not
converges to y.


3.2

Continuous Maps

Definition 3.10 (- continuity). A function f : R R is - continuous at x0 R when f (x) gets closer to


f (x0 ) as x gets closer to x0 .More precisely,
(--cont at x0 ) > 0, > 0 such that f (x) ( f (x0 ) , f (x0 ) + ), x (x0 , x0 + ).
In terms of the basis of topology B := {Bx, := (x , x + ) | x R,  R>0 },
(--cont at x0 ) B f (x0 ), , Bx0 , such that f (Bx0 , ) B f (x0 ), .
(Cont at x0 ) B f (x0 ), , Bx0 , such that Bx0 , f 1 (B f (x0 ), )
A map f : R R is an - continuous function if f is - continuous at every x R.
(Cont) B f (x), , f 1 (B f (x), ) is open .
Using (G1)
(Cont)

for all open set U R, f 1 (U) is open.

Definition 3.11. Let (X, TX ) and (Y, TY ) be topological spaces. Let BY be a basis of TY . A map f : X Y is
continuous at x0 X if
(Cont at x0 ) V f (x0 ) : a nbhd of f (x0 ) in Y, Ux0 a nbhd of x0 in X such that U x0 f 1 (V f (x0 ) ).
By using (G1),
(Cont at x0 ) V f (x0 ) BY : a nbhd of f (x0 ) in Y, Ux0 a nbhd of x0 in X such that U x0 f 1 (V f (x0 ) ).
A map f : X Y is continuous if f is continuous at every point of X.
(Cont) V TY , f 1 (V) TX , i.e. the preimage of open sets are open.
By using (G1), we can just check the condition for open sets in the basis:
f : X Y continuous (Cont) V BY , f 1 (V) TX
By the way we get the definition of continuity of a map, we have
Theorem 3.12. A map f : R R is continuous at x0 R relative to the standard topology if and only if f is
- continuous at x0 R.
Example 3.13. Here are some trivial example
Let f : X Y be a map of topological spaces. Show that f is always continuous, if X has the discrete
topology.
Let R s and R f be the set of all real numbers with the standard topology and the finite complement topology.
If idR : R R be the identity map, i.e. idR (x) = x. Then id : R s R f is continuous but id : R f R s is
not continuous. This is because the standard topology is strictly finer than the finite complement topology,
i.e. the standard topology has strictly more open sets than the finite complement topology. In general, we
have
If (X, T ) is finer than (X, T 0 ), then idX : (X, T ) (X, T 0 ) is continuous.
12

(Constant functions) If f : X Y maps all points of X to a single point y0 Y, then f is a continuous


function.
Exercise 3.14. We can formulate the continuity by using closed sets: show that a map f : X Y of topological
spaces is continuous if and only if for every closed set CY of Y, the preimage f 1 (CY ) is closed in X.
Exercise 3.15. Prove that f : X Y is continuous if and only if for every subset A of X, we have f (A) f (A).
Exercise 3.16. Define a map f : R R2 by x 7 (cos x, sin x). Then f is continuous relative to the standard
topologies.
Exercise 3.17. Define a map f : R R by

|x|
x 7

|x|

if x is rational
if x is irrational.

Then f is continuous at x = 0 but not continuous at other points.


Lemma 3.18. Let f : X Y be a continuous map and let {xn | n Z>0 } X be a sequence which converges to
x X. Then the sequence { f (xn )} Y converges to f (x).
Proof. Let V be any neighborhood of f (x). Then f 1 (V) is an open set containing x so its a neighborhood of x.
By the definition of convergence, there is N such that xn f 1 (V) for all n > N. This N satisfies that f (xn ) V
for all n > N so that { f (xn )} converges to f (x).


3.3

Homeomorphisms

Definition 3.19. A map f : X Y of topological spaces is a homeomorphism if f is bijective and both f and
f 1 are continuous.
Remark 3.20. f is a homeomorphism if f is bijective and
for any open set U X, f (U) is open in Y,
for any open set V Y, f 1 (V) is open in X.
Indeed, if we have bases for topologies for X and Y, we just need to see the conditions above for basis elements.
Example 3.21.
1

The function f : R R defined by f (x) = x3 is a homeomorphism. The inverse function f 1 (y) = x 3 is


also continuous.
f (x) = x2 is not a homeomorphism because it is not a bijection. But we restrict f to some subset, it is a
homeomorphism relative to the subspace topology. For example, f : [0, ) [0, ), then f is a bijective
1
continuous map and f 1 (x) = x 2 is also continuous.
Consider he function f : [0, 2) S 1 given by t 7 (cos t, sin t) where S 1 is the unit circle in R2 with
the subspace topology. It is a bijective continuous map but f 1 is not continuous. This is because, for
example, the image of the open set [0, 1) under f is not open in S 1 .
How to show f ([0, 1)) is not closed in S 1 : Let Bball be the basis of standard topology of R2 given by
open disk. The basis of the subspace topology on S 1 is B := {S 1 B | B Bball }. Let B0 be an arbitrary
open disk centered at f (0). Then S 1 B0 can not be contained in f ([0, 1)), since there is always a small
 such that f () < f ([0, 1)) and f () S 1 B0 . This shows that f ([0, 1)) is not open in the subspace
topology S 1
Exercise 3.22. Show that the open interval (/2, /2) of R with the subspace topology is homeomorphic to R.
Show that any open interval is homeomorphic to R.
13

Definition 3.23. A map f : X Y is a topological embedding if f is injective and f : X f (X) is a


homeomorphism where f (X) has the subspace topology inherited from Y.
Exercise 3.24. Let X and Y be topological spaces and A X a subspace. Let f : A Y be a continuous
function and assume that Y is Hausdorff. Show that if f can be extend to a continuous function f : A Y, i.e.
if there is a continuous function f : A Y such that f|A = f , then it is unique.

3.4

Properties of continuous functions

Lemma 3.25. The composition g f of continuous functions f : X Y and g : Y Z is continuous.


Proof. Let U be an open set in Z. Then g1 (U) is open in Y since g is continuous. Then f 1 (g1 (U)) =
(g f )1 (U) is open in X since f is continuous.

Exercise 3.26. Suppose that X, Y, Z are topological spaces. Let f : X Y and g : Y Z be maps of sets. Prove
or disprove the following statement:
(a) If f : X Y is continuous and the composition map g f : X Z is continuous, then g : Y Z is
continuous.
(b) If g : Y Z is continuous and the composition map g f : X Z is continuous, then f : X Y is
continuous.
Lemma 3.27. Suppose that X is a union of open sets Ui , i I. Suppose we have continuous maps fi : Ui Y
such that fi |Ui U j = f j |Ui U j , then there is a unique continuous map f : X Y such that f |Ui = fi .
Proof. There is a map f : X Y such that f |Ui = fi by saying for each x X, let f (x) := fi (Ui ). This is
well-defined since the choice of Ui doesnt change the map f . Furthermore, if f, g : X Y are maps such that
f |Ui = fi . Then for any x X, there is Ui such that x Ui and f (x) = fi (x) = g(x) by the conditions. Thus f = g.
So its unique. To show that such f is continuous, let V be an open set in Y. Then
[
[
fi1 (V).
f 1 (V) Ui =
f 1 (V) =
i

The first equality follows from the assumption that X is a union of Ui s. Thus f 1 (V) is an open set since it is a
union of open sets (each fi1 (V) is an open set in Ui and it is an open in X since Ui is open in X.)

Lemma 3.28. A map f : Z X Y is continuous if and only if 1 f : Z X and 2 f : Z Y are
continuous.
Proof. Let U1 U2 be an open set in X Y. Then f 1 (U1 U2 ) is an open set in Z. Let U2 = Y. Then
f 1 (U1 Y) = (1 f )1 (U1 ) (2 f )1 (Y) = (1 f )1 (U1 ) Z = (1 f )1 (U1 )
Thus 1 f is continuous. Similarly for 2 f . On the other hand,
f 1 (U1 U2 ) = (1 f )1 (U1 ) (2 f )1 (U2 )
implies that if U1 and U2 are open sets in X and Y respectively, then f 1 (U1 U2 ) is an open set since (1
f )1 (U1 ) and (2 f )1 (U2 ) are open in Z. Since every open set in X Y is of the form U1 U2 , we can conclude
that f is continuous.

Theorem 3.29. If f : X Y is an injective continuous map and Y is Hausdorff, then X must be Hausdorff.
Proof. Let x1 , x2 X are distinct points. Then f (x1 ) and f (x2 ) are distinct points in Y since f is injective.
Therefore there are neighborhoods V1 and V2 of f (x1 ) and f (x2 ) respectively such that V1 V2 = . Since f
is continuous, f 1 (V1 ) and f 1 (V2 ) are open sets and in particular neighborhoods of x1 and x2 . Since f 1 (V1 )
f 1 (V2 ) = f 1 (V1 V2 ) = f 1 () = (see HW1), we found the disjoint neighborhoods of x1 and x2 , thus X is
Hausdorff.

14

3.5

Quotient Topology

Definition 3.30. Let : X Y be a surjective map of topological spaces. The map is a quotient map if
a subset U in Y is open if and only if the preimage 1 (U) is open in X.
Note that only if part is the continuity of , so this condition is stronger than being continuous.
Exercise 3.31. A map f : X Y of topological spaces is called a open (closed) map if the image of every open
(closed) set in X is again open (closed). Show that a continuous surjective map f : X Y is a quotient map if it
is either an open or closed map.
Example 3.32. Define a map f : [0, 1] S 1 by x 7 (cos 2x, sin 2x) where S 1 is the unit circle in R2 with
the induced topology. It is a continuous surjective map. It is not an open map but it is a closed map. Observe the
images of (1/2, 1] and [1/2, 1]. Thus, it is a quotient map.
Definition 3.33. Let X be a topological space and A a set. Let f : X A be a surjective map. There is a unique
topology on A which makes f to be a quotient map. It is called the quotient topology and the topological space
A is called the quotient space.
Proof. The definition of the quotient map actually determines the topology of A: TA must be the set of all subsets
U A such that f 1 (U) is open in X. The axiom (T1) is obvious. The axioms (T2,3) follows from the fact that
the preimage preserves the unions and the intersections.

Remark 3.34. An equivalence relation on a set X defines a surjective map f : X A where A is the set of all
equivalence classes (see [Set]). The typical construction of a quotient space is given by identifying equivalent
points. Let be an equivalence relation on X and then denote X/ the set obtained by identifying equivalent
points. Then we have the natural surjective map q : X X/, x 7 [x].
Example 3.35. Let X := [0, 1] and identify 0 and 1, i.e. the equivalence relation is given by a a for all
a [0, 1] and 0 1. The map f in Example 3.32 factors through X/:
/ S1
[0, 1]I
O
II
II
g
I
q III
$
[0, 1]/
f

f
x MM / (cos 2x, sin 2x)
O
MMM
MMM
g
M
q MMM
M&
[x]

Then we can show that g is a homeomorphism, using the following useful lemma:
Lemma 3.36. Consider the following commutative diagram, i.e. g f1 = f2 :




 f1

X@
@@
@@
f2 @@

/ Z.
g

If f1 and f2 are quotient maps and g is a bijection, then g must be a homeomorphism.


Proof. It suffices to show that g is continuous, since then g1 is also continuous (the claim holds if we replace
g by g1 ). Let U be an open set in Z. We want to show that V := g1 (U) is an open set in Y, but a set V
in Y is an open set if and only if f11 (V) is an open set in X by the definition of the quotient topology. Since
f11 (V) = f11 (g1 (U)) = f21 (U) is an open set (the quotient topology of Z), V must be an open set.

Example 3.37 (2-Sphere ver 1). Let D := {(x, y) R2 | x2 + y2 1} be the unit disk in R2 . Identify all points on
the boundary of the disk, i.e. p p for all Int X and p q if p, q D. It can be shown to be homeomorphic to
the unit 2 sphere which is defined by
S 2 := {(x, y, z) R3 | x2 + y2 + z2 = 1} in R3 with standard topology.
15

Exercise 3.38 (2-sphere ver 2). Consider the disjoint union X := D1 t D2 of two unit disk D1 and D2 . Identify
(x1 , y1 ) (x2 , y2 ) if x1 = x2 and y1 = y2 where (x1 , y1 ) D1 and (x2 , y2 ) D2 . Then show that X/ is
homeomorphic to the unit 2-sphere S 2 in R3 .
Example 3.39 (Torus). Consider the unit square X := [0, 1] [0, 1] in R2 . Identify (0, y) (1, y) and (x, 0)
(x, 1) for all x, y [0, 1]. A torus T is defined by X/. Show that it is homeomorphic to S 1 S 1 .
Exercise 3.40. Prove the following:
(a) If f : X Y and g : Y Z are quotient maps, then g f : X Z is a quotient map.
(b) Let f : X Y and g : Y Z be continuous maps. If f and g f are quotient maps, then g is a quotient
map.

16

Compactness, Metric, and Universality

4.1

Definition of compactness and the compact subspaces of R

Definition 4.1. Let X be a topological space. A collection of open sets Ua , a A is an open covering of X if
X = a U a .
X is compact if every open covering of X contains a finite subcollection that also covers X.
Example 4.2.
1. R is not compact: Consider the infinite covering (n, n + 2), n Z:
2. X = {0} {1, 1/2, 1/3, } is compact: every neighborhood of 0 contains all but finitely many points of
X. Thus for any open covering of X, take one open set U containing 0 and choose other open sets that
correspond to each of the finite many points not in U.
3. (0, 1) is not compact since it is homeomorphic to R.
Remark 4.3. The set of real numbers satisfy the following properties:
(R1) If a non-empty subset A R is bounded above, i.e. there exists b R such that a b for all a A, then
there is the least upper bound (or supremum) of A, i.e. there exists the smallest b R such that a b for
all a A. The least upper bound of A is denoted by sup A.
(R2) If x < y, then there is a number z R such that x < z < y.
An order relation on a set satisfying these two properties is called a linear continuum.
Theorem 4.4 (27.1 [Mun]). Closed intervals of R are compact.
Proof. Let U , A be an open covering of [a, b] R.
(1) Let x [a, b), then there is y (x, b] such that [x, y] is covered by one open set in A := {U }.
Each U is a union of open intervals. Let (h1 , h2 ) be an open interval of U such that h1 < x < h2 .
By (R2), there is y such that x < y < h2 . Now it is clear that [x, y] is covered by (h1 , h2 ) and so by
U .
(2) Let C be the set of all y (a, b] such that [a, y] is covered by finitely many open sets in A. Then C is not
empty because applying (1) for x = a, then there is y such that [a, y] is covered by one of U . Now C is
bounded above since C (, b]. By (R1), the least upper bound c which should satisfy a < c b.
(3) c belongs to C, i.e. [a, c] is covered by finitely many open sets in A.
Suppose that c < C. Let (h1 , h2 ) be one of open intervals of U , which contains c. Then there is
z C such that z (h1 , c) because c is the smallest that bounds C. By the definition of C, [a, z]
is covered by finitely many open sets in A. Since [z, c] U , [a, z] [z, c] = [a, c] is covered by
finitely many open sets. Contradiction.
(4) c is actually b.
Suppose that c < b. Apply (1) to x := c. Then there is y (c, b] such that [c, y] is covered by one
open set U . Since c C, [a, c] is covered by finitely many open sets as we proved in (3). Thus
[a, c] [c, y] = [a, y] is covered by finitely many open sets, i.e. y C. This contradict fact that c
bounds C. Thus c = b.


Thus [a, b] is covered by finitely open sets in A.

17

4.2

Useful Theorems about compactness

Theorem 4.5 (26.2 [Mun]). Every closed subspace C of a compact space X is compact.
Proof. Let {U }A be an open covering of C. By the definition of subspace topology, each U = C V for
some open set V in X. Then {V } {X C} is an open covering of X, therefore there is a finite subcollection
{Oi , i = 1, , n} that covers X, and so certainly {Oi C, i = 1, , n} covers C. If X C is in the subcollection,
(X C) C = , so we can assume that {Oi C, i = 1, , n} is a finite subcollection of {U , A}. Thus we
have a finite subcollection that covers C.

Theorem 4.6 (26.7 [Mun]). The product of finitely many compact spaces is compact.
Proof. If we just prove it for product of two compact spaces, then the claim follows from induction. Let X and
Y be compact spaces. Let TX , TY be topologies of X, Y and let TXY be the product topology of X Y.
(1) x X and N TXY such that N {x} Y, W TX such that x W and N W Y.
An opet set N in X Y containing {x} Y is called a tube about {x} Y. This claim says, although
a tube may not be of the form W Y, but for a given tube, we can find a smaller tube that is of the
form W Y. To prove this, we need Y to be compact. The given N is of the form U V . Then
{(U V ) ({x} V )} is an open covering of {x} Y since N {x} Y. Since {x} Y  Y, {x} Y
is compact and so there is a finite subcollection {U1 V1 , , Un Vn } that covers {x} Y. Then
W := ni=1 Ui is an open set in X (since it is a finite intersection) and we can assume that x W
since if x < Ui , we can get rid of Ui Vi from the list. Now
N ni=1 Ui Vi W Y {x} Y.
(2) Let {N } be an arbitrary open covering of X Y. For each x X, we have a subcollection {N1 ,x , , Nn ,x }
that covers {x} Y since {x} Y is compact. Then N x = ni=1 Ni ,x {x} Y. By (1), there is W x such
that N x W x Y {x} Y. Since X is compact and {W x , x X} is an open covering of X, we have a
subcollection {W x j , j = 1, , m}. Now {Ni ,x j , 1 i n, 1 j m} gives a finite subcollection of {N }
that covers X Y:
i, j Ni ,x j = j N xi j W x j Y X Y.

Theorem 4.7 (26.5 [Mun]). The image of a compact space under a continuous map is compact.
Theorem 4.8 (26.3 [Mun]). Every compact subspace Y of a Hausdorff space X is closed.
Proof. We will show that X Y is open, i.e. for every x X Y, there is a neighborhood of x contained in
X Y. For every y Y, there are disjoint neighborhoods U x,y of x and Uy of y (Hausdorff). By collecting
such Uy s, we obtain an open covering of Y. Note that U x,y may be different for every y. Nevertheless by the
compactness of Y, we can choose the finite subcollection {Uy1 , , Uyn } that covers Y. The finite intersection
U x of U x,yi , i = 1, , n is again an open set and it is disjoint from Y. Therefore we found a neighborhood U x
contained in X Y.

Theorem 4.9 (26.6 [Mun]). Let f : X Y be a continuous bijection. If X is compact and Y is Hausdorff, then
f must be a homeomorphism.
Proof. It suffices to show that f is a closed map since f is bijective. If A is closed in X, then A is compact by
Theorem 4.5. Then by Theorem 4.7, f (A) is compact. Thus by Theorem 4.8, f (A) is closed.

Example 4.10.
(1) The cartesian product of closed intervals (a box) in Rn is compact.

18

(2) Every closed subset of Rn contained in a box is compact.


(3) The unit n-sphere and the closed n-ball are compact.
(4) R with the finite complement topology is not Hausdorff. The subspace Z is compact: for any open covering
of Z, take an open set containing 0. With the finite complement topology, this open set misses only finitely
many points. By choosing an open set for each missed point, we find the subcollection. Thus Z is compact.
However Z is not closed.
(5) A closed interval in R with the finite complement topology is compact.
Theorem 4.11 (27.3 [Mun]). A subspace A of Rn is compact if and only if it is closed and is bounded in the
euclidean metric d, i.e. there is M > 0 such that d(~x, ~y) < M for all ~x, ~y A.
Proof.
() This direction is already hinted in the example above. We can show that it is bounded, then we can
put it in a box or a closed n-ball. Lets fix a point ~p in A. Since A is bounded, so there is M > 0 such that
d(~p, ~x) < M for all ~x A. Hence A is a the closed subset of closed n-ball Bd (~p, M) which is compact. The
claim follows from Theorem 4.5.
() This direction is new, but its easy. Lets cover A by open balls, namely, A ~xA Bd (~x,  x ). Since A
is compact, we can find a finite subcovering: A m
xi , i ). Now take arbitary ~p1 , p~2 A. There must
i=1 Bd (~
be some i1 and i2 such that ~p1 Bd (~xi1 , i1 ) and ~p2 Bd (~xi2 , i2 ). By the triangle inequality of the metric,
we have
d(~p1 , p~2 ) d(~p1 , ~xi1 ) + d(~xi1 , ~xi2 ) + d(~xi2 , ~p2 ) d(~xi1 , ~xi2 ) + 1 + 2 M
Pm
P
n
where M = i, j=1 d(~xi , ~x j ) + m
i=1 i . Thus A is bounded. It is certainly closed since R is closed and by
Theorem 4.8.

Remark 4.12. We can not generalized the above theorem to arbitrary metric spaces. Here is a counter example.
Let R be the metric space with Euclidean metric. It is easy to see that R {a} has induced metric whose topology
is the subspace topology of the standard topology. For a closed interval [b, c] containing a, is a compact space,
but [b, c] {a} is not compact. [b, c] {a} is closed in R {a} and bounded in the induced metric. Thus the
theorem fails for the metric space R {a}. This failure is related to the concept completeness of the metric.
R {a} is not a complete metric space but R is. See 45[Mun] to see more.
Exercise 4.13. Prove that every subspace of R with the finite complement topology is compact.

4.3

Metric

Definition 4.14 (20 [Mun]). A metric on a set X is a function d : X X R satisfing


1. d(x, y) > 0 for all x, y X and the equality holds iff x = y.
2. d(x, y) = d(y, x) for all x, y X.
3. d(x, y) + d(y, z) d(x, z) for all x, y, z X.
d(x, y) is often called the distance between x and y.
Definition 4.15. Let (X, d) be a set X with a metric d. For x X and  > 0, the -ball Bd (x, ) centered at x is
the subset of X given by
Bd (x, ) := {y X | d(x, y) < }.
The collection Bd := {Bd (x, ) | x X,  R>0 } is a basis of a topology called the metric topology. Check that
Bd satisfies the axiom (B1) and (B2) in Section 1. Call it the open ball basis. If (X, T ) is a topological space
and T can be realized as a metric topology, then (X, T ) is called metrizable.
19

Remark 4.16. A metrizable topological space is certainly a Hausdorff space. If x , y, then d := d(x, y) > 0 and
so d/2 > 0. Now Bd (x, d/2) and Bd (y, d/2) separate x and y.
Definition 4.17 (The Euclidean metric on Rn ). Let ~x, ~y Rn .
The inner product h~x, ~yi := x1 y1 + + xn yn .
p
The norm ||~x|| := h~x, ~xi.
The Euclidean metric d(~x, ~y) := ||~x ~y||.
Remark 4.18. The standard topology of Rn coincides with the metric topology given by the Euclidean metric.
Remark 4.19. There is another metric on Rn called the square metric given by
(~x, ~y) := max{|x1 y1 |, , |xn yn |}.
This is also a metric and its metric topology coincides with the standard topology.
Lemma 4.20. Let (X, T ) be a topological space and A X a subset. Let x X. If there is a sequence {xn } A
If X is metrizable, the converse holds, i.e. if x A,
there is {xn } that converges
that converges to x, then x A.
to x.
Proof. The first claim is trivial by the definition of convergence. Indeed, {xn } x if and only if U x , {xn } U x
is finite. Thus U x {xn } is infinite, therefore U x A , . For the second claim, choose a metric d so that T is
its metric topology. Let B := Bd be the open ball basis. Let Bx, := Bd (x, ). First apply Theorem 2.16 (b). We
have
x A Bx, B, Bx, A , Bx,1/n , n Z>0 , Bx,1/n A ,
where the second equivalence follows from the fact that  > 0, n such that 1/n < . Now pick xn Bx,1/n A.
Then the sequence {xn } A converges to x since Bx,1/n Bx,1/m if n > m.

Theorem 4.21. Let X, Y be topological spaces and f : X Y a map. The continuity of f implies that
(S 1) for every convergent sequence {xn } x in X, the sequence { f (xn )} converges to f (x).
On the other hand, if X is metrizable, (S 1) implies the continuity of f .
Proof. The first claim is Lemma 3.18. To prove the second claim, we will use Lemma 4.20 and HW3 (3):
f (A), A X. Let A be a subset of X and let x A.
By the second claim of
f : X Y continuous f (A)
Lemma 4.20, x A implies that there is a sequence {xn } x. By the assumption, { f (xn )} f (x). Then by the
f (A).
first claim of Lemma 4.20, f (x) f (A). Therefore f (A)


4.4

Limit Point Compactness and sequentially compact 28 [Mun]

Definition 4.22. Let X be a topological space and A a subset. Recall the following definitions:
x X is a limit(accumulation, cluster) point of A if x A {x}.
A sequence {xn X | n Z>0 } converges to x X if for every neighborhood U x of x, there is N Z>0
such that xn U x for all n > N.
Definition 4.23. Let X be a topological space.
X is limit point compact if, for any infinite subset A of X, there is a cluster point of A in X.
X is sequentially compact if every sequence {xn } in X contains a subsequence that converges to a point x
in X.

20

Theorem 4.24 (28.1 [Mun]). Compactness implies limit point compactness. Converse is not true in general.
Proof. Let X be a compact space. Suppose that X is not limit point compact. Let A be an infinite subset such
that there is no cluster point of A in X, i.e. x X, x < A {x}. By Theorem 2.16, there is an open set U x
of such that x U x and U x (A {x}) = . This implies that for every x X A, there is U x such that
x U x X A so that A is closed. Now consider the following open covering of X: {Ua }aA {X A} where
Ua satifies Ua (A {a}) = , i.e the element of A contained in Ua is only just a. Since X is compact, there is
a finite subcovering {Ua1 , , Uam , X A}. However the union of these can contain only finitely many elements
of A which contradict to the assumption that A is an infinite subset. Thus there must be an cluster point of A in
X.

Theorem 4.25 (28.2 [Mun]). If X is a metrizable topological space, then all three compactness of X are equivalent.
Proof.
(i) Compact Limit point compact: the previous theorem.
(ii) Limit point compact Sequentially compact: Let xn , n Z>0 be a sequence of points in X. If A := {xn }
is a finite subset of X, then there are infinitely many xn s that are the same point x X. Then the
subsequence {xn | x = xn } {xn } trivially converges to x. If A is an infinite subset, there is a cluster point
x of A in X, i.e. x A {x}. By Theorem 2.16, every neighborhood of x intersects with A {x}. Now
consider the sequence of open balls around x, namely Bd (x, 1) Bd (x, 1/2) Bd (x, 1/3) . Each
open ball must contain an element of A {x}, say xni . The subsequence {xni , i = 1, 2, } {xn } converges
to x apparently.
(iii) Sequentially compact Compact:
(1) Given  > 0, there is a finite covering of X by -balls.
Proof by contradiction: suppose that there is  > 0 such that there is no finite covering by -balls. We
will find a sequence that doesnt have a convergent subsequence. Choose x1 X. Bd (x1 , ) doesnt
cover X, so we can find x2 X Bd (x1 , ). Then we can still find x3 X Bd (x1 , ) Bd (x2 , ),
and in general, we can find xn+1 X (ni=1 Bd (xn , )), the finite collection {Bd (x1 , ), , Bd (xn , )}
never covers X. Thus we have a sequence {xn , n Z>0 }. This sequence doesnt have a convergent
subsequence because any /2-ball can contain at most one of xn s.
(2) Let A := {U } be an open covering of X. Then by the Lebesgue number theorem, there is > 0 such
that each open set of diameter less than is contained in one of open sets in A. Let  := /3. By (1),
we can cover X by finitely many -balls B1 , , Bm . Each open ball Bi has diameter 2/3 so it is less
than , therefore there is an open set Ui in A. Thus the finite subcollection {U1 , , Um } covers X.

Lemma 4.26 (27 [Mun] Lebesgue number lemma). Let X is a sequentially compact metric space with metric
d. Then for any open covering A := {U } of X, there is a positive number > 0 such that
? for each subset A of X with d(A) < , there is an open set U A such that A U.
The diameter d(A) of a subset A is defined by the least upper bound of the set {d(a1 , a2 ) | a1 , a2 A} A.
Proof. Proof by contradiction. Let {U } be an open covering of X such that there is no > 0 which satisfy ?. So
let Cn be a subset of diameter less than 1/n that is not contained in any of {U }. Let {xn , xn Cn } be a sequence.
Since X is sequentially compact, there is a subsequence {xni } which converges to a point a X. Since {U } is an
open cover, there is U containing a and there is Bd (a, ) such that a Bd (a, ) U . Since {xni } converges to
a, we can find a large ni such that xni Cni Bd (xni , /2) Bd (a, ) U which contradict the assumption that
Cni is not in any of {U }.

21

4.5

Completeness of metric and Theorem 4.11

Definition 4.27. Let (X, d) be a metric space. A sequence (xn , n Z>0 ) is a Cauchy sequence in (X, d) if
(Cauchy)

 > 0, N Z>0 such that d(xn , xm ) < , n, m > N.

A metric space (X, d) is complete if every Cauchy sequence in X converges.


Note that in a metric space, every convergent sequence must be Cauchy because of the definition of a convergent sequence (Definition 3.6).
Definition 4.28. A metric space (X, d) is totally bounded if  > 0, there is a finite covering of X by -balls.
Theorem 4.29 (45.1, [Mun]). A metric space (X, d) is compact if and only if it is complete and totally bounded.
Corollary 4.30. Let (X, d) be a complete metric space. A subset A is compact if and only if A is closed and A is
totally bounded.
Proof. If A is compact, then since X is Hausdoff, A is closed. It is bounded by the same argument as in the proof
of Theorem 4.11. On the other hand, if A is closed, then the induced metric is complete by Lemma 4.31. Thus
by Theorem 4.29.

Lemma 4.31 (p.269 [Mun]). Let (X, d) be a complete metric space. The induced metric on a closed set A is
complete.
Example 4.32.
Let A be a subset of Rn which is bounded w.r.t. the Euclidean metrix d. Then (A, d) is a metric space (with
the restriction of d) and it is totally bounded. Let M > 0 be the number which bounds A, i.e. d(x, y) < M
for all x, y A. Then there is a box [a, b]n where b a = M that contains A. Take arbitrary  > 0. We
can divide this box into a finite number of small boxes such that each can be contained in an epsilon ball.
Thus A can be covered by finitely many -balls.
The standard metric on Rn is complete (Theorem 43.2 [Mun]).

4.6

Universal properties of maps and induced topologies

Theorem 4.33. The induced topologies have universal properties:


Let j : A X be an injective map of sets. For every map f : Z X such that Im f Im j, there is the
unique map g : Z A such that j g = f :
Z@
@@ f
@@
!g
@@

 
/ X.
A
j
If X and Z are topological spaces and f is continuous, then the subspace topology on A makes g continuous.
For every pair of maps f1 : Z X and f2 : Z Y of sets, there exists the unique map g : Z X Y such
that 1 g = f1 and 2 g = f2 where 1 and 2 are the projections to the first and the second factors:
Z
!g


f2
X YE
EE
y
EE
yy
E
yy1
y
2 EE
y
E" 

|yy
X
Y
f1

22

If X, Y, Z are topological spaces and f1 , f2 continuous, then the product topology on X Y makes g continuous.
Let p : X Y be a surjective map of sets. For every map f : X Z that is constant on p1 (y) for each
y Y, there is the unique map g : Y Z such that g p = f :
X?
?? f
??
p
??


Y !g / Z
If X, Z are topological spaces and f is continuous, then the quotient topology on Y makes g contituous.

23

Connected spaces

5.1

Connected spaces: the concept

Definition 5.1. A topological space X is connected if X is not a disjoint union of non-empty open sets.
Lemma 5.2. The following are equivalent
(0) X is not a disjoint union of non-empty open sets
(1) X is not a disjoint union of non-empty closed sets
(2) The only subsets of X that are both open and closed are and X.
Proof.
(0)(2) If U1 is open and closed but not and not X, then U2 := X U1 is open and closed and not . Thus
we find the disjoint union X = U1 t U2 by open sets.
(0)(2) If X = U1 t U2 with U1 , U2 , , i.e. not connected, then U1 = X U2 is closed. Thus U1 is open and
closed which is not and not X since U2 , .
(0)(1) If X = U1 t U2 and U1 and U2 are non-empty and open, then U1 are U2 are nonempty and closed.
Similarly X = C1 t C2 and C1 and C2 are nonempty and closed, then C1 are C2 are nonempty open.

Example 5.3.
1. A set X with more than one element is not connected if we put the discrete topology.
2. Q R with the subspace topology induced from the standard topology is not connected as follows. Let
R be an irrational number. Then (, ) and (, ) are open sets of R and so U1 := Q (, ) and
U2 := Q (, ) are open sets of Q. Since < Q, Q = U1 t U2 .
3. Z R with the subspace topology induced from the finite complement topology is connected! We can
not have that Z = C1 t C2 where C1 and C2 are nonempty closed sets, since then C1 or C2 must be an
infinite set which can not be so (all closed sets are finite by definition). Likewise, Q R with the subspace
topology induced from the finite complement topology is connected!
4. We can generalize 3 to an infinite set with f.c topology.

5.2

Connected spaces: basic propeties

Again, it is not so easy to show some space is connected, like compact spaces. We need to find basic facts and
theorems with which we can show a space is connected.
Theorem 5.4 (23.5 [Mun]). The image of a connected space under a continuous map is connected. In particular,
if X  Y and X is connected, then Y is also connected.
Proof. Let f : X Y be a continuous map and X is connected. Since f : X f (X) is cotinuous, we can assume
f is surjective without loss of generality. Suppose Y is not connected, i.e. Y = U1 t U2 and U1 , U2 6 . Then
X = f 1 (Y) = f 1 (U1 ) t f 1 (U2 ). Since f is continuous, f 1 (U1 ) and f 1 (U2 ) are open and non-empty.

Theorem 5.5 (23.2 [Mun]). If X = U1 t U2 for some non-empty open sets and Y is a connected subspace, then
Y U1 or Y U2 .
Proof. If not, Y U1 and Y U2 are both non-empty. Thus Y = (Y U1 ) t (Y U2 ). Each Y Ui is open and
non-empty, so we have a contradiction to the fact that Y is connected.

Theorem 5.6 (23.3 [Mun]). Arbitrary union of connected subspaces that have a common point of a topological
space X is connected.
24

T
Proof. Let {A } is a collection of connected subspaces of X. Let a A be a common point. Suppose
S
A := A is not connected, i.e. A = U1 t U2 where U1 , U2 are non-empty open in A. If a U1 , A U1 for
all by Theorem 5.5. Thus A U1 which contradict the assumption that U2 is non-empty. The case a U2 is
similar. So A must be connected.

Theorem 5.7. A finite cartesian product of connected spaces is connected.
Proof. Let X and Y be connected and let (a, b) X Y. We have X {b}  X and {a} Y  Y, so by Theorem
5.4, they are connected. Now consider, for each x X,
C x := (X {b}) ({x} Y).
C x is connected because it is a union of connected subspaces that have a common point x b by Theorem 5.6.
Then X Y = xC x and (a, b) C x for all x X. So again by Theorem 5.6, X Y is connected.

Theorem 5.8. If A is a connected subspace of a topological space X, then the closure A is also a connected
subspace. Moreover, any subset B such that A B A is a connected subsapce.
Proof. Suppose B is not connected, i.e. B = U1 t U2 for some non-empty open sets U1 , U2 . By Theorem 5.5,
By
A U1 or A U2 . We can assume A U1 without loss of generality. Since U2 is non-empty, x U2 A.
Theorem 2.16, x A iff U x , U x A = . Since x U2 and U2 A = , we have a contradiction.

Exercise 5.9. Let p : X Y be a quotient map. Show that, if p1 (y) is connected for each y Y and Y is
connected, then X is connected.

5.3 R is connected
Theorem 5.10. R is connected.
Proof. Suppose that R = A t B where A and B are non-empty open sets. Let a A and b B. Consider
[a, b] = ([a, b] A) t ([a, b] B) =: A0 t B0 .
Since A0 is bounded by b, there is c := sup A0 .
Case c B0 . Since B0 is open in [a, b], there is  > 0 such that (c , c + ) B0 if c < b or (c , b] B0
if c = b. In either case, since c bounds A0 , (c , b] B0 which contradict that c is the smallest bounding
A0 .
Case c A0 . Since A0 is open in [a, b], there is  > 0 such that (c , c + ) A0 if a < c or [a, c + ) A0
if a = c. This contradicts with c bounding A0 .

Example 5.11. (a, b) R is connected. [a, b], (a, b], [a, b) are all connected. Rn is connected. All open/closed
boxes are connected. S 1 is connected. A torus S 1 S 1 is connected.
Proof. These follows directly from the same arguments in Theorem 5.10, but we can also derive them from the
basic properties of connected spaces. (a, b) is connected since it is homeomorphic to R. Then it follows from
Theorem 5.8 that (a, b), (a, b], [a, b) are all connected. Rn and all open/closed boxes are connected by Theorem
5.7. S 1 is connected since it is the image of a continuous map f : [0, 2] S 1 (x 7 (cos x, sin x)). By Theorem

5.7, S 1 S 1 is connected.

25

5.4

Path connected

Definition 5.12. A topological space X is path-connected if for each x, y X, there is a continuous map
f : [0, 1] X such that f (0) = x and f (1) = y. This map f is called a path from x to y.
Theorem 5.13. If a topological space X is path-connected, then it is connected.
Proof. Suppose that X is not connected, i.e. X = U1 t U2 for non-empty open sets U1 , U2 . Since [0, 1] is
connected by Example 5.11, if there is a path f : [0, 1] X, then [0, 1] C or [0, 1] D. So if we take x C
and y D, then there is no path from x to y, i.e. X is not path-connected.

Example 5.14 (disks). The unit n-disk Dn in Rn by the equation
Dn := {~x := (x1 , , xn ) Rn | ||~x|| :=

x12 + + xn2 1}.

Then Dn is path-connected and so by Theorem 5.13.


Proof. For given ~x, ~y Dn , there is a path f : [0, 1] Rn defined by
f (t) := (1 t)~x + t~y.

(5.1)

We can show that this path stays inside of Dn :


|| f (t)|| = ||(1 t)~x + t~y || ||(1 t)~x || + || t~y || = (1 t)|| ~x || + t|| ~y || (1 t)1 + t 1 = 1
The inequality follows from the triangle inequality of the standard metric Rn .

Example 5.15 (open balls). The unit open n-balls Bn is given by


Bn := {~x := (x1 , , xn ) Rn | ||~x|| :=

x12 + + xn2 < 1}

It is path-connected and so it is connected. Since Bn = Dn , this also implies that Dn is connected.


Proof. The proof is similar as in the case of n-disks. Define a path from ~x to ~y by
f (t) := (1 t)~x + t~y.
Then
|| f (t)|| = ||(1 t)~x + t~y || ||(1 t)~x || + || t~y || = (1 t)|| ~x || + t|| ~y || < (1 t)1 + t 1 = 1.

Remark 5.16. The closure of a connected space is connected, but the closure of a path connected space may not
be path connected (Example 5.18). So showing open balls are path connected (Example 5.15) is not sufficient to
show n-disks are path connected.
Lemma 5.17 (Exercise). If f : X Y is a continuous map and X is path-connected, then Im f is pathconnected.
Example 5.18 (Topologists sine curve). We give an example that is connected but not path-connected. Consider
the following subset of Rn :
S := {(s, sin(1/s)) R2 | 0 < s 1}.
It is connected because it is the image of the connected space (0, 1] under a continuous map. It is not so difficult
to show that the closure S is the union of S and I := {0} [1, 1]. By Theorem 5.8, S is connected. Below we
show that S is not path-connected.

26

Assume that there is a path f : [0, 1] S which connects the origin o and some point p S . Since
f 1 (I) is a closed set in [0, 1], we find b := sup f 1 (I) f 1 (I) by Lemma 5.19 and the restriction
f |[b,1] : [b, 1] S is continuous and satisfies f (b) = o and f (t) S for all t (b, 1]. Since [b, 1]  [0, 1],
we have a path g : [0, 1] S such that g(0) = o and g(t) S for t (0, 1].
Denote g(t) = (x(t), y(t)) where y(t) = sin(1/x(t)). For a given n, we can find u such that 0 < u < x(1/n)
and sin(1/u) = (1)n : Consider the sequence of points on S
!

!
1
1
n
,
sin
,
(1)
, n, m Z>0
+
n
+
2m
=

2
2 + n + 2m
2 + n + 2m
By taking m large enough, un := un,m =
un,m , m Z>0 converges to 0).

2 +n+2m

is less that x(1/n) since 0 < x(1/n) (use the fact that

By x : [0, 1] R is continuous, the intermediate value theorem implies that there is tn such that 0 < tn <
1/n and x(tn ) = un . Then tn converges to 0.
However ( f (tn ), sin(1/ f (tn ))) = (un , sin(1/un )) = (un , (1)n ) does not converge to the origin. This contradicts with the continuity of g by Lemma 3.18.
Lemma 5.19. Every closed set A of [0, 1] contains b := sup A, i.e. b A such that a b for all a A.
Proof. Since A is bounded above by 1, there is b := sup A. If b < A, then there is a small open interval
Ib := [b , b + ] such that A Ib by the fact that A = A and Theorem 2.16. Then b (/2) also bounds A, so it
contradict with that fact that b is the supremum of A.

Example 5.20 (unit n-sphere). Define the unit n-sphere S n in Rn+1 by
S n := {~x Rn+1 | ||~x|| = 1}
If n 1, it is path-connected. To show the path-connectedness, consider the continuous surjective map
~x
g : Rn+1 {~0}, g(~x) :=
.
||~x||
Observe that Rn+1 {~0} is path connected: for every two points ~x, ~y Rn+1 {0}, there is always a third point
~z Rn+1 {0} such that the straight lines (defined by the equation (5.1)) from ~x to ~z and from ~z to ~y that dont
go through the origin. Concatenating these lines, we have a path from ~x to ~y. Now the path-connectedness of S n
follows from 5.17.

5.5

Components, locally connected and locally path-connected

Definition 5.21. Let X be a topological space. A connected component of X is an equivalence class of the
equivalence relation given by
x y if there is a connected subspace containing x and y.
A path-connected component of X is an equivalence class of the equivalence relation given by
x y if there is a path connecting x and y.
Example 5.22. Let S be the topologists sine curve defined in Example 5.18. S is connected but not pathconnected. So the connected component is all of S but the path-connected components are {0} [1, 1] and
S.
Definition 5.23. Let X be a topological space.
27

X is locally connected if for every point x X and every neighborhood U of x, there is a connected
neighborhood V of x contained in U.
X is locally path-connected if for every point x X and every neighborhood U of x, there is a pathconnected neighborhood V of x contained in U.
Example 5.24. Here are examples which show that (path-)connectedness and local (path-)connectedness dont
imply each other.
(a) R {0} is not connected but locally connected. It is not path-connected but locally path-connected.
(b) The topologists sine curve is connected but not locally connected. Take an open ball B of radius less than
1 centered at the origin. Every open set in B that contains the origin is not connected.
(c) Consider the subspace X := {(x, 1/n) R2 | x R, n Z>0 } ({0} R) (R {0}) of R2 . Then X is
path-connected but not locally path-connected. It is also connected but not locally connected.

28

Topological manifolds and embedding into RN

Topological Manifolds 7, 30, 32, 33, 36 [Mun]

6.1

In this section, we introduce the nice topological spaces which underlies the most of the geometry, called the
topological manifolds. The algebraic topology we study has a well-developed theory on the topological manifolds. The main goal of this section is to prove that any topological manifold can be topologically embedded in
RN for some large N. In the proof, the partition of unity plays a key role.
Definition 6.1 (7,30). A topological space X has a countable basis if there is a basis B for the topology that
has only countably many open sets, i.e. B = {Un | n Z>0 }.
Definition 6.2 (36). A topological m-manifold is a Hausdorff space X with a countable basis such that each
point x X has a neighborhood that is homeomorphic to an open subset of Rm .
Definition 6.3 (36).
For a function : X R, the support of is defined by the closure of the preimage of R {0}:
Supp() := 1 (R {0}).
Let {U1 , , Un } be a finite open covering of a topological space X. Then the collection of functions
i : X [0, 1], i = 1, , n
is a partition of unity associated to the covering {Ui } if
(i) Supp(i ) Ui for all i = 1, , n.
P
(ii) ni=1 i (x) = 1 for each x X.
Lemma 6.4 (32). A compact Hausdorff space X satisfies the following condition (normality):
For every disjoint closed subsets A and B, there are open sets U A A and U B B such that U A U B = .
Proof. By Theorem 4.5, A and B are compact. For every a A, there are open sets Ua containing a and Va
containing B: for each b take disjoint open sets Ub containing a and Vb containing b, then {Vb } covers B so take
finite subcollection {Vbi }i=1, ,n . Then the union Va := ni=1 Vbi containing B and the intersection Ua := ni=1 Ubi
are the desired open sets. Now collect Ua s to form an open covering of A. We can make it into a finite collection
m
{Uai }i=1, ,m . Then m
i=1 U ai is an open set containing A which is disjoint from the open set i=1 Vai which contains
B.

Lemma 6.5 (Urysohn Lemma 33). Let X be a topological space that satisfies the normality. Let A and B are
disjoint closed subsets. Then there is a continuous function f : X [0, 1] such that f (A) = {0} and f (B) = {1}.
Theorem 6.6. Let X be a compact Hausdorff space and {Ui }i=1, ,n a finite open covering. Then there is a
partition of unity associated to {Ui }.
Proof.

(1) There is an open covering {Vi }i=1, ,n of X such that V i Ui : first, apply the normality of X to the
disjoint closed subsets
A := X (U2 Un ), B := X U1 .
Let U A and U B be the open sets separating A and B. Let V1 := U A . Then {V1 , U2 , , Un } covers X and
V 1 U2 . The next, apply the normality again for
A := X V1 (U3 Un ), B := X U2 .

29

Similary let V2 := U A . We have a cover {V1 , V2 , U3 , , Un } such that V 1 U1 and V 2 U2 . Similarly


for V3 , apply the normality for
A := X (V1 V2 ) (U4 Un ), B := X U3 .
We can keep doing these steps to replace all Ui s by desired Vi s.
i Vi . Using the Urysohns
(2) Apply (1) to {Vi } again to obtain another open covering {Wi } such that W
lemma, we find functions
i ) = {1} and (X Vi ) = {0}.
i : X [0, 1] such that (W
Observe that
Supp(i ) V i Ui .
Since {Wi } is a covering, (x) :=

Pn
i=1

i (x) > 0 for all x X. Define


i (x) :=

Then

n
X
i=1

i (x)
.
(x)
n

i (x) =

1 X
i (x) = 1
(x) i=1

and
Supp(i ) = Supp(i ) V i Ui .

Theorem 6.7. If X is a compact topological m-manifold, then X can be topologically embedded in RN for some
positive integer N.
Proof.
From the definition of manifolds and the compactness, we can find a finite open cover {Ui }ni=1 together
with imbeddings gi : Ui , Rm (homeomorphism to the image).
Since X is compact and Hausdorff, it satisfies the normality condition and hence we find a partition of
unity {i : X [0, 1]} associted to {Ui }. Let Ai := Supp(i ).
Define functions hi : X Rm by

i (x) gi (x)
hi (x) =

(0, , 0)

if x Ui
if x X Ai .

It is well-defined continuous functions (See Lemma 3.27).


The embedding of X into some RN is
m
m
F:XR
R
R}
| {z
}R
| {z
n times

n times

given by
F(x) := (1 (x), , n (x), h1 (x), , hn (x)).
It is continuous by Lemma 3.28. If F is injective, then F : X Im F is a continuous bijection. Therefore
by Theorem 4.9, it must be a homeomorphism.

30

P
Suppose F(x) = F(y) so that i (x) = i (y) and hi (x) = hi (y) for all i. Since i i (x) = 1, i (x) > 0 for
some i which also implies i (y) > 0. Thus x, y Ui . Now dividing hi (x) = h j (y) by the positive number
i (x) = i (y), we get gi (x) = gi (y). Since gi is injective, we have x = y.

Remark 6.8. The claim still holds even if a manifold X is not compact (p.225 [Mun]). If a manifold X is not
compact, we cant use Lemma 6.4. But to find the partition of unity, we need to show X is normal. We can
show that a manifold X satisfies the regularity, i.e. open sets can separate a point and a closed set. Then together
with the second-countability, i.e. there is countably basis, we can prove that X satisfies normality (Theorem 32.1
[Mun]). This explains why we include the second-countability condition in the definition of manifolds.

31

Group theory

Groups are very simple algebraic objects. It is a set with binary operations with very symmetric structures. Well,
it is the algebraic structure to keep track of symmetry of some geometric spaces after all.

7.1

Definition of groups and homomorphisms

Definition 7.1. A group (G, ) is a set G together with a map G G G, (a, b) 7 ab, called the multiplication,
satisfying
(Associativity) (a b) c = a (b c) for all a, b, c G.
(Identity) There is an element e G such that a e = e a = a for all a G.
(Inverses) For each a G, there an element a1 G such that a a1 = e = a1 a.
A subgroup H of a group (G, ) is a subset H of G such that the restriction H H H, (a, b) 7 a b makes H
a group. we write H G. A group G is called Abelian if the multiplication is commutative, i.e. a b = b a for
all a, b G.
Definition 7.2. A map : G H between groups is a group homomorphism if
(ab) = (a)(b).
If it is bijective, then the inverse map is automatically a group homomorphism (unlike the continuous maps!).
We need to show that 1 (x)1 (y) = 1 (xy):
(1 (x)1 (y)) = (1 (x))(1 (y)) = xy = (1 (xy)).
Since is injective, it follows that 1 (x)1 (y) = 1 (xy).

7.2

Examples

Example 7.3. (R, +) and (R := R {0}, ) are groups. Then Z, Q (R, +) and Q (R , ). However, Z {0}
is not a subgroup of R because the only elements of Z {0} that are invertible are 1.
Example 7.4. Let X be a finite set. The set S X of all bijections X X is a finite group where the multiplication
is given by the composition of maps. It is called a permutation group. If X := {1, , n}, the permutation group
in this case is often denoted by S n . If the cardinality of X is n, then S X  S n .
Example 7.5. Let X be a topological space. Then the set Aut(X) of all homeomorphisms from X to X itself is a
group where the multiplication is given by compositions.
Example 7.6. Consider U(1) = {ei = cos + i sin C | 0 < 2} C. Then U(1) is a subgroup of
(C , ) with respect to the multiplication. Indeed, ei1 ei2 = ei(1 +2 ) . We can actaully see that there is a group
homomorphism
exp : (R, +) U(1), x 7 eix .
k

This is surjective but not injective. Moreover, the subset {e2i n | k = 0, 1, , n 1} is a finite subgroup of U(1).
It is called a cyclic group of order n.
Example 7.7. Let Mat(n, R) be the set of all n n matrices. It is not a group with the matrix multiplication. But
inside of Mat(n, R), there are a bunch of groups:
GL(n, R) =

{M | det M , 0}

SL(n, R) =

{M | det M = 1}

O(n, R) =

{M | M M t = In }

SO(n, R) =

{M | M M t = In , det M = 1}

32

Similarly Mat(n, C) is not a group but inside there are a bunch:


GL(n, C) =

{M | det M , 0}

SL(n, C) =

{M | det M = 1}

U(n) =
SU(n) =

{M | M M = In }
t

{M | M M = In , det M = 1}

Example 7.8. A vector space V is a group with respect to the sum. So we can say a vector space is a group with
more structures. The linear map V W is a group homomorphism in this sense. Again, it has more structures.
The general linear group GL(n, R) is exactly the set of all invertible linear maps from Rn Rn .

7.3

More definitions

The most basic concept is the concept of the normal subgroup. It is defined as the subgroups such that the
quotient is naturally a group again. It is the important fact that the pre-image of the identity, called the kernel
of a homomorphism, is normal.
Definition 7.9. Let H be a subgroup of a group G. Let G/H be the quotient of G be the following equivalence
relation: x y if y = xh for some h. For every x G, the equivalence class of x is xH := {xh | h H} and there
is a bijection H xH sending h to xh.
Lemma 7.10. G/H has the induced group structure if xH H x for every x G.
Proof. The natural multiplication on G/H = {xH | x G} is
xH yH = xyH.
However, since x0 xH implies that x0 H = xH, we have to make sure that
xyH = x0 y0 H if x0 xH and y0 yH.
Since x0 = xh and y0 = yk for some h, k H, the right hand side is
x0 y0 H = xhykH = xhyH
since kH = H. Now if xH H x, x G, then for every h H, xh = h0 x for some h0 H. So hy = yh0 for some
h0 H. Thus
xhyH = xyh0 H = xyH.
This proves that the multiplication is well-defined. Now the identity is obviously 1H and the inverse of xH is
x1 H. Thus G/H is naturally a group.

Definition 7.11. A normal subgroup of a group G is a subgroup N such that xN N x, x G. If N is a normal
subgroup, then G/N is a group and the quotient map G G/N is a group homomorphism.
Lemma 7.12. Let : G H be a group homomorphism. The subset 1 (1H ) G is a normal subgroup of G.
Proof. It is a subgroup since (1G ) = 1H and (g1 g2 ) = (g1 )(g2 ) = 1H 1H = 1H for all g1 , g2 1 (1H ). To
prove it is normal, we compute
(g1 (1H )g1 ) = (g)(1 (1H ))(g1 ) = (g)(g1 ) = 1H , g G.
This means that g1 (1H )g1 1 (1H ) for all g G, which exactly means that g1 (1H ) 1 (1H )g for all
g G.


33

Definition 7.13. The pre-image of a group homomorphism : G H is called the kernel of , denoted by
ker := 1 (1H ).
Example 7.14. If G is an abelian group, then every subgroup is a normal subgroup.
Z is a normal subgroup of R with respect to +. The quotient R/Z is isomorphic to U(1). To see that,
consider the diagram:
exp
/ U(1)
RC
O
CC
CC
CC
C!
R/Z
The exponential map in Example 7.6 factors through R/Z and the resulting map f is a bijective homomorphism. So it is an isormophism as discussed in Definition 7.2.
For a positive integer p Z>0 , pZ := {pn | n Z} is a normal group of Z. The quotient Z/pZ = {i + pZ | i =
0, 1, , p 1} is a well-defined group. It is isomorphic to the cyclic group of order p defined in Example
7.6. Namely there is an isomorphism
k

Z/pZ {e2i p | k = 1, , p 1},

k + pZ 7 e2i p .

We can check that it is well-defined (i doesnt depend of the choice of representative k) and it is a bijective
homomorphism.
Example 7.15. A vector space is an abelian group with respect to the sum. A linear map is then a group
homomorphism. Let : V W be a linear map. Then ker V is a normal subgroup of V and V/ ker V is an
abelian group again. Of course!
In the group theory, it is easy to check the injectivity of a homomorphism by the next lemma.
Lemma 7.16. Let : G H be a group homomorphism. Then is injective if and only if ker is trivial, i.e.
ker = {1G }.
Proof. If is injective, it is obvious that ker = {1G }. Suppose that ker = {1G }. Let g1 , g2 G such that
1
(g1 ) = (g2 ). Then (g1 g1
2 ) = 1H , so g1 g2 = 1G , which implies g1 = g2 by multiplying g2 from both
sides.

Corollary 7.17. If : G H is a surjective homomorphism, then G/ ker G is isomorphic to H.
Proof. The homomorphism : G H naturally factors through G/ ker G H. This induced map has trivial
kernel. Thus it is injective. Since is surjective, the induced map is a bijective homomorphism. Thus it is an
isomorphism.


7.4

Groups given by generators and relations

Definition 7.18. Let {g1 , , gn } be a finite set. The free group hg1 , , gn i generated by {g1 , , gn } is the col1
lection of reduced finite words in {g1 , , gn , g1
1 , , gn }, including the empty word denoted by 1. reduced
means no element in word sits next to its inverse.
Example 7.19. A free group generated by one element is isomorphic to Z.
Definition 7.20. Let G := hg1 , , gn i be a free group. The list of relations in the free group is just a list of
reduced words W:={w1 , , wk } and we can form a normal subgroup by setting
N := the smallest normal subgroup containing W.

Then G/N is the group generated by {g1 , , gn } with the relations {w1 , , wk }. We denote this group as
hg1 , , gn | w1 = w2 = = wk = 1i.
34

Example 7.21. The group


ha, b | aba1 b1 = 1i
is isomorphic to Z2 .

7.5

Topological groups

Definition 7.22. A topological group G is a group G together with a Hausdorff topology such that
(i) m : G G G, (g, h) 7 gh is continuous.
(ii) i : G G, g 7 g1 is continuous.
Remark 7.23 (p.145 p.146 [Mun]). It is actually enough to assume that {1G } is closed. We can prove from this
assumption that G is Hausdorff. See Proposition 5.7, 5.8 [P].
Example 7.24. Every group can be considered as a topological group with a discrete topology.
Example 7.25. (R, +), (C, +), (R>0 , ), (R , ), (C , ) are topological groups with the topology induced from
the standard topology.
Example 7.26. GL(n, R) and GL(n, C) are topological groups with respect to the subspace topology induced
2
2
from the standard topologies of Rn and Cn . All the groups listed in Example 7.7 are topological groups with
respect to the subspace topologies, by the following lemma.
Lemma 7.27. Let G be a topological group. Every subgroup H G is a topological group with the subspace
topology.
Example 7.28. (Z, +), (Q, +) are topological groups with respect to the group structure restricted from (R, +)
and the subspace topologies. In particular, Z is a discrete group. U(1) is a subgroup of (C , ) and so it is a
topological group with respect to the subspace topology (it is homeomorphic to S 1 .)

7.6

Group actions on topological spaces

Definition 7.29 (Ex.31.8 [Mun]). A (continuous) action of a topological group G on a topological space X is a
continuous map : G X X, denoted by (g)x := g x, such that
(i) 1G x = x, x X.
(ii) (g1 g2 ) x = g1 (g2 x), g1 , g1 G, x X.
An orbit of the G-action on X is a subset O := {g x | g G} X. The relation defined by
x y if g x = y for some g G
is an equivalence relation and an equivalence class is nothing but an orbit. The quotient of X by the group
G-action is the quotient space X/G of X defined by this relation.

Definition 7.30. Let Homeo(X) be the set of all homeomorphism f : X X. Then f g := f g defines a group
structure on Homeo(X). Namely, the composition is a homeomorphism again. The identity is the identify map
and the inverse of f in this group is the inverse as a map. If there is an action of a topological group G on X,
then (g) : X X, x 7 gx is a homeomorphism and G Homeo(X), g 7 (g) is a group homomorphism.
Example 7.31. If G is a topological group and H a subgroup. There are two actions of H on G defined by
(h, g) 7 hg or (h, g) 7 gh1 . The quotient spaces H\G and G/H are called a homogeneous spaces. If N is
a closed and normal subgroup, then G/N = N\G and it has an induced group structure which makes G/N a
topological group. See also Ex 5 p.146 [Mun].

35

Example 7.32. Z acts on R by Z R R, (n, x) 7 x + n. The quotient R/Z is homeomorphic to a circle S 1 .


Z Z acts on R R by ((n, m), (x, y)) 7 (x + n, y + m) and the quotient R R/Z Z is homeomorphic to a torus
S 1 S 1.
Example 7.33. C acts on Cn+1 {~0} by (z0 , , zn ) := (z0 , , zn ). The quotient Cn+1 {~0}/C is the
complex projective space denoted by CPn . Consider S 2n+1 Cn+1 {~0} which is the set of unit vectors in
Cn+1 . Then the restriction of the quotient map to S 2n+1 is surjective. There is an induced action of the subgroup
U(1) C on S 2n+1 and the quotient S 2n+1 /U(1) is exactly CPn .
Example 7.34. R acts on Rn+1 {~0} by (x0 , , xn ) := (x0 , , xn ). THe quotient Rn+1 {~0}/R is the real
projective space denoted by RPn . Consider S n Rn+1 {~0} which is the set of unit vectors. Then the restriction
of the quotient map to S n is surjective. There is an induced action of the subgroup {1, 1} R on S n and its
quotient S n /{1, 1} is exactly RPn .
Example 7.35. There is an action of R on R2 by (x1 , x2 ) := (x1 , x2 ). The quotient R2 /R is not Hausdorff.
Example 7.36. Let G be GL(n, C). Let B be the closed subgroup of all upper triangular matrices in G. Consider
the action of B on G defined by the left multiplication. The quotient space G/B is called the flag manifolds.

36

Homotopy of Paths 51 [Mun]

From this week, we venture into algebraic topology. Algebraic topology is a branch of mathematics which
uses tools from abstract algebra to study topological spaces. The basic goal is to find algebraic invariants that
classify topological spaces up to homeomorphism, though usually most classify up to homotopy equivalence
Wikipedia. The most fundamental algebraic object we extract from a space is a group of loops. Here is how
we should start: Let X be a path-connected space and fix a point x0 X. Let L(X, x0 ) be the set of all paths
f : [0, 1] X from x0 to x0 , i.e. loops based at x0 . Define a multiplication in L(X, x0 ) as follows: for loops
f1 , f2 , define f1 f2 to be

x [0, 1/2]
f1 (2x)
( f1 f2 )(x) :=

f2 (2x 1) x [1/2, 1]
This operation doesnt make L(X, x0 ) a group. For example, the associativity of the product fails. To produce a
reasonable algebraic object out of this operation, we must pass it to the (path-)homotopy class. We actually start
by defined this operation in the collection of all paths.

8.1

Homotopy and Path Homotopy equivalence

Definition 8.1 (p323 [Mun]). Let f, g : X Y be continuous maps. f is homotopic to g, denoted by f  g, if


there is a continuous map F : X [0, 1] Y such that
F(x, 0) = f (x)

and

F(x, 1) = g(x).

This map F is called a homotopy between f and g.


Definition 8.2 (p.323 [Mun]). Let f, g : I X be paths from x to y where I := [0, 1]. f is path-homotopic to g,
denoted by f  p g, if there is a homotopy F : I [0, 1] X such that
F(0, s) = x

and

F(1, s) = y.

Remark 8.3. Let A X be a subset. A homotopy F : X [0, 1] Y between some maps is said to be relative
to A if F(a, t) is independent of t [0, 1] for each a A. A path-homotopy between paths f and g is nothing but
a homotopy relative to I = {0, 1}.
Lemma 8.4 (51.1 [Mun]).  and  p are equivalence relations.
Proof. Let f, g, h : X Y be continuous maps. Lets use a temporary notation: F : f g is a homotopy
F : X [0, 1] Y from f to g, i.e. F(x, 0) = f (x) and F(x, 1) = g(x)
(Reflexibity) The homotopy F(x, t) := f (x) makes F : f f .
(Symmetry) If F : f g, then G(x, t) := F(x, 1 t) makes G : g h.
(Transitivity) If F1 : f g and F2 : g h, then define

F(x, 2t)
G(x, t) :=

F(x, 2t 1)

t [0, 1/2]
t [1/2, 1]

By the pasting lemma, this is a well-defined X[0, 1] Y and such that G(x, 0) = f (x) and G(x, 1) = h(x),
so this makes G : f h.
These construction of homotopies for the axioms of an equivalence relation works for path homotopies, i.e. the
construction preserves the relativeness of the homotopies. Thus  p is also an equivalence relation.

Lemma 8.5 (Pasting lemma 18.3 [Mun]). Let A and B be closed subsets of X and let f : A Y and g : B Y
be continuous maps such that f |AB = g|AB . Then the map h : A B Y defined by h(x) := f (x) if x A and
h(x) := g(x) if x B is a wel-defined continuous map.
37

The following are basic facts about homotopy and path homotopy of maps and paths into Rn .
Lemma 8.6.
(1) Any two continuous maps f, g : X Rn are homotopic.
(2) A subspace A Rn is convex if the straight line segment between any ~a and ~b in A is contained in A. Any
two path f, g in A from ~x to ~y are path homotopic.
Proof.
(1) Define F : X [0, 1] Rn by

F(x, t) := (1 t) f (x) + tg(x).

This is a continuous map because it is a composition of the following maps.


X [0, 1]

( f,g,id)

/ Rn Rn [0, 1]

(1t)~x1 +t~x2

/ Rn .

Since F(x, 0) = f (x) and F(x, 1) = g(x), it is a homotopy from f to g.


(2) The same homotopy defined in (1) works as a path homotopy. Define F : I [0, 1] Rn by
F(s, t) := (1 t) f (s) + tg(s).
For each s = a, {F(a, t)|t [0, 1]} is a line segment from f (a) to g(a) so that F(s, t) A for all s, t I[0, 1].
Thus F : I [0, 1] A is a homotopy from f to g. By definition F(s, 0) = ~x and F(s, 1) = ~y, it is indeed
a path homotopy from f to g.


8.2

Definition of a product among paths and the homotopy invariance

Definition 8.7 (p.326 [Mun]). Let f : I X be a path from x0 to x1 and g : I X a path from x1 to x2 . Define
the product f g to be the path from x0 to x2 given by

for s [0, 1/2]


f (2s)
.
( f g)(s) :=

g(2s 1) for s [1/2, 1]


By the pasting lemma, f g is a well-defined continuous function from [0, 1] to X such that ( f g)(0) = x0 and
( f g)(1) = x2 . So it is a path from x0 to x2 .
Theorem 8.8 (51.2 [Mun]). Let f, g, h be paths in X. Let [ f ], [g], [h] be path-homotopy classes.
(0) induces a well-defined product on path-homotopy classes of paths.
(1) (Associativity) [ f ] ([g] [h]) = ([ f ] [g]) [h].
(2) (Identities) Let e x : [0, 1] X be a constant path. Then for every path f : [0, 1] X from x to y, we have
[ f ] [ey ] = [ f ]

and

[e x ] [ f ] = [ f ].

(3) (Inverse) For every path f : [0, 1] X from x to y, let f be a path given by f(s) := f (1 s). Then
[ f ] [ f] = [e x ] and [ f] [ f ] = [ey ].
Proof.

38

(0) We need to show if f  p f 0 and g  p g0 , then f g  p f 0 g0 , i.e. [ f ] [g] = [ f g] = [ f 0 g0 ] = [ f 0 ] [g0 ]


so that the product doesnt depend of the choice of the representatives.
For path-homotopies F : f p f 0 and G : g p g0 , define

s [0, 1/2]
F(2s, t)
H(s, t) :=

G(2s 1, t) s [1/2, 1]
Then H : I [0, 1] X is a well-defined continuous map by the pasting lemma and H(s, 0) = ( f g)(s)
and H(s, 1) = ( f 0 g0 )(s). Thus H : f g f 0 g0 .
(2) Consider paths in I: e0 : I I, e0 (s) = 0 and i : I I, i(s) = s. Then i is path homotopic to e0 i since I is
convex (Lemma 8.6 (2)). By Lemma 8.9 (1), f i and f (e0 i) are path homotopic. Since,
f = f i,

and e x f = ( f e0 ) ( f i) = f (e0 f ) by Lemma 8.9 (1),

We have f f  p e x . It follows from the similar argument that f f  p ey . we have f  p e x f . It follows


from the same argument that f  p f ey .
(3) We use the same paths e0 , i in I. i(s) = i(s). We have i i  p e0 because I is convex (Lemma 8.6 (2)). Thus
by Lemma 8.9 (1), f (i i)  p f (e0 ) are path homotopic. Since
f (i i) = ( f i) ( f i) = f f by by Lemma 8.9 (1) , and

f e0 = e x ,

(1) For every a, b [0, 1] such that 0 < a < b < 1, we define a triple product ( f g h)a,b : I X of paths
f, g, h : I X such that f (1) = g(0) and g(1) = h(0) as follows

f ( as )
s [0, a]

sa
( f g h)a,b (s) :=
g( ba ) s [a, b]

h( sb ) s [b, 1]
1b
This is a well-defined continuous map by the pasting lemma again. We can check ( f g) h = ( f g h) 14 , 12
and f (g h) = ( f g h) 21 , 34 . Thus we are done if we show ( f g h)a,b  p ( f g h)c,d for every pairs
a < b and c < d.
Consider a path p : I I whose graph is given by the three line segments (0, 0)(a, c), (a, c)(b, d),
(b, d)(1, 1). Then p is path-homotopic to i : I I, s 7 s since I is convex (Lemma 8.6 (2)). Let
F : I [0, 1] I be the path-homotopy from p to i. By Lemma 8.9 (1), ( f g h)c,d F is a
path-homotopy from ( f g h)c,d p to ( f g h)c,d i. Since ( f g h)c,d p = ( f g h)a,b and
( f g h)c,d i = ( f g h)c,d , we have ( f g h)a,b  ( f g h)c,d .

Lemma 8.9. Let f, g : I X be paths and let k : X Y be a continuous map.
(1) If F : f p g is a path homotopy, then k F : k f p k g is a path homotopy.
(2) If f (1) = g(0), then k ( f g) = (k f ) (k g).

39

Fundamental Group and Covering Spaces

Definition 9.1 (p.331 [Mun]). A loop bases at x0 X is a path from x0 to x0 . Let 1 (X, x0 ) be the set of pathhomotopy classes of loops based at x0 . It is a group by Theorem 8.8. It is called the fundamental group of X
based at x0 .
Since all the path defining a class in 1 (X, x0 ) has x0 as the initial and terminal points, the product
is defined for all pairs of classes. The associativity ([ f ] [g]) [h] = [ f ] ([g] [h]) follows from
Theorem 8.8 (1). The identity is [e x0 ] and for [ f ] 1 (X, x0 ), the inverse [ f ]1 is [ f].
Lemma 9.2. If a subspace A Rn is convex, then 1 (A, a0 ) is trivial for any a0 A.
Proof. By Lemma 8.6 (2), any loop f based at a0 is path-homotopic to the constant loop ea0 .

9.1

Non-canonical uniqueness of the fundamental group of a path-connected space.

Theorem 9.3 (52.1 [Mun]). Let x0 , x1 X and : [0, 1] X a path from x0 to x1 . Define a map
: 1 (X, x0 ) 1 (X, x1 ), [ f ] 7 []
[ f ] [].
Then is a group isomorphism.
Proof.

1. is a group homomrphism:
([
f ]) ([g])

= []
[ f ] [] []
[g] [] = []
[ f ] [g] [] = ([
f ] [g]).
The second equality follows from Theorem 8.8 (3).

2. To show that is an isomorphism, we show that there is an inverse homomorphism. Let := ,


then is
1 :
f ]) = []
[ f ] [] = [] [ f ] [].
([

f ])) = []
(
([
[] [ f ] []
[] = [ f ].
Similarly ([
f ]) = [ f ].

Remark 9.4. If X is path-connected, then fundamental groups based at all points are isomorphic. But there is
no natural isomorphism between way. The isomorphism depends on the path-homotopy classes of the chosen
path from x0 to x1 .
Definition 9.5. A space X is simply-connected if it is path-connected and 1 (X, x0 ) is a trivial group {1}.
Example 9.6. Any convex set A Rn is path-connected. Also 1 (A, a0 ) is trivial by Lemma 8.6 which states
that all paths with the same initial and terminal points are path-homotopic (so any loop at a0 is path-homotopic
to the constant look at a0 )
Lemma 9.7. If X is simply-connected, then for any x, y X, all paths from x to y are path-homotopic.
Proof. Let f and g be paths from x to y. Then f g is a loop based at x. Thus by the assumption and by Theorem
9.3,
[ f g ] (X, x) = {1}.
Thus [e x ] = [ f g ] = [ f ] [g]. Multiply [g] from right, we get [g] = [ f ] by Theorem 8.8 (2), (3).

40

9.2

Functoriality of fundamental groups

Theorem 9.8 (Functor from based spaces to groups). Let h : X Y be a continuous map. Then there is a group
homomorphism
h : 1 (X, x0 ) 1 (Y, h(x0 ))
defined by
h ([ f ]) := [h f ].
Proof.

1. (Well-defined) If f  p f 0 , then h f  p h f 0 by Lemma 8.9 (1).

2. (Group Homo) h ([ f ]) h ([g]) = [h f ] [h g] = [(h f ) (h g)] = [h ( f g)] = h ([ f ] [g]) where


the third equality follows from Lemma 8.9 (2).

Remark 9.9. The group homomorphism h induced from h : X Y depends on the choice of base points
x0 X and f (x0 ) Y. To make this association clear, we should introduce a map beween pairs: for a subspace
A X and B Y, a map h : (X, A) (Y, B) is a continuous map h : X Y such that h(A) B. In this way, we
have an associate without ambiguity:
h : (X, x0 ) (Y, y0 )

h : 1 (X, x0 ) 1 (Y, y0 ).

Theorem 9.10 (Functoriality).


1. If h : (X, x0 ) (Y, y0 ) and k : (Y, y0 ) (Z, z0 ), we have (k h) = k h .
2. If idX : (X, x0 ) (X, x0 ) is the identity map, then (idX ) : 1 (X, x0 ) 1 (X, x0 ) is the identity homomorphism.
Proof.
(k h) ([ f ]) = [k h f ] = k ([h f ]) = k h ([ f ])
(idX ) ([ f ]) = [idX f ] = [ f ].

Corollary 9.11. If h : (X, x0 ) (Y, y0 ) is a homeomorphism, then h : 1 (X, x0 ) 1 (Y, y0 ) is an isomorphism.
Proof. Let h1 be the inverse of h. Then h h1 = idY and h1 h = idX imply that
h (h1 ) = id1 (Y,y0 ) ,

(h1 ) h = id1 (X,x0 ) .

Thus (h )1 = (h1 ) and h is an isomorphism.

Definition 9.12. A category C consists of a collection Ob(C) of objects and, for each objects A, B, a collection
MorC (A, B) of morphisms from A to B. The following axioms must be satisfied:
1. (Composition of morphisms) For f MorC (A, B) and g MorC (B, C), there is a unique g f
MorC (A, C).
2. (Associativity) k (g f ) = (k g) f .
3. (Identity) For every object A Ob(C), there is the identity morphism idA MorC (A, A) such that f idA =
f and idB f for every g MorC (A, B).
A (covariant) functor F from a category C to D is an association
1. F : Ob(C) Ob(D).
2. F : MorC (A, B) MorD (A, B) for all A, B Ob(C).
41

satisfying F (g f ) = F (g) F ( f ) and F (idA ) = idF (A) .


Example 9.13. Let Topb be the category that consists of a topological space with a based point (X, x0 ) and
continuous maps f : (X.x0 ) (Y, y0 ). Let Grp be the category that consists of groups and homomorphisms.
Then the association
1. (On Objects) (X, x0 ) to 1 (X, x0 )
2. (On Morphisms) f : (X, x0 ) (Y, y0 ) to f : 1 (X, x0 ) 1 (Y, y0 )
defines a functor Topb to Grp.

9.3

Covering spaces and the example

Definition 9.14. A surjective continuous map p : E B is a covering map if it satisfies


For each b B, there is an open nbd Ub such that p1 (Ub ) = t V where p|V : V  Ub
We say, Ub is evenly covered by p for the underlined condition. E is called a covering space of B.
Lemma 9.15 (p.336 [Mun]). If p : E B is a covering map, then
1. For each b B, p1 (b) E has the discrete topology.
2. p is an open map. In particular, it is a quotient map.
Proof.

1. Let Ub be a nbhd of b evenly covered by p, i.e. 1 (Ub ) = t V and p : V  U. Then each V


contains exactly one element of p1 (b), thus it defines the discrete topology on p1 (b).

2. Let O be an open set in E. We need to show that p(O) is open. Let x p(O) and let U x be a nbhd of
x evenly covered by p, 1 (U x ) = tV . Since V O is open and p : V  U x , p(V O) is open and
x p(V O) p(O).

Theorem 9.16 (53.1 [Mun]). The map p : R S 1 defined by p(x) := (cos 2x, sin 2x) is a covering map.
Proof.

1. It is a continuous and surjective map.

2. Consider an open covering U1 , , U4 where U1 , U3 are right and left circles and U2 , U4 are upper and
lower half circles. We show that each Ui is evenly covered by p. Then since Ui covered S 1 , p is a covering
map.
We show it for U1 . Other Ui s are similarly proved. First
p1 (U1 ) =

G
1
1
(n , n + ).
4
4
nZ

Let Vn := (n 14 , n + 14 ). p|Vn : Vn U1 is obviously injective and surjective. Since Vn is


compact and U1 is Hausdorff, p|Vn is a homeomorphism by Theorem 4.9. Thus p|Vn : Vn U1 is a
homeomorphism too.

Theorem 9.17 (53.3). If p : E B and p : E 0 B0 are covering maps, then (p, p0 ) : E E 0 B0 B is a
covering map.
Proof. It is obvious that if U B is evenly covered by p and U 0 B0 is evenly covered by p0 , then U U 0 is
evenly covered by (p, p0 ).


42

Example 9.18. Consider the covering map p : R S 1 from Theorem 9.16. The above theorem says
p := (p, p) : R R S 1 S 1
is a covering map.
Theorem 9.19 (53.2 [Mun]). Let p : E B be a covering map. Let B0 B be a subspace, then let E0 :=
p1 (B0 ). Then p|E0 : E0 B0 is a covering map.
Proof. If U B is evenly covered by p, then U B0 B0 is evenly covered by p|E0 .

Remark 9.20. If you restrict p : E B to a subspace E0 E and define B0 := p(E0 ), then p|E0 : E0 B0
may fail to be a covering map. For example, consider E0 := (0, ) R in Theorem 9.16. p|E0 : E0 S 1 is a
surjective continuous map. However, for any open set U around (1, 0) S 1 , p1 (U) = (0, ) t (1 , 1 + ) t
and p|(0,) : (0, ) U can never be a homeomorphism.
Example 9.21. Consider p : R2 S 1 S 1 from Example 9.18 and let b0 := p(0) S 1 . Let B0 := S 1 {b0 }
{b0 } S 1 . This B0 is called the figure-eight. By Theorem 9.19, if E0 := p1 (B0 ), then p|E0 : E0 B0 is a
covering map. E0 is the infinite grid given by
E0 = (R Z) (Z R).
This is one covering space for the figure-eight and we will see others later.

43

Fundamental Groups and Covering Spaces, 54 [Mun]

10

In this section, we study the crucial connection between the concepts of fundamental groups and covering spaces.
This connection allows us to compute the fundamental group of spaces. The key concept to connect those two
is a lifting of a map along another map. As a first application, we compute the fundamental group of S 1 .
Step 1. Let p : E B be a covering. Let [ f ] 1 (B, b0 ). The loop f at b0 is uniquely lifted
to a path f in E once we choose where f starts.
Step 2. If [ f ] = [ f 0 ], the unique lifts f, f0 starting from the same point have the same ending.
Thus there is a map [ f ] 7 f(1).
Step 3. Apply it to p : R S 1 . We have a map 1 (S 1 , b0 ) 1 (b0 )  Z. The simple
connectedness R implies that this map is bijective. Moreover, we can show that this map is a
group homomorphism.

10.1

A lifting of a map along another map and liftings of paths along a covering map

Definition 10.1. Let p : E B be a continuous map. A lifting of a continuous map f : X B is a continuous


map f : X E such that p f = f :
?E
lifting f
p


/B

Example 10.2. Consider p : R S 1 , x 7 (cos 2x, sin 2x). Let f : [0, 1] S 1 , s 7 (cos x, sin x). Then
f : [0, 1] R, s 7 s/2 is a lifting of f . Also f : [0, 1] R, s 7 s/2 + 2 or in general s 7 s/2 + 2n
where n is a fixed integer, is a lifting. Observe that in this case, the lifting is determined by the initial point, i.e
by f(0) 2Z.
Lemma 10.3 (Unique Path Lifting for Covering 54.1 [Mun]). If p : E B is a covering map and f : [0, 1] B
is a path with the initial point b0 := f (0), then for each e0 p1 (b0 ), there is a unique lifting f : [0, 1] E such
that f(0) = e0 .
Remark 10.4. p1 (b0 ) is called the fiber of p at b0 . The above lemma says, each path in B with initial point b0
can be lifted uniquely to a path in E once we choose a point e0 in the fiber of b0 where the lifted path should
start from.
Proof.

1. Cover the image f (I) of the path by b f (I) Ub where Ub is evenly covered by p. Since f is continuous, { f 1 (Ub )} is an open cover of I. By Lebesgue Measure Lemma (note I is compact), there is > 0
such that any subset with max distance less than is contained in one of f 1 (Ub ). Therefore if we devide
I into ni=1 [si , si+1 ] in such a way that si+1 si < , then each [si , si+1 ] is contained in one of f 1 (Ub ), i.e.
f ([si , si+1 ]) Ub .

2. We construct f by induction on i = 1, , n:
(a) Lifting f |[0,s1 ] . Let f ([0, s1 ]) Ub1 . Since Ub1 is evenly covered, let p1 (Ub1 ) = tV . Let e0 V1 .
Since p|V1 : V1 Ub1 is a homeomorphism, the inverse (p|V1 )1 is continuous, therefore define
f|[0,s1 ] (s) := (p|V1 )1 f (s).
It is easy to see f|[0,s1 ] (0) = (p|V1 )1 f (0) = (p|V1 )1 (b0 ) = e0 and p f|[0,s1 ] = f .

44

(b) Suppose we have a desired lifting f|[0,si ] of f |[0,si ] . Let f ([si , si+1 ]) Ubi+1 . Let p1 (Ubi+1 ) = tV .
Since f (si ) = p f|[0,si ] (si ) Ubi+1 , there is Vi such that f|[0,si ] (si ) Vi . Define

s [0, si ]
f|[0,si ] (s)
f|[0,si+1 ] :=

1
(p|V ) ( f (s)) s [si , si+1 ]
i
By the argument in (a), the second map is a well-defined continuous map and by the way we chose
V , those two functions agree at s = si . Thus by the pasting lemma, f|[0,si+1 ] is a well-defined
continuous map. The conditions for lifting are obviously satisfied.
3. Uniqueness of lifting. Let f0 be another lifting. Induction on i.
0
(a) f|[0,s1 ] = f[0,s
: Since f(0) = f0 (0) = e0 , the images of both are in the same V1 since [0, s1 ] is
1]
connected so that the images must lie entirely in the connected component containing e0 . Thus

f|[0,s1 ] (s) = (p|V )1 f (s) = f0 |[0,s1 ] (s).


0
so that f(si ) = f0 (si ). Then the images of [si , si+1 ] under f and f0 are
(b) Suppose that f|[0,si ] = f[0,s
i]
both in Vi+1 because of the connectedness of [si , si+1 ] as in (a). Therefore similarly to (a), we have
f|[si ,si+1 ] (s) = f0 |[si ,si+1 ] (s). Thus together with the assumption, f|[0,si+1 ] = f0 |[0,si+1 ] .

10.2

Lifting Path-Homotopy

Theorem 10.5 (Unique Homotopy Lifting, 54.2, 54.3 [Mun]).


If p : E B is a covering map and F : I [0, 1] B be a continuous map with F(0, 0) = b0 . For each
0) = e0 .
e0 p1 (b0 ), there is a unique lifting F : I [0, 1] E such that F(0,
If F is a path-homotopy from f to g, then F is a path-homotopy from the lifting f of f at e0 to the lifting g
of g at e0 . In particular, f(1) = g (1).
Proof.
The argument to show there is a unique lifting is essentially the same as Theorem 10.3.
1. We can divide I [0, 1] into small rectangles [si , si+1 ] [t j , t j+1 ], 1 i n, 1 j m , so that the
image of each under F is contained in an evenly covered open set.
2. Number those rectangles by k = 1, , nm in such a way that consecutively numbered rectangles
share some points in the image under F. Then construct F inductively on k.
0) = F 0 (0, 0) = e0 . Then show the uniqueness also by
3. Let F 0 be another lifting such that F(0,
induction on k.
We have F(s, 0) = f (s), F(s, 1) = g(s), F(0, t) = f (0) = g(0) = b0 and F(1, t) = f (1) = g(1) = b1 .
t) is a lifting at e0 which must be a constant path
1. Regard F(0, t) as a constant path at b0 . Then F(0,
t) is
at e0 by the uniqueness of path lifting. Similarly regarding F(1, t) as a constant path at b1 , F(1,
a constant path at some point in p1 (b1 ). Thus F must be a path-homotopy.
0) and F(s,
1) are liftings of f and g at e0 , by the uniqueness, F is a path-homotopy from
2. Since F(s,
f to g .


45

Corollary 10.6. Let p : E B be a covering. Fix b0 and e0 p1 (e0 ). Define the following map of sets
e0 : { loops f at b0 in B } 1 (b0 ),

f 7 f(1).

where f is the unique lifting of f at e0 . This map factors through


e0 : 1 (B, b0 ) 1 (b0 ), [ f ] 7 f(1)
since if f  p f 0 , then f(1) = f0 (1) from the above theorem.

10.3

Surjectivity and bijectivity of e0

Theorem 10.7. Let p : E B be a covering map. Let e0 p1 (b0 ). (1) If E is path-connected, then e0 is
surjective. (2) If E is simply connected, i.e. path-connected and 1 is trivial, then e0 is bijective.
Proof.

1. Let e1 p1 (b0 ). Since E is path-connected, there is a path f from e0 to e1 . Composing with p,


we have a loop f := p f at b0 .

2. We need to show the map is injective if E is simply-connected. Let [ f ], [g] 1 (B, b0 ) such that e0 [ f ] =
e0 [g], i.e. if f, g are lifts of f, g beginning at e0 , then f(1) = g (1). By Lemma 9.7, there is a pathhomotopy F : f p g . Then F := p F is clearly a path-homotopy from f to g (check the conditions!)
so that [ f ] = [g].


10.4 1 (S 1 , b0 )  Z
Theorem 10.8. 1 (S 1 , b0 ) is isomorphic to Z.
Proof. Since R is simply-connected (Example 9.6), the map e0 : 1 (S 1 , b0 )  p1 (b0 ). Let b0 := (1, 0), then
p1 (b0 ) = Z R. If we can show that e0 is actually a group homomorphism, we are done. Let e0 := 0 R and
:= e0 . We need to show
([ f ] [g]) = ([ f ]) + ([g]).
Let f, g be the lifts of f, g at e0 . Let f(1) = n and g (1) = m so that ([ f ]) = n and ([g]) = m. Define a path
g 0 : I R by
g0 (s) := n + g .
Then g 0 is the lift of g at n so that f g 0 is well-defined. Since p ( f g 0 ) = (p f) p g 0 = f g by Lemma
8.9, we see that f g 0 is the lift of f g and ( f g 0 )(1) = n + g (1) = n + m. Thus ([ f ] [g]) = n + m.


10.5

Retraction and fixed points theorem

Definition 10.9. Let A be a subspace of X. A continuous map r : X A is a retraction of X to A if r(a) = a.


Lemma 10.10. Let i : A X be an inclusion of a subspace A of X. If there is a retraction r : X A, then
i : 1 (A, a0 ) 1 (X, a0 ) is injective. Furthermore r : 1 (X, a0 ) 1 (A, a0 ) is surjective.
Proof. Notice that r i = id(A,a0 ) . By Theorem 9.10 (2), r i = id1 (A,a0 ) . This implies that i is injective and r
is surjective.
Let f : X Y and g : Y X be maps of sets. If g f = idX , then f is injective and g is surjective.
Why? This is because, if f is not injective, then g f = idX can not be injective and if g is not

surjective, then g f = idX can not be surjective.
Theorem 10.11 (55.2, [Mun]). There is no retraction of B2 to S 1 where B2 is the 2-dimensional disk.

46

Proof. If there is a retraction, by Lemma , the inclusion j : S 1 B2 induces an injective map j : 1 (S 1 , x0 )


1 (B2 , x0 ). Since B2 is simply-connected (Lemma 9.6) and 1 (S 1 , x0 ), we have a contraction.

Theorem 10.12 (55.6, [Mun]). If f : B2 B2 is continuous, then there is x B2 such that f (x) = x.
Proof. Suppose that there is no such fixed point, i.e. f (x) , x for all x B2 . Then for each x, consider the half
line from f (x) to x. This line intersects with S 1 . Let this point be denoted by r(x) So define a map
r : B2 S 1 , x 7 r(x).
This map is well-defined because there is no fixed point. This map is continuous.
It is intuitively obvious. Any rigorous proof is welcome!
This map r is a retraction from B2 to S 1 and it contradict with Theorem 10.11.

10.6

Deformation retract 58

Lemma 10.13 (58.1). Let h, k : X Y be continuous maps and suppose h(x0 ) = y0 and k(x0 ) = y0 . If there is
a homotopy F : X [0, 1] Y from h to k such that F(x0 , t) = y0 for all t [0, 1], then h = k : 1 (X, x0 )
1 (Y, y0 ).
Definition 10.14 (p.361). Let A X be a subspace and let j : A , X be the inclusion map. A homotopy
H : X [0, 1] X is a deformation retraction of X onto A if
H(x, 0) = x,

H(x, 1) A, x X,

and H(a, t) = a, a A.

If wefine r : X A by r(x) := H(x, 1) A. Then r is a retraction. Furthermore, H is a homotopy from idX to


j r. In this case, A is called a deformation retract of X.
Theorem 10.15. If A is a deformation retract of X, then j : 1 (A, a) 1 (X, a) is an isomorphism.
Proof.

1. By Lemma 10.5, j is injective.

2. By Lemma 10.13 and definition of deformation retraction, ( j r) : 1 (X, a) 1 (X, a) is the identity
map. By ( j r) = j r , j must be surjective.

Example 10.16. S n is a deformation retract of Rn+1 ~0. Thus the inclusion j : S n Rn+1 ~0 induces an
isomorphism of the fundamental groups.
Let X := Rn+1 ~0. Consider H : X [0, 1] X defined by
H(x, t) := (1 t)x + tx/||x||.
It is a continuous map. Thus it is a homotopy. H(x, 0) = x and H(x, 1) = x/||x||. So if we define
r : X S n by r(x) := H(x, 1), then H is a homotopy from idX to j r. Since for all a S n ,
H(a, t) = (1 t)a + ta//||a|| = (1 t)a + ta = a, H is a deformation retraction.

47

Application and more computations of 1

11
11.1

Homotopy invariance of fundamental groups

Theorem 11.1 (Lemma 58.4). Let f : (X, x0 ) (Y, y0 ) and g : (X, x0 ) (Y, y1 ) be based continuous maps.
If h and k is homotopic, then there is a path from y0 to y1 such that h = k , i.e. the following diagram
commutes:
/ 1 (Y, y0 )
1 (X, x)K
KKKh
KKK

k KKK
% 
1 (Y, y1 ).
Proof. We will prove k ([ f ]) = h ([ f ]), i.e. [k f ] = [ (h f ) ], which is equivalent to [ (k f )] =
[(h f ) ]. Consider
I [0, 1]

/ [0, 1] [0, 1]

/ X [0, 1]

/ Y.

where
G is a path-homotopy between (s, 0) (1, s) (go right and then up 0 1 ) and (0, s) (s, 1) (go up and then
right 0 1 ).
F(s, t) := ( f (s), t). In particular, F(0, t) = F(1, t) = (x0 , t) =: c(t). Then c = F 1 = F 0 .
H is a homotopy from h to k.
Since H : X [0, 1] Y is a homotopy from h to k, H(x0 , 0) = h(x0 ) = y0 and H(x0 , 1) = k(x0 ) = y1 . Thus
:= H|{x0 }[0,1] is a path from y0 to y1 , i.e. (t) := H(x0 , t) = H c(t).
Then we show H F G is a path-homotopy between (h f ) and (k f ).
G : 0 1 p 0 1 implies F G : F (0 1 ) p F (0 1 ) since F is a continuous map. Thus
F G : (F 0 ) (F 1 ) p (F 0 ) (F 1 ).
H F G : (H F 0 ) (H F 1 ) p (H F 0 ) (H F 1 )
| {z } | {z }
| {z } | {z }
h f

k f


Definition 11.2. X and Y have the same homotopy type if there are maps f : X Y and g : Y X such that
f g  idY and g f  idX . In this case, f : X Y is called a homotopy equivalence and g is called a homotopy
inverse of f
Example 11.3. If A is a deformation retract of X, then the retraction map r : X A and the inclusion map
j : A , X are homotopy equivalences and A and X have the same homotopy type. To see this, consider
r : X A and j : A , X. Since r j = idA , obviously r j is homotopic to idA . The deformation retraction is
a homotopy from idX to j r.
Example 11.4.

1. The figure-eight is a deformation retract of R2 p q.

2. The theta figure S 1 (0 [1, 1]) is a deformation retract of R2 p q.


Theorem 11.5 (58.7). If f : X Y is a homotopy equivalence and f (x) = y, then f : 1 (X, x) 1 (Y, y) is an
isomorphism.

48

Proof. Let g : Y X be a homotopy inverse to f , i.e. there are homotopies f g  idY and g f  idX . Pick
some x0 X and consider the maps
(X, x0 )

/ (Y, y0 )

/ (X, x1 )

/ (Y, y1 ),

where y0 , x1 , y1 are consecutively picked as y0 := f (x0 ), x1 := g(y0 ) and y1 := f (x1 ). They induces
1 (X, x0 )

/ 1 (Y, y0 )

/ 1 (X, x1 )

/ 1 (Y, y1 ).

By Theorem 11.1, f g  idY and g f  idX implies


f g = (idY ) = ,

g f = (idX ) =

for some paths and . Since is an isomorphism, g is injective and f is surjective. Since is an isomorphism,
g is surjective and f is injective. Therefore, f and g are isomorphisms.

Corollary 11.6. If X and Y are path-connected and have the same homotopy type, then their fundamental groups
are isomorphic.

11.2

Fundamental group of S n

Theorem 11.7 (59.1). Let X = U V where U, V are open sets and let i : U , X and j : V , X be inclusions.
Suppose that U V is path-connected. Let x0 U V. Then images of
i : 1 (U, x0 ) 1 (X, x0 ) and

j : 1 (V, x0 ) 1 (X, x0 )

generate 1 (X, x0 ), i.e. any element is a product of elements in Im i Im j .


Theorem 11.8 (59.3). If n 2, S n is simply-connected.
Proof. Let S n := {~x Rn+1 , |~x| = 1} and ~p := (0, , 0, 1), ~q := (0, , 0, 1) S n . Let U := S n {~p} and
V := S n {~q}.
1. The stereographic projection is the map f p : U Rn defined by
(x1 , , xn , xn+1 ) 7

1
(x1 , , xn ).
1 xn+1

It is a homeomorphism because g : Rn U defined by


~y = (y1 , , yn ) 7

2y1
2yn
2
, ,
,1
2
2
1 + |~y|
1 + |~y|
1 + |~y|2

is the inverse of f . Thus U is path-connected and 1 (U, ~x0 ) is trivial. Similar for V.
2. The intersection U V = S n {~p, ~q} is path-connected, since f |UV : S n {~p, ~q} Rn ~0 is a homeomorphism and Rn ~0 is path-connected from Example .
3. Applying Theorem 11.7, 1 (S n , ~x0 ) is generated by the images of the fundamental groups of U and V. But
both of them are trivial, so 1 (S n , ~x0 ) is trivial. Since S n is path-connected by Example , S n is simplyconnected.


49

11.3

Fundamental theorem of algebra

Lemma 11.9. Let f : S 1 S 1 , z 7 zn where S 1 := U(1) C. Then f : 1 (S 1 , 1) 7 1 (S 1 , 1) is given by


[ f ] 7 [ f ]n = [ f ] [ f ].
Proof. Under the isomorphism in Theorem 10.8, we must prove f (1) = n: Since Z is generated by 1, it is enough
to show f (1) = n, i.e. f (m) = f (1 + + 1) = f (1) + + f (1) = m f (1) = mn. 1 Z is given by g : I S 1 ,
g(s) = cos 2s + i sin 2s = e2is since the lift g at 0 R is then given by g : I R, s 7 s. Now f g(s) = e2ns
and the lift of f g is ]
f g(s) = ns. Therefore f ([g]) = [ f g] = ]
f g(1) = n.

Lemma 11.10 (55.3). For a continuous map h : S 1 X, the following conditions are equivalent:
1. h is nullhomotopic, i.e. homotopic to a constant map.
2. h extends to a continuous map k : B2 X, i.e. if j : S 1 B2 is the natural inclusion, then h = k j.
3. h is the trivial homomorphism, i.e. h ([ f ]) = 1 for all [ f ] 1 (S 1 , b).
Proof.

(1 2) Let H : S 1 [0, 1] X be a homotopy from h to a constant map. Define a continuous


map : S 1 [0, 1] B2 by (b, t) := (1 t)b. Since is constant on S 1 {1} and injective elsewhere, H
factors through :
H
/X
S 1 [0, I1]
O
II
II
k
I
II
I$
B2
Since is a quotient map (*1), k must be a continuous map (*2). Since |S 1 {0} is the natural inclusion of
S 1 into B2 , k is an extension of h.
*1 Let 0 : S 1 [0, 1] S 1 / be the quotient map collapsing S 1 {1} to a point. Then factors through
0 , inducing a bijection j : S 1 / B2 , which is continuous from 2. Since S 1 / is compact and
B2 is Hausdorff, j is a homeomorphism. Thus must be a quotient map too.
*2 In general, if we have the diagram

X ? f / YO
??
?? h
g ??

Z

where f is continuous and g is a quotient map. Then h is a continuous map. We need to show that if
U Y is open, then h1 (U) is open. Since f 1 (U) = (h g)1 (U) = g1 (h1 (U)) is open and g is a
quotient map h1 (U) must be open.
(2 3) h = k j implies that h factors through
h : 1 (S 1 , b)

/ 1 (B2 , b)

/ 1 (X, h(b)).

Since B2 is convex, the middle term is trivial, so h must be a trivial homomorphism.


(3 1). Let p : R S 1 be the standard covering map used in Theorem 10.8. Then p|I : I S 1 is a
loop and represent 1 Z  1 (S 1 , b0 ). Since h is trivial, h p|I is path-homotopic to a constant loop at
x0 := h(b0 ). Let F : I [0, 1] X be the path-homotopy. F factors through p|I id[0,1] :
F
/X
O
JJ
JJ
JJ
H
p|Iid JJ
%
S1 I

I [0, 1]
J

50

Since p|I is a quotient map, the induced map H is continuous (*2). H is a homotopy from h to a constant
map:
F(s, 0) = h p|I (s) = H|S 1 {0} p|I (s) H|S 1 {0} (b, 1) = h(b).
x0 = F(s, 1) = H|S 1 {1} p|I (s)

H|S 1 {1} (b, 0) = x0 .


Theorem 11.11. A polynomial of degree n with coefficients in C has n roots (counted with multiplicities).
Proof. Let f (z) C[z] be a polynomial. Suppose the theorem below, say a is a root. Divide f (z) by (z a):
f (z)/(z a) = g(z) + R/(z a)
where g(z) is a polynomial of degree n 1 and R C is the remainder. Multiply (z a):
f (z) = g(z)(z a) + R.
Since f (a) = 0, R = 0. Thus f (z) = g(z)(z a). Keep this process n-times, we get f (z) = (z a1 ) (z an ).

Theorem 11.12. A polynomial of degree n with coefficients in C has at least one root.
Proof.
1. Since S 1 is a deformation retract of C 0, by applying Theorem 10.15 to j, we have j : 1 (S 1 , 1) 
1 (C ~0, 1). By Lemma 11.9, f : S 1 S 1 , z 7 zn induces f : 1 (S 1 , 1) 7 1 (S 1 , 1), [g] 7 [g]n which
is injective. Therefore k := j f : S 1 C 0, z 7 zn must induces an injective map k : 1 (S 1 , 1)
1 (C 0, 1) must be injective. Since 1 (S 1 , 1)  Z, k must be a non-trivial map. By Lemma 11.10, k is
not nullhomotopic.
2. We prove the claim in a special case: let g(z) = zn +an1 zn1 + +a1 z +a0 = 0 where |an1 |+ +|a0 | < 1.
Suppose that there is no root. Then regarding B2 as a disk in C, G : B2 C 0, r 7 g(z) is well-defined.
Since G|S 1 : S 1 B2 is a map extendable to B2 , by Lemma 11.10, G|S 1 is nullhomotopic.
3. Define a homotopy F : S 1 [0, 1] C 0 by F(z, t) := zn + t(an1 zn1 + + a0 ). It is well-defined since
F(z, t) , 0:
|F(z, t)| |zn | |t(an1 zn1 + + a0 )| 1 t|an1 zn1 + + a0 |
1 t(|an1 zn1 | + + |a0 |) 1 t(|an1 | + + |a0 |) > 0
The first inequality uses Remark 11.13 and the last strict inequality uses the condition |an1 |+ +|a0 | < 1.
This F is a homotopy from k to G|S 1 . Since G|S 1 is homotopic to a constant map (nullhomotopic), k must
be homotopic to a constant map too. But this contradict to (1).
4. Consider the general equation zn + an1 zn1 + + a1 z + a0 = 0. Let w = cz where c , 0. The equation
becomes
(cw)n + an1 (cw)n1 + + a1 (cz) + a0 = 0 wn +

a1
a0
an1 n1
w + + n1 w + n = 0
c
c
c

z = z0 is a root iff w = w0 is a root. By choosing a large c, the w-equation has a root by (3), therefore we
have a root for z-equation too.

Remark 11.13. For any complex numbers a, b, we have |a+b| |a||b|: apply triangle inequality to (a+b)+(b).
|a| = |(a + b) + (b)| |a + b| + | b| = |a + b| + |b|

51

|a| |b| |a + b|.

11.4

Fundamental group of torus

Theorem 11.14 (60.1). 1 (X Y, (x, y)) is isomorphism to 1 (X, x) 1 (Y, y).


Proof. Let p : X Y X and q : X Y Y be projection maps. Define a map
: 1 (X Y, (x, y)) 1 (X, x) 1 (Y, y),

[ f ] 7 ([p f ], [q f ]).

is a homomorphism:
([ f ] [g]) = ([ f g]) = ([p ( f g)], [q ( f g)]) = ([p f ] [p g], [q f ] [q g])
= ([p f ], [q f ]) ([p g], [q g]) = ([ f ]) ([g]).
Note that for given groups G and H, the natural group multiplication in G H is (g, h) (g0 , h0 ) := (gg0 , hh0 ).
( Surjective) Let ([g], [h]) 1 (X, x) 1 (Y, y). Defne [ f ] 1 (X Y, (x, y)) by f (s) := (g(s), h(s)). Then
([ f ]) = ([g], [h]).
( Injective) For a group homomorphism, if its kernel is trivial, then it is injective (Lemma 7.16). Let
[ f ] ker , i.e. there are path-homotopies G : p f p e x and H : q g p ey . We need to show
F : f  p e(x,y) . Define F : I [0, 1] X Y by F(s, t) := (G(s, t), H(s, t)). Then F is a path homotopy
from f to the constant loop e(x,y) :
F(s, 0) = (G(s, 0), H(s, 0)) = (g(s), h(s)),

F(s, 1) = (G(s, 1), H(s, 1)) = (e x (s), ey (s)) = e(x,y) (s).




Corollary 11.15 (60.2). 1 (T, b)  Z Z where T is the torus S 1 S 1 .

52

12

Fundamental groups of surfaces

12.1

Fundamental groups of a double torus

Theorem 12.1. The fundamental group of the eight figure is a free group generated by two elements.
Proof. The simply connected cover of the eight figure is given by rose.
http://en.wikipedia.org/wiki/Rose (topology).
It is an infinite tree graph, i.e. no loop in the graph. We want to show the rose is simple-connected. Since the
image of a path is compact so that it is contained in a finite graph, it is suffice to show that every finite tree graph
is contractible. Take a finite tree graph. First contract the edges that has a vertex with no other edges. Then keep
this process until there are no edges. By Theorem 10.7, there is a bijection between the fundamental group and
the fiber of the covering. Let f and g are loops for each circle. Then if you lift any two distinct words, the ending
points of them must be different. Thus there can not be a relation among the words in f and g.

In general, the fundamental group of a wedge of circles are known to be a free group
Theorem 12.2 (71.1). Let X be a union of circles S 1 , , S n where p is the only common point of circles. Then
1 (X, p) is a free group generated by f1 , , fn where fi is a generator of 1 (S i , p).
Definition 12.3. Let X1 and X2 be a topological surface. A connected sum X1 ]X2 is given by taking an open
disc from each Xi and pasting the remaining pieces along their edges. Note that here we have essentially two
choices of how we glue.
Corollary 12.4. Let T ]T be a double torus. Then 1 (T ]T, b) contains a free group generated by two elements
as a subgroup. In particualr, it is non-abelian group.
Proof. There is a retraction from T ]T to the eight figure (Figure 60.2 [Mun]). Thus by Lemma 10.5, the induced
map from the inclusion
j : 1 (, b) 1 (T ]T, b),


is an injective map.

12.2

Constructing various surfaces by identifying edges of polytopes

Let be an 2m-gon polytope with edges e1 , , e2m (numbered counter clockwisely). Create m-pairs among
{ei }, each labeled by a1 , , am . Orient the boundary of counter clockwise. Assign i := 1 to each ei . Orient
ei compatibly with the orientation on if  = +1. Orient oppositely if  = 1. All these information is written
on the right hand side of
(e1 , , e2m ) = (ai11 , , ai22mm ).
Now identify paired edges consistently with the orientation and obtain X := / . We have
Theorem 12.5 (74.1). X = / is a compact topological surface.
12.2.1

Fundamental group of surfaces constructed from polytopes

Theorem 12.6 (72.1). Let X be a Hausdorff space and A a closed path-connected subspace of X (with inclusion
i : A , X). If there is a continuous map h : B2 X such that h|Int B2 is a bijection onto X A and h|B2 : S 1 A
is a map into A. Then
i : 1 (A, a) 1 (X, a)
is surjective and the kernel is the least normal subgroup containing (h|B2 ) () where is a generator of 1 (S 1 , b)
with h(b) = a.

53

Remark 12.7 (Outline of how to compute the fundamental group of surface /). Apply the theorem to
the construction in the previous section. Let X = / and since B2  , regard as B2 in the theorem. Thus
h : X is the quotient map. Now  S 1 and let A := h(). Then A is the wedge of some circles with a
common point b. Let 1 , , n be generators of each circle in A. Thus by Theorem 12.2, 1 (A, b) is the free
group generated by 1 , , n . Let [] be the generator of presented by a counter clockwise loop. If we
consider the edges are counter-clockwise paths on the boundary of , then = e1 e2m . We can regard aikk
to be h ek . Thus h = ai11 , , ai22mm . Then 1 , , n are presented by compositions of h ek , so [h ] = 1
gives a relation among 1 , , n in the free group.
Torus, n-fold torus, RP2 , Klein Bottle

12.2.2

Theorem 12.8 (73.1). A torus T is given by the identification date (, , 1 , 1 ) for a 4-gon. We have
1 (T, a)  h, | 1 1 = 1i
1
1 1
Theorem 12.9 (74.3). An n-fold torus X = T ] ]T is given by (1 , 1 , 1
1 , 1 , , n , n , n , n ) for 4ngon. We have
1 (T, a)  h1 , , n , 1 , , n | [1 , 1 ] [n , n ] = 1i
1
where [i , i ] = i i 1
i i .

Theorem 12.10 (74.4). A n-fold projective space X = RP2 ] ]RP2 is given by 2n-gon with
(1 , 1 , , n , n ).
We have
1 (X, a) = h1 , , n | 21 2n = 1i.
Exercise 12.11 (EX3, p.454). The klein bottle K is given by 4-gon with (, , 1 , ). We have
1 (K, a) = h, | 1 = 1i

12.3

Properly discontinuous actions, covering spaces, and fundamental groups

Definition 12.12 (p.490 [Mun]). Let G be a discrete group continuously acting on X. The action is properly
discontinuous if
(PdC) x X, U x an open nbhd of x such that g(U) U = , g G with g , 1G .
Theorem 12.13 (81.5 [Mun]). Let X be path-connected, locally path-connected. Let G be a discrete group
continuously acting on X. Then the G-action is properly discontinuous if and only if the quotient map : X
X/G is a covering map.
Proof.
Suppose that the action is p.d. For x X, let U x be the nbhd of x in (PdC). Then
1 ((U x )) = {y X, y = gx0 , x0 U x , g G} = gG g(U x ) = tgG g(U x ).
This implies (1) (U x ) is an open nbhd of (x) because is a quotient map and the right hand side is
an open set, (2) 1 ((U x )) is a disjoint union of open sets. The restriction |g(U x ) : g(U x ) (U x ) is a
homeomorphism: it is continuous. It is bijective since otherwise it contradicts to the disjointness. The
inverse is continuous because is an open map: if U X is open, then 1 ((U)) = gG g(U x ) (not
necessarily disjoint), is a union open sets, so open.
Suppose that q is a covering map. ........exercise.
54


Example 12.14 (60.3-4). The antipodal quotient map S 2 RP2 is a covering map and 1 (RP2 , b)  Z/2Z.
Proof. RP2 = S 2 /Z2 where Z2 := {1, 1} acts on S 2 by 1 : x 7 x. This action is continuous because
R3 R3 , x 7 x is a homeomorphism. It is also a properly discontinuous action since the distance between x
and x is 2 (consider the metric topology induced from R3 and use the -ball to separate x and x). Thus the
quotient map 2 RP2 is a covering map. Since S 2 is simply-connected and the cardinality of a fiber is 2, by
Theorem 10.7, 1 (RP2 , b0 ) has cardinality 2. The group of cardinality 2 must be Z/2Z.

k

Example 12.15. Let S 3 := {(z, w) C2 , |z|2 + |w|2 = 1}. Let Z3 = {e2i 3 , k = 0, 1, 2} U(1). Define Z3 -action
on S 3 by : (z, w) 7 (z, 2 w). This action is properly discontinuous. Similarly to the argument in Theorem
12.14, 1 (S 3 /Z3 )  Z3 .
k

Example 12.16. Let S 5 := {(z, w, v) C2 , |z|2 + |w|2 + |v|3 = 1}. Let Z4 = {e2i 4 , k = 0, 1, 2, 3} U(1). Define Z4 action on S 5 by : (z, w, v) 7 (z, 2 w, 3 v). This action is properly discontinuous. As a set 1 (S 3 /Z3 )  Z4 .
But there are exactly two groups of cardinality 4. Z2 Z2 has also cardinality 4. The following theorem can be
proved by studying classifying spaces and the covering transformation.
Theorem 12.17 (7.3, p.151, [Bredon]). If X is simple-connected and locally path-connected and a discrete
group G acts on X properly discontinuously, then 1 (X/G, [x0 ])  G.
Proof. (outline)
1. Define the group of transformation (Deck transformations) of a covering p : E B: the group of
homeomorphisms f of E which satisfy p f = p.
2. We show that the group of transformation is isomorphic to the fundamental group of B if E is simplyconnected (Cor 81.4), [Mun].
3. We show that the group of transformation of X X/G is isomorphic to G.
4. We show that the quotient space of E by the action of the group of transformations is homeomorphic to B.


55

13
13.1

Cauchy Integral Formula, Jordan Curve Theorem and Winding Number Theorem
Cauchy Integral Formula 66 [Mun]

Definition 13.1. A complex analytic function is a complex valued function f (z) defined on an open set D in C
such that f is infinitely differentiable and the taylor series at z0 D converges to f (z) for z in the nbhd of z0 .
A holomorphic function is a complex valued function f (z) defined on D which is differentiable by z everywhere in D. It is exactly a function f (x + iy) = u(x, y) + v(x, y) such that partial derivatives of u and v are
continuous and satisfies the Cauchy-Riemann equations u x vy = 0, v x + uy = 0. A big theorem in complex
analysis is that complex analytic functions are exactly holomorphic functions.
There is some analogy between Greens theorem and the following theorems (Cauchy-Riemann equations
seems saying the the divergence and the circulation density of the vector field hv, ui is zero.)
Theorem 13.2 (Theorem 5, p.92, 1.2 [Ahlfors]). Let D be a simply connected open set in C.
(1) (Cauchys theorem) If h(z) is analytic in D then for any closed curve in D,
Z
h(z)dz = 0.

(2) If h(z) is analytic in D {z0 } and limzz0 (z z0 )h(z) = 0, then for any closed curve in D {z0 },
Z
h(z)dz = 0.

Now we define so-called winding numbers. It is a mathematically rigorous definition of how many times a
loop in C goes around a point a C. This number is crucial when we compute a line integral of a complex
analytic function.
Definition 13.3. Let : I R2 be a loop in R2 and let a R2 such that a < (I). Define a loop g : I S 1 R2
in S 1 by
(s) a
.
g(s) :=
|(s) a|
Consider the standard covering map p : R S 1 , t e2it . Take any lift g , then g (1) g (0) is always an integer.
Moreover it doesnt depend on the choice of lifting, because if g is a lifting, then the uniqueness of liftings
implies that other liftings are given by g (s) + m for some m Z. Define the winding number of around a R2
to be
n(, a) := g (1) g (0).
The following lemma is a consequence of an easy computation, although it is quite essential computation we
can apply to more general integrals.
Theorem 13.4 (Lemma 66.3). Let : I C be a piecewise differentiable loop in C. Let a C such that
a < (I). Then
Z
1
dz
n(, a) =
.
2i z a
Proof. Let g be a lifting of g, so that, for t I,
e2ig(t) = g(t) =

(t) a
.
|(t) a|

Let r(t) := |(t) a|, then


(t) = r(t) e2ig(t) + a, 0 (t) = r0 (t) e2ig(t) + r(t) 2ie2ig(t) g 0 (s).
56

!
Z 1
Z 1 0
dz
1
0 (t)
1
r (t)
0
=
dt =
+ 2ig (t) dt
2i 0 (t) a
2i 0 r(t)
za
"
#1
"
#1
1
1
=
log r(t) + 2ig(t) =
2ig(t) = g (1) g (0).
2i
2i
0
0

1
2i


As a corollary of above two theorems, we obtain the Cauchy Integral Formula which seems a bit insufficient
because of the appearance of the winding number.
Theorem 13.5 (Theorem 6, 2.2, p.95 [Ahlfors]). Let f (z) be a analytic function over an open disk D in C. Let
: I D C be a piecewise-differentiable loop in D. For a D such that a < (I), we have
Z
f (z)
1
dz = n(, a) f (a).
2i z a
Proof. The proof is basically the application of the previous theorem. Let
h(z) :=

f (z) f (a)
.
za

Then h(z) is analytic on D {a}. However


lim(z a)h(z) = lim f (z) f (a) = 0.
za

za

Thus for any closed curve in D {a},


Z

f (z) f (a)
dz = 0,
za

which, together with Theorem 13.4, implies the formula.

When n(, ) = 1, we have the classicall Cauchy Integral Formula,


Z
1
f (z)
dz = f (a).
2i z a
The really topological problem is when n(, a) = 1? To formulate the Cauchy Integral Formula without the
winding number, we need the following theorems in topology that seems trivial but is not easy to prove at all.
Theorem 13.6 (Jordan Curve Theorem). Let : I R2 be a loop such that (t) = (t0 ) iff t = t0 or t =
0, t0 = 1 (simple loop). Then R2 Im has two connected component, one is bounded and the other is bounded.
Furthermore, the boundary of each component coincides with Im .
Theorem 13.7 (Winding number theorem). If is a simple loop and a is a point in the bounded component of
R2 Im , then n(, a) = 1. We say orient the loop counter clockwise with respect to a if n(, a) = 1.
With these theorems, we have
Theorem 13.8. Let f (z) be an analytic function on an open region D. Then
Z
f (z)
dz = 2i
za
for all loops in D such that is counter clockwise around a and the bounded component of R2 Im is
contained in D.

57

13.1.1

Residue Theorem

Definition 13.9 (Wiki, Residues). The residue of an analytic function f at an isolated singularity a, denoted
R
Resz=a ( f, a) is the unique value R such that f (z) za
has an analytic antiderivative in a small punctured disk
0 < |z a| < . Alternatively, residues can be calculated by finding Laurent series expansions, and are sometimes
defined in terms of them.
Theorem 13.10 (Residue Theorem). Let f (z) be an analytic function on an open region D {a}. Then
Z
f (z)dz = 2i n(, a) Resz=a ( f, a)

for all loops in D and the bounded component of R2 Im is contained in D.

13.2

Jordan Curve Theorem

Jordan Curve Theorem states something we want to believe without proving it. Its amazing how complicated to
prove such a statement. However, on the other hand, there is the following theorem we havent prove:
Theorem 13.11 (Peano Space Filling Curve, 44). There exists a continuous map f : [0, 1] [0, 1] [0, 1]
such that Im f = [0, 1] [0, 1].
The point here is that the intuition is important to guide our investigation but it can not be trusted completely
unless it is proved.
Definition 13.12. A loop : I X is a simple loop if (t) = (t0 ) iff t = t0 or t = 0, t0 = 1. factors though the
standard map p : I S 1 , t 7 e2it and the induced map h : S 1 X must induces a homeomorphism S 1  Im .
Theorem 13.13 (25.5 [Mun]). If X is locally-path connected, then its connected components and its pathconnected components are the same.
Lemma 13.14. If is a simple loop, then S 2 Im is locally path-connected. Thus its connected components
and its path-connected components are the same.
Proof. S 2 is locally path connected. Any open set in a locally path connected space is locally path connected.
Thus the second clam follows from the theorem above.

Lemma 13.15.
(1) (61.1) Let C be a compact subspace of S 2 and b S 2 C. Let h : S 2 {b}  R2 be a homeomorphism,
which also induces a homeomorphism h : S 2 {b} C  R2 C. Consider a connected component
U S 2 C. Then h(U) is an unbounded connected component of R2 h(C) iff b U. In particular, the
unbounded connected component of R2 h(C) is unique.
(2) (61.2) Let A be a compact space and g : A R2 {~0} a continuous map. If ~0 lies in the unbounded
connected component of R2 g(A), then g is homotopic to a constant map.
(3) (Borsuk Lemma 62.2) Let a, b S 2 and A a compact space. If there is a continuous injective map
f : A S 2 {a, b} which is homotopic to a constant map, then a, b lie in the same connected component.
Proof.
(2) Consider a big ball B centered at ~0 in R2 so that g(A) is contained in B (its possible to take such a ball, since
g(A) is compact so the distance from g(A) is bounded). If ~p R2 B, then ~p must lie in the unbounded
component of R2 g(A). Thus ~p and ~0 lie in the same unbounded component. Let : I R2 g(A) be a
path from ~0 to ~p. Define
G : A [0, 1] R2 {~0}, G(x, t) := g(x) (t).
58

G(x, t) , ~0 follows from the fact that is a path in R2 g(A). G is a homotopy from g to k : A
R2 {~0}, k(x) = g(x) ~p. Now define
H : A [0, 1] R2 {~0, H(x, t) := tg(x) ~p.
It is a homotopy from a constant map to k. Thus we have a homotopy from g to a constant map.

Theorem 13.16 (Jordan separation theorem). If : I S 2 is a simple loop, then S 2 Im is not connected.
Proof.

1. By Lemma 13.14, it suffices to show that S 2 Im is not path-connected. Assume that it is


path-connected.

2. Im = A B where A and B are image of some paths and A B = {a, b}. Let U := S 2 A and V := S 2 B,
then
U V = S 2 Im .
Thus by the assumption, U V is path-connected, so we can apply the first van Kampen Theorem.
3. U V = S 2 {a, b} which is homeomorphic to R2 {~0}. So its fundamental group is isomorphic to
Z. On the other hand, we will prove that the inclusions U , U V and V , U V induce trivial
homomorphisms on fundamental groups so that, by the first van Kampen Theorem, U V has the trivial
fundamental group. This leads to a contradiction.
4. Let f : I U = S 2 A be a loop. It factors through the standard quotient map p : I S 1 , t 7 e2it . Let
h : S 1 S 2 A be the map such that h p = f . Let i : S 2 A , S 2 {a, b} and j : S 2 A , S 2 be the
natural inclusions. Then since i h(S 1 ) doesnt not intersect with A, we know that a and b are connected
by the path A so that they are in the same path-connected component of S 2 j h(S 1 ). We can now show
that i h : S 1 S 2 {a, b} is null-homotopic (*), so that it induces a trivial homomorphism by Lemma
11.10. Therefore
i h ([p]) = i ([h p]) = i ([ f ]) = 1.
5. (*) is basically Lemma 13.15 (1) and (2).

Theorem 13.17 (A non-separation lemma). If : I S 2 is a simple path, then S 2 Im has exactly one
component.
Proof. Since Im is contractible (*), idIm : Im Im is homotopic to a constant map. It implies that if
a, b S 2 Im , then the inclusion g : Im , S 2 {a, b} is homotopic to a constant map. By Lemma 13.15
(3), a and b are in the same component. Thus S 2 Im has no more than one connected component.
is simple if it is continuous injective map. Since I is compact and S 2 is Hausdorff, is a topological
embedding, i.e. I is homeomorphic to Im .
We can show there is at least one component, i.e. S 2 , Im . We know that Im is not homeomorphic to
S 2 by taking one point out, one is connected but the other is not. If Im , S 2 is a continuous injective
and both spaces are compact and Hausdorff, if surjective, then it must be homeomorphism. Constradiction.

Theorem 13.18 (63.1, converse to the first Seifert-van Kampen). Let X = U V where U, V are open sets in X.
Suppose U V = A t B where A, B are open sets.
(1) Let a A and b B. If is a path in U from a to b and is a path in V from b to a, then the loop f :=
at a generates an infinite cyclic subgroup of 1 (X, a).
59

(2) Let a, a0 A. If is a path in U from a to a0 and is a path in V from a0 to a, then the loop g :=
generates a subgroup of 1 (X, a) which intersect the subgroup generated by [ f ] trivially, i.e. h[g]ih[ f ]i =
{1}.
Theorem 13.19 (Jordan Curve Theorem). Let : I S 2 be a simple loop. Then S 2 Im has exactly two
connected components W1 and W2 . Furthermore, the boundary of each component coincides with Im .
Proof.

1. By Separation theorem, there are at least two (path-)connected components. Decompose Im into
two simple paths C1 and C2 with C1 C2 = {x, y}. Let Ui := S 2 Ci . By non-separation theorem, they
are connected. Then U1 U2 = S 2 Im .

2. We assume that S 2 Im has more than two connected components and derive a contradiction. Say A1 , A2
are two distinct components and B is the union of other components.
3. Let a A1 , a0 A2 and b B. Let be a path in U from a to a0 and in U from a to b. Let be a path in
V from a0 to a and in V from b to a. Consider the loops in U V at a, f : at a and g = .
4. By Theorem 13.18 (1) applied to U V = (A1 A2 ) t B, g generates an infinite cyclic subgroup of
1 (S 2 {x, y}, a) and by Theorem 13.18 (1) applied to U V = A1 t (A2 t B), f generates an infinite cyclic
subgroup of 1 (S 2 {x, y}, a).
5. Since 1 (S 2 {x, y}, a)  1 (R2 {0}, p)  Z, if s is its generator, then [g] = sm and [ f ] = sn . Thus
[g]n = [ f ]m which contradict with Theorem 13.18 (2).


13.3

Winding Number Theorem

Lemma 13.20 (65.2, Winding number theorem). ] Let C be a simple closed curve (the image of a simple loop)
in S 2 . If p, q S 2 lie in different components of S 2 C, then the inclusion map j : C , S 2 {p, q} induces an
isomorphism of fundamental groups. In other words, let C be a simple closed curve in R2 . If p lie in the bounded
component of R2 C, then the inclusion map j : S 2 , R2 p induces an isomorphism of fundamental groups.
Theorem 13.21. If is a simple loop and a is a point in the bounded component of R2 Im , then n(, a) = 1.
We say orient the loop counter clockwise with respect to a if n(, a) = 1.
Proof.
Without loss of generality, we can assume a = ~0 because n(, a) = n( a, ~0) and a is in the unbounded
component of R2 (I) if and only if 0 is in the unbounded component of R2 ( a)(I)).
: I R2 ~0 factors through the standard map p : I S 1 and induces a map h : S 1 R2 ~0 such that
h : S 1 (I) is a homeomorphism. Since [p] is a generator of 1 (S 1 , 0), h [p] is a generator of 1 (R2 ~0)
if ~0 is in the bounded component of R2 Im by Lemma 13.20. If ~0 is in the unbounded component of
R2 Im , then h is nullhomotopic by Lemma 13.15 so that h [p] is trivial by Lemma 11.10.
Consider the deformation retraction r : R2 ~0 S 1 , x x/|x|, then the induced map r : 1 (R2 ~0)
1 (S 1 ) is an isomorphism by Theorem 10.15. Then r [] = [r ] is a generator in the bounded case
and is trivial in the unbounded case. Since r (s) = (s)/|(s)|, n(, ~0) = 1 in the bounded case and
n(, ~0) = 0 in the unbounded case.


60

14

Classification of compact topological surfaces.

14.1

Triangulation

Definition 14.1. Let X be a compact topological surface. A curved triangle in X is a subset A in X together with
a homeomophism h : T A where T is a closed triangular region in R2 . A triangulation of X is a collection
{A1 , , An } of curved triangles such that
(Tr1) i Ai = X
(Tr2) For i , j, Ai A j = , a vertex or an edge of both.
(Tr3) If Ai A j is an edge, then h1
j hi on the corresponding edge of T i is linear.
Theorem 14.2 (c.f. [DM]). Every compact surface is triangulable.
Theorem 14.3 (78.1). If X is a compact triangulable surface, then X is homeomorphic to the quotient space
obtained from a collection of disjoint triangular regions in R2 by identifying their edges in pairs.
Proof. Let {A1 , , An } be a triangulation of X. Then consider the map : T 1 T n X where each T i maps
to X via hi and which is automatically a quotient map (a surjective map from a compact space E to a Hausdorff
space X is a closed map and so a quotient map: a closed set A E is compact since E is comact, the image of
a compact subspace is compact, a compact subspace in a Hausdorff space is closed.) There are following two
things to prove.
1. For each edge e of Ai , there is exactly one other A j such that Ai A j = e so that h1
j hi will identify the
corresponding edges of T i and T j in pairs.
2. There is no additional vertex identification, i.e. if Ai A j = v is a vertex, then there is a sequence
Ai = Ai1 , , Air = A j of triangles having v as a vertex such that Aik A jk+1 is an edge containing v.
See the proofs for these claims at page 472 - 475 [Mun].

Theorem 14.4 (78.2). If X is a compact connected triangulable surface, then X is homeomorphic to a space
obtained from a polygonal region in R2 by identifying the edges in pairs.
Proof. By the preceding theorem, we have a collection of triangular regions T 1 , , T n in R2 , together with the
oriented labels on the edges. Start with two triangles having the same oriented label on edges. By identifying
them, we have n 1 regions. Next take two distinct regions having the same label and identify them. Continuing
this process n 1 times, we have a single polygonal region with oriented labels on edges in pairs.


14.2

Classification of polygon quotients

Recall how to construct a surface out of a polygonal region in R2 .


Let be an 2m-gon polytope with edges e1 , , e2m (numbered counter clockwisely). Create mpairs among {ei }, each labeled by a1 , , am . Orient the boundary of counter clockwise. Assign
i := 1 to each ei . Orient ei compatibly with the orientation on if  = +1. Orient oppositely if
 = 1. All these information is written on the right hand side of
(e1 , , e2m ) = (ai11 , , ai22mm ).
Now identify paired edges consistently with the orientation and obtain X := / .

61

Definition 14.5 (Cut and Paste). A convex polygonal region can be cut along a line connecting two vertices and
decomposed into two pieces, provided the there are at least four vertices. Now the quotient space is obtained
by glueing two polygonal regions, that is, by pasting the cutting section as they were before cutting. This new
presentation do not change the resulting quotient. We should generalize the above construction to a collection of
polygonal regions with labeling, i.e. let P1 , , Pr be polygonal regions with 2m edges in total.
2m
(ai11 , , | , , | | , , ai2m
)
| {z } |{z}
| {z }

label on P1 labels on P2

label on Pr

Call this a labeling scheme for X.


Definition 14.6. Define the following elementary operation on labeling schemes:
2m
2m
1. (Cut) (ai11 , , ai2m
), m > 1 (ai11 , , c1 | c, , ai2m
)
2m
2m
2. (Paste) (ai11 , , c1 | c, , ai2m
) (ai11 , , ai2m
)

1 1
2m
3. (Relabel) (ai11 , , ai2m
) replace a label i by i , or by 1
= i ).
i ( so that (i )
2m
2m
2m1
4. (Permute) (ai11 , , ai2m
) (ai2m
, ai11 , , ai2m1
)

2 1
2m
2m
5. (Flip) (ai11 , , ai2m
) (a
i2m , , ai2 , ai1 )

6. (RelabelI) If ab and, ab or b1 a1 appear, we can relabel ab by c and b1 a1 by c1 after combining two


edges to one.
Note that 3,4,5 can be applied to the labeling on each Pi .
Theorem 14.7. The elementary operations dont affect the resulting quotient.


Proof. It might be just easy to picture them.

Theorem 14.8. If X is obtained from a polygonal region in R2 by identifying edges in pairs, then X is homeomorphic to one of the folloiwng: S 2 , T ] ]T , RP2 ] ]RP2 . Furthermore, those spaces in the list are all
non-homeomorphic to each other (non-homotopic to each other).
Proof. A labeling scheme can be transformed, via elementary operations, to one of the following:
(a, a1 , b, b1 ) (S 2 )
(a, b, a, b) (RP2 )
(a1 , a1 , , am , am ), m > 1 (RP2 ] ]RP2 )
1
1
1
(a1 , a1
1 , b1 , b1 , , an , an , bn , bn ) (T ] ]T )

The fundamental groups are given by the corresponding relations and they are all different (the number of
generators are different or the order of the generators are different).


62

14.3

Euler characteristic

Definition 14.9. The Euler characteristic of a triangulated surface X is defined by


(X) := V E + F
where V is the number of vertices, E is the number of edges and F is the number of triangles (faces).
Theorem 14.10. The Euler characteristic is independent of the choice of the triangulation.
Theorem 14.11. The Euler characteristics of the spaces in the list of the classification are
(S 2 ) = 3 3 + 2 = 2.
(RP2 ) = 2 3 + 2 = 1.
(RP2 ] ]RP2 ) = 2 m.
| {z }
m

(T ] ]T ) = 2 2n.
| {z }
n

Proof. In general, for two surfaces S 1 and S 2 , we have


(S 1 ]S 2 ) = (S 1 ) + (S 2 )

2
|{z}

+3
}3
| {z

one triangle glue 3 edges and 3 vertices

Thus, once we compute (S 2 ) = 2 and (RP2 ) = 1, we have


(RP2 ] ]RP2 ) = m 2(m 1) = 2 m
| {z }
m

(T ] ]T ) = (S 2 ] T ] ]T ) = 2 2n.
| {z }
| {z }
n

14.4

What happen to the Klein bottle?

The Klein bottle is given by the labeling scheme (a, b, a1 , b).


aba1 b

cut

/ abc|c1 a1 b

perm.

/ cab|c1 a1 b

f lip

/ cab|b1 ac
/ caac
paste

perm

/ aacc

Thus the Klein bottle is RP2 ]RP2 .


Exercise 14.12. What in the classification list corresponds to RP2 ]T ? Note that RP2 ]T is given by the labeling
scheme
(a, b, a, b, c, d, c1 , d1 ).

63

References
[Mun] Muncres, Topology, 2nd edition
[Set]

Basic Set Theory, http://www.math.cornell.edu/matsumura/math4530/basic set theory.pdf

[DK]

Lie Groups. Book.

[P]

Paulsen, AN INTRODUCTION TO THE THEORY OF TOPOLOGICAL GROUPS AND THEIR REPRESENTATIONS, www.math.uh.edu/ vern/grouprepn.pdf

[GGK] Ginzburg, Guillemin, Karshon, Moment Maps, Cobordisms and Group Actions
[Bredon] Bredon, Topology and Geometry, book
[Ahlfors] Ahlfors, Complex Analysis, book
[DM]

Doyle, Moran, A Short Proof that Compact 2-Manifolds Can Be Triangulated, Inventiones math., 5,
160-162 (1968), 3 pages.

64

MATH 4530 Topology. HW 1


Please declare any collaborations with classmates; if you find solutions in books or online,
acknowledge your sources in either case, write your answers in your own words.
Please attempt all questions and justify your answers.

(1) (Set theory) Let R be the set of real numbers. Consider the map f : R R defined by f (x) := x2 .
Find a maximal subset M of R such that the restriction map f | M is injective, i.e. find a subset M
of R such that f | M is injective and and there is no subset of R containing M properly.
(2) (Set theory) Let g : X Y be a map. Let A X and B Y be subsets. If g is injective, then we
can show that A = g1 (g(A)). The proof goes as follows:
To prove the equality, we need to show that (a) A g1 (g(A)) and (b) g1 (g(A)) A.
(a) Let a A. We need to show that a g1 (g(A)). By the definition of pre-image, we have
a g1 (g(A))

if and only if

g(a) g(A).

The RHS is obviously true and hence the LHS, a g1 (g(A)), is true.
(b) Let c g1 (g(A)) X and then we need to show c A. Again by def of pre-image,
c g1 (g(A))

g(c) g(A).

RHS means that there is a A such that g(a) = g(c). However, by the injectivity of g, we have c = a.
Thus c A.

Now prove that, if g is surjective, then g(g1 (B)) = B.


(3) (Set theory) Let f : X Y be a map and let A1 , A2 X be subsets.
(3.1) Prove the following
(a) f (A1 A2 ) f (A1 ) f (A2 ).
(b) f (A1 A2 ) f (A1 ) f (A2 ); the equality holds if f is injective.
(3.1) Define a map f : Z {0, 1} by sending even integers to 0 and odd integers to 1. Find subsets
A1 and A2 of Z such that f (A1 A2 ) , f (A1 ) f (A2 ).
(4) (Topology) A discrete topology on a set X is given by the collection T of all subsets. Prove that
it satisfies all axioms.
(5) (Topology) Let R be the set of real numbers. Define open sets in R by subsets that are complement
of finite subsets or the whole set R. Check that it defines a topology in R (known as the finite
complement topology).
(6) (Topology) Show that R {1/n | n = 1, 2, } is not an open set in the standard topology of R.
You are only allowed to use the materials from Section 1 of the lecture notes.
(7) (Topology) Let (X, T ) be a topological space. Prove that a collection B of open sets is a basis for
T if and only if for every U T and x U, there is B B such that x B U.
References
[M]
[S]
[L]

Munkres, Topology.
Basic Set Theory, http://www.math.cornell.edu/matsumura/math4530/basic set theory.pdf
Lecture Notes, available at http://www.math.cornell.edu/matsumura/math4530/math4530web.html

MATH 4530 Topology. HW 1 Solution


Please declare any collaborations with classmates; if you find solutions in books or online,
acknowledge your sources in either case, write your answers in your own words.
Please attempt all questions and justify your answers.

(1) (Set theory) Let R be the set of real numbers. Consider the map f : R R defined by f (x) := x2 .
Find a maximal subset M of R such that the restriction map f | M is injective, i.e. find a subset M
of R such that f | M is injective and and there is no subset of R containing M properly.
Solution: R0 = {x R | x 0} or R0 = {x R | x 0}.
(2) (Set theory) Let g : X Y be a map. Let A X and B Y be subsets. If g is injective, then we
can show that A = g1 (g(A)). The proof goes as follows:
To prove the equality, we need to show that (a) A g1 (g(A)) and (b) g1 (g(A)) A.
(a) Let a A. We need to show that a g1 (g(A)). By the definition of pre-image, we have
a g1 (g(A))

if and only if

g(a) g(A).

The RHS is obviously true and hence the LHS, a g1 (g(A)), is true.
(b) Let c g1 (g(A)) X and then we need to show c A. Again by def of pre-image,
c g1 (g(A))

g(c) g(A).

RHS means that there is a A such that g(a) = g(c). However, by the injectivity of g, we have c = a.
Thus c A.

Now prove that, if g is surjective, then g(g1 (B)) = B.


Solution: Let b B, then clearly by subjectivity b g(g1 (B)), so B g(g1 (B)). Now let
c g(g1 (B)), then c = g(a) for some a g1 (B) and so c = g(a) B, which gives the other
inclusion.
(3) (Set theory) Let f : X Y be a map and let A1 , A2 X be subsets.
(3.1) Prove the following
(a) f (A1 A2 ) f (A1 ) f (A2 ).
(b) f (A1 A2 ) f (A1 ) f (A2 ); the equality holds if f is injective.
(3.1) Define a map f : Z {0, 1} by sending even integers to 0 and odd integers to 1. Find subsets
A1 and A2 of Z such that f (A1 A2 ) , f (A1 ) f (A2 ).
Solution:
(3.1) (a) Let y f (A1 ) f (A2 ). If y f (A1 ), then x A1 such that f (x) = y. Since x A1 , we
have x A1 A2 , so y f (A1 A2 ). Same argument works if y f (A2 ).
(b) Let y f (A1 A2 ). Then x A1 A2 such that f (x) = y. Since x A1 and x A2 ,
we get y f (A1 ) f (A2 ).
Now assume f is injective and let y f (A1 ) f (A2 ). Since f is injective there is a
unique pre-image of y, say f (x) = y. Then x A1 and x A2 and so y f (A1 A2 ).
(3.2) The key is that f is not injective so that two points can go to the same point under f . For
example, 0 and 2 go to 0 under f . Let A1 = {0} and A2 = {2}. See that this gives an example
such that f (A1 A2 ) , f (A1 ) f (A2 ).
1

(4) (Topology) A discrete topology on a set X is given by the collection T of all subsets. Prove that
it satisfies all axioms.
Solution: Clear by definition of the power set.
(5) (Topology) Let R be the set of real numbers. Define open sets in R by subsets that are complement
of finite subsets or the whole set R. Check that it defines a topology in R (known as the finite
complement topology).
Solution:
(T1) R = C T and = RC T .
(T2) Let Ui T and say UiC = Vi for all i. Note that Vi are finite subsets or the whole set R. Let
S
S
T
T
U = Ui and let V = U C = ( Ui )C = (UiC ) = Vi . If all of the Vi were R, then V = R
and U T , otherwise some Vi is finite and so V is finite, giving U T .
(T3) Again let Ui T and say UiC = Vi for 1 i n. Again Vi are finite subsets or the whole set
S
S
T
T
R. Let U = n1 Ui and let V = U C = ( n1 Ui )C = n1 (UiC ) = n1 Vi . If any of the Vi were R,
then V = R and U T , otherwise V is finite since we are taking finite union of finite sets and
hence U T .
(6) (Topology) Show that R {1/n | n = 1, 2, } is not an open set in the standard topology of R.
You are only allowed to use the materials from Section 1 of the lecture notes.
Solution: Let A := R {1/n | n = 1, 2, }. Notice that 0 A. To show A is open, we
need to prove the condition (G1) in Lemma 1.5. [L]. The standard topology is given by B =
{ all open intervals} (Example 1.7 [L]). Take any open interval containing 0, say (a, b) where
a < 0 < b. We can find n such that 0 < 1/n < b, so that 1/n (a, b). Thus any interval (a, b) is
not a subset of A.
(7) (Topology) Let (X, T ) be a topological space. Prove that a collection B of open sets is a basis for
T if and only if for every U T and x U, there is B B such that x B U.
Solution: One direction is the content of Lemma 13.2 of [M]. Please see the proof written
there. The other direction is the straightforward consequence of the condition (G1) in the Lemma
1.5 of [L].
R
[M]
[S]
[L]

Munkres, Topology.
Basic Set Theory, http://www.math.cornell.edu/matsumura/math4530/basic set theory.pdf
Lecture Notes, available at http://www.math.cornell.edu/matsumura/math4530/math4530web.html

MATH 4530 Topology. HW 2


Please declare any collaborations with classmates; if you find solutions in books or online,
acknowledge your sources in either case, write your answers in your own words.
Please attempt all questions and justify your answers.

(1) Let K be a collection of subsets of X satisfying


(C1) , X K.
(C2) Any intersection of subsets in K is also in K.
(C3) Any finite union of subsets in K is also in K.
Then show that
(a) T := {X C | C K} is a topology, i.e. it satisfies (T1,2,3).
(b) If T is a topology, K := {X U | U T } satisfies (C1,2,3).
(2) Let Y be a subspace of a topological space (X, T ). Prove that, if A is a closed subset of Y and Y is
a closed subset in X, then A is a closed subset of X.
(3) Let Cn be the circle of radius 1n in R2 , centered at the origin. Let

[
A :=
Cn .
n=1

Determine whether or not A is a closed subset of R2 with the standard topology.


(4) Let A, B, Ai denote subsets of a topological space X. Then

(a) Prove that A B = A B.


(b) Prove that or find a counterexample for (i.e. disprove)
[
[
Ai =
Ai .
iI

iI

(c) Prove or disprove that A B = A B.


(d) Prove or disprove that
\
\
Ai =
Ai .
iI

iI

Please note that = means that and . So, in the above problems, prove or disprove those two
inclusions separately.
(5) The boundary A of a subset A of a topological space X is defined by
A := A Int A.
From this definition, it follows that A is the disjoint union of A and Int A, i.e. A = A Int A and
A Int A = .
Find the boundary, the closure and the interior of the following subsets in the specified topology:
(a) (0, 1] in R with the standard topology.
(b) (0, 1] in R with the finite complement topology.
(c) Q in R with the standard topology.
References
[M]
[S]
[L]

Munkres, Topology.
Basic Set Theory, http://www.math.cornell.edu/matsumura/math4530/basic set theory.pdf
Lecture notes, available at http://www.math.cornell.edu/matsumura/math4530/math4530web.html

MATH 4530 Topology. HW 2 Solutions


Please declare any collaborations with classmates; if you find solutions in books or online,
acknowledge your sources in either case, write your answers in your own words.
Please attempt all questions and justify your answers.

(1) Let K be a collection of subsets of X satisfying


(C1) , X K.
(C2) Any intersection of subsets in K is also in K.
(C3) Any finite union of subsets in K is also in K.
Then show that
(a) T := {X C | C K} is a topology, i.e. it satisfies (T1,2,3).
(b) If T is a topology, K := {X U | U T } satisfies (C1,2,3).
Solution:
(a) (T1) = X X T , X = X T by (C1).
(T2) Let U[
say Ui = X Ci for
i T and[
\all i I, Ci K.
Then
Ui =
(X Ci ) = X
Ci T , by (C2).
iI

iI

iI

i=1

i=1

i=1

(T3) Let Ui T and say Ui = X Ci for 1 i n, Ci K.


n
n
n
\
\
[
Then
Ui =
(X Ci ) = X
Ci T , by (C3).
(b) Let T be a topology satisfying (T1), (T2), (T3) as usual.
(C1) = X X K, X = X K by (T1).
(C2) Let Ci\
K and\
say Ci = X Ui for
[all i I, Ui T .
Then
Ci =
(X Ui ) = X
Ui K, by (T2).
iI

iI

iI

i=1

i=1

i=1

(C3) Let Ci K and say Ci = X Ui for 1 i n, Ui T .


n
n
n
[
[
\
Then
Ci =
(X Ui ) = X
Ui K, by (T3).
(2) Let Y be a subspace of a topological space (X, T ). Prove that, if A is a closed subset of Y and Y is
a closed subset in X, then A is a closed subset of X.
Solution: Since Y is closed in X, there is an open set V T such that X Y = V. Since A is
closed in Y so that Y A is open in Y, there is an open set U T such that Y A = Y U by
definition of subspace topology. Then X A = V U (?) which implies A is closed. Note: To see
(?) is true, draw the Venns diagram and locate the regions of X, A, Y, U, V.
(3) Let Cn be the circle of radius

1
n

in R2 , centered at the origin. Let


A :=

Cn .

n=1

Determine whether or not A is a closed subset of R2 with the standard topology.


Solution: It is NOT closed. To see that, take the complement of A in R2 . Since the origin (0, 0)
is not in A: for any n Z1 , the circle Cn doesnt contain (0, 0), we know that (0, 0) R2 A.
Recall that a basis of the standard topology of R2 is
{Ua,b,c,d := (a, b) (c, d) | a < b, c < d}.
1

See Example in page 3 of [L] or Secion 2.1 of [L]. If R2 A is an open set, then there must be
some Ua,b,c,d containing (0, 0) such that Ua,b,c,d R2 A (This is the condition (G1)). However,
for any Ua,b,c,d containing the origin (0, 0), let  := min{|a|, |b|, |c|, |d|} and then there is some large
n such that 1/n < . For this n, Cn Ua,b,c,d . So Ua,b,c,d can never be contained in R2 A.
(4) Let A, B, Ai denote subsets of a topological space X. Then

(a) Prove that A B = A B.


(b) Prove that
[
[
Ai .
Ai
iI

iI

Find a counterexample of the other inclusion.


Note: to disprove means to find a counterexample
(c) Prove or disprove that A B = A B.
for the statement.
(d) Prove or disprove that
\
\
Ai .
Ai =
iI

iI

Solution:
then every neighborhood of x intersects with A or every neighborhood of x
(a) If x A B,
intersects with B. Thus every neighborhood intersects with A B. Therefore x A B.
Therefore A B A B.
It is tempting to show the other inclusion as follows: if x A B, then every neighborhood
But this
of x intersects with A B, so intersects with A or with B, thus x A or x B.
argument is wrong. Why? Anyway, we actually prove this way: We know that A B is
closed and contains A B. Since A B is the intersection of all closed sets containing A B,
so A B must contains A B. This proves the other inclusion.
S
S
(b) iI Ai iI Ai follows from the same argument as in (a). The argument of the other
S
inclusion (the correct one!) doesnt work because iI Ai may not be a closed set: finite
union of closed sets is closed but infinite union may not be closed as in Exercise (3) HW2. So
actually the example in Exercise (3) HW2 gives a counter example: as proved in (3), every
basis neighborhood of (0, 0) intersects with some Cn , thus,
(0, 0)

[
n=1

Cn

but

(0, 0) <

[
n=1

C n =

Cn .

n=1

(c) Since A B contains A B and is closed, A B contains A B by the definition of closure.


i.e. every neighborhood of x intersects with A and
For the other inclusion, let x A B,
every neighborhood of x intersects with B. But we are not sure the non-trivial intersection is
actually in A B. Now here is the counter example:
A = {1/n | n Z1 } B = {1/n | n Z1 }.
However, A B = . Thus 0 A B , A B = = .
Then obviously 0 A and 0 B.
T
T
(d) iI Ai iI Ai can be proved in the same way as in (c) since any intersection of closed
sets is closed. The other inclusion fails as the counterexample in (c) works here too.
(5) The boundary A of a subset A of a topological space X is defined by
A := A Int A.
From this definition, it follows that A is the disjoint union of A and Int A, i.e. A = A Int A and
A Int A = .
Find the boundary, the closure and the interior of the following subsets in the specified topology:

(a) (0, 1] in R with the standard topology.


(b) (0, 1] in R with the finite complement topology.
(c) Q in R with the standard topology.
Solution: The easiest way to figure out the closure and the interior here is to use Definition
2.10 of the lecture notes. Then using the formula above, we get the boundary of the sets.
(a) The closure is the interval [0, 1], the interior is open interval (0, 1) and the boundary is the set
{0, 1}.
(b) The closure is R, the interior is and the boundary is R.
(c) The closure is R, the interior is and the boundary is R.
References
[M]
[S]
[L]

Munkres, Topology.
Basic Set Theory, http://www.math.cornell.edu/matsumura/math4530/basic set theory.pdf
Lecture notes, available at http://www.math.cornell.edu/matsumura/math4530/math4530web.html

MATH 4530 Topology. HW 3


Please declare any collaborations with classmates; if you find solutions in books or online,
acknowledge your sources in either case, write your answers in your own words.
Please attempt all questions and justify your answers.

(1) Show that X is a Hausdorff space if and only if the diagonal := {(x, x) | x X} X X is closed
with respect to the product topology.
(2) Find all points that the sequence {xn = 1/n | Z>0 } converges to with respect to the following
topology of R. Justify your answer.
(a) Standard Topology
(b) Finite Complement Topology
(c) Discrete Topology
(d) Lower Limit Topology
(3) Let X and Y be topological spaces. Prove that f : X Y is continuous if and only if for every
subset A of X, we have f (A) f (A).
(4) Define a map f : R R by

if x is rational
|x|
x 7

|x| if x is irrational.
Show that f is continuous at x = 0 but not continuous at other points.
(5) Let X and Y be sets. Let 1 : X Y X and 2 : X Y Y be the projections to the first and
the second factors.
(a) (Set Theory) For a given set Z with maps f1 : Z X and f2 : Z Y, find a map g : Z
X Y such that f1 = 1 g and f2 = 2 g. Show that such g is unique .
(b) Suppose X and Y are topological spaces. Show that 1 and 2 are continuous maps with
respect to the product topology Tprod on X Y. Show that any topology T on X Y such that
1 and 2 are continuous, must be finer than the product topology.
(c) Suppose X and Y are topological spaces. For a given topological space Z with continuous
maps f1 : Z X and f2 : Z Y, show that the map g you found in (a) is continuous with
respect to the product topology on X Y.
(d) Explain why there is no finer topology on X Y than the product topology such that (c) holds.
(6) Show that the open interval (/2, /2) of R with the subspace topology is homeomorphic to R.
Show that any open interval is homeomorphic to R.
(7) Suppose that X, Y, Z are topological spaces. Let f : X Y and g : Y Z be maps of sets. Prove
or disprove the following statement:
(a) If f : X Y is continuous and the composition map g f : X Z is continuous, then
g : Y Z is continuous.
(b) If g : Y Z is continuous and the composition map g f : X Z is continuous, then
f : X Y is continuous.
References
[M]
[S]
[L]

Munkres, Topology.
Basic Set Theory, http://www.math.cornell.edu/matsumura/math4530/basic set theory.pdf
Lecture notes, available at http://www.math.cornell.edu/matsumura/math4530/math4530web.html

MATH 4530 Topology. HW 3 Solution

(1) Show that X is a Hausdorff space if and only if the diagonal := {(x, x) | x X} X X is closed
with respect to the product topology.
Solution:
: Assume that the diagonal is closed. Consider a, b X with a , b, note that (a, b) X X \ .
Now since is closed with respect to the product topology, open set U = U1 U2 TXX
such that U = . This implies that we found U1 , U2 TX satisfying a U1 , b U2 and
U1 U2 = . So X is Hausdorff.
: Assume that X is Hausdorff. Given a point (a, b) X X, with a , b, we will show that
(a, b) is not a limit point of . Consider the open sets U1 , U2 TX , with a U1 , b U2 and
U1 U2 = (such sets exit since X is Hausdorff). Then U1 U2 X X \ is an open set
in the product topology containing the point (a, b). Since contains all its limit points, it is
closed.
(2) Find all points that the sequence {xn = 1/n | Z>0 } converges to with respect to the following
topology of R. Justify your answer.
(a) Standard Topology
(b) Finite Complement Topology
(c) Discrete Topology
(d) Lower Limit Topology
Solution:
(a) {0}. For any open set (a, b) around 0, we have 1/n (a, b) for all n > 1/b.
(b) R. Since the complement of any non-empty open set, U, in the finite complement topology
is finite, we must have U contains infinitely many points of the sequence {xn }.
(c) . For any point in discrete topology, we can consider an open set containing just that point.
(d) {0}. For any open set [a, b) around 0, we have 1/n [a, b) for all n > 1/b.
(3) Let X and Y be topological spaces. Prove that f : X Y is continuous if and only if for every
subset A of X, we have f (A) f (A).
Solution:
() f is continuous, so by Ex 3.14 [?], for every closed subset CY in Y, f 1 (CY ) is closed in X.
f 1 ( f (A)) is closed in X and A f 1 ( f (A))
A f 1 ( f (A)) by the definition of closure.
f (A) f (A)

by the definition of preimages

() Let CY be a closed set in Y. Let A := f 1 (CY ). To show that f 1 (CY ) is closed, we just need
f (A) = CY = CY . Thus A f 1 (CY ) = A. Since
to show A = A. By the assumption, f (A)

A A by the definition of closure, we have A = A.


(4) Define a map f : R R by

|x|
x 7

|x|

if x is rational
if x is irrational.

Show that f is continuous at x = 0 but not continuous at other points.


1

Solution: First lets prove continuity at 0. Consider any basis element of R containing f (0) =
0, say (a, b). Note that the pre-image of (a, b) contains the open interval ( min{a, b}, min{a, b}),
and hence f is continuous at 0. Now for any positive p R , consider an open set (p/2, p + 1).
Note that the pre-image of this set consists only of rational points and thus does not contain any
open subsets around p. So, f is not continuous for p positive. Similarly, for any negative p R
, consider an open set (p 1, p/2). Note that the pre-image of this set consists only of irrational
points and thus does not contain any open subsets around p. So, f is not continuous for p negative.
(5) Let X and Y be sets. Let 1 : X Y X and 2 : X Y Y be the projections to the first and
the second factors.
(a) (Set Theory) For a given set Z with maps f1 : Z X and f2 : Z Y, find a map g : Z
X Y such that f1 = 1 g and f2 = 2 g. Show that such g is unique .
(b) Suppose X and Y are topological spaces. Show that 1 and 2 are continuous maps with
respect to the product topology Tprod on X Y. Show that any topology T on X Y such that
1 and 2 are continuous, must be finer than the product topology.
(c) Suppose X and Y are topological spaces. For a given topological space Z with continuous
maps f1 : Z X and f2 : Z Y, show that the map g you found in (a) is continuous with
respect to the product topology on X Y.
(d) Explain why there is no finer topology on X Y than the product topology such that (c) holds.
Solution:
(a) g : Z X Y is given by g(z) = ( f1 (z), f2 (z)), which clearly satisfies the required conditions.
Let g0 : Z X Y be a map satisfying the conditions. Let g0 (z) = (g01 (z), g02 (z)). Then the
conditions make sure that g01 = f1 and g02 = f2 .
(b) For any open set U1 in X, 1
1 (U 1 ) = U 1 Y which is an open set in the product topology. The
same works for 2 . Say T is some topology on XY such that 1 and 2 are continuous. Then
U1 Y and X U2 must be open sets in (X Y, T ), thus U1 U2 = (U1 Y) (X U2 ) T .
Thus T prod T .
(c) For any open set U1 U2 in X Y, we need to show that g1 (U1 U2 ) Z is open.
z g1 (U1 U2 ) g(z) U1 U2 f1 (z) U1 and f2 (z) U2
z f11 (U1 ) and z f21 (U2 ) z f 1 (U1 ) f 1 (U2 ).
Thus g1 (U1 U2 ) = f11 (U1 ) f21 (U2 ) which is open since f1 and f2 are continuous.
(d) Apply the construction in (a) to Z = X Y and 1 and 2 . Then g = idXY . Say we put some
topology T in X Y. Since idX : (X Y, T prod ) (X Y, T ) must be continuous, T cannot
be finer than the product topology.
(6) Show that the open interval (/2, /2) of R with the subspace topology is homeomorphic to R.
Show that any open interval (a, b) is homeomorphic to R.
Solution: Define f : (/2, /2) R to be f () := tan which is one to one and continuous.
The function tan1 is also continuous. Thus f is a homeomorphism. Now any two interval (a, b)
and (c, d) are homeomorphic, by
(a, b)

xa

/ (0, b a)

x/(ba)

/ (0, 1)

/ (0, d c)

(dc)x

x+c

/ (c, d).

So (a, b) is homeomorphic to (/2, /2) and (/2, /2) is homeomorphic to R, so (a, b) is


homeomorphic to R.
(7) Suppose that X, Y, Z are topological spaces. Let f : X Y and g : Y Z be maps of sets. Prove
or disprove the following statement:

(a) If f : X Y is continuous and the composition map g f : X Z is continuous, then


g : Y Z is continuous.
(b) If g : Y Z is continuous and the composition map g f : X Z is continuous, then
f : X Y is continuous.
Solution: Let R s be R with standard topology and let R f.c. be R with finite complement
topology. The following diagrams will give the counterexamples for (a) and (b), respectively :
let X ?
?f

/Y

??
?? g
? 

g f

be

Rs C
C

/ R f.c.

CC
CC
CC
! 

Rs

R f.c.

/ Rs
EE
EE
EE
EE
" 

R f.c.

where all maps are identity maps. You can actually replace R s and R f.c. with any (X, T ) and
(X, T 0 ) such that T is finer than T 0 .
References
[M]
[S]
[L]

Munkres, Topology.
Basic Set Theory, http://www.math.cornell.edu/matsumura/math4530/basic set theory.pdf
Lecture notes, available at http://www.math.cornell.edu/matsumura/math4530/math4530web.html

MATH 4530 Topology. HW 4


Please declare any collaborations with classmates; if you find solutions in books or online,
acknowledge your sources in either case, write your answers in your own words.
Please attempt all questions and justify your answers.

(1) Let C and D be compact subspaces of a topological space X. Show that A := C D is compact.
(2) Prove the following:
(a) If f : X Y and g : Y Z are quotient maps, then g f : X Z is a quotient map.
(b) Let f : X Y and g : Y Z be continuous maps. If f and g f are quotient maps, then g
is a quotient map.
(3) Prove the following theorem: The image of a compact space under a continuous map is compact.
(4) Let f (x1 , , xn ) be a polynomial in variables x1 , , xn . Show that the set of solutions to f = 0
is a closed set in Rn .
(5) Let SL(2, R) be the set of all 2 2 matrices of determinant 1. Define a topology on SL(2, R) as a
subspace of R4 .
(a) Show that it is closed.
(b) Show that it is not compact. Hint: Show that it is not sequentially compact and apply Theorem
28.2 [M]. Or, apply Theorem 27.3 [M].
(6) Let C := {z = x + iy | x, y R} be the set of all complex numbers. By identifying C  R2 , we have
a topology on C induced from the standard topology of R2 . Conisder
X := C2 {(0, 0)} = {(z, w) C2 | (z, w) , (0, 0)}.
Define an equivalence relation on X by (z1 , w1 ) (z2 , w2 ) if (z1 , w1 ) = (z2 , w2 ) for some
C {0}. The quotient space X/ is called the projective space CP1 . Show that CP1 is compact.
Hint: find a surjective continuous map S 3 CP1 and use the fact that S 3 is compact.
References
[M]
[S]
[L]

Munkres, Topology.
Basic Set Theory, http://www.math.cornell.edu/matsumura/math4530/basic set theory.pdf
Lecture notes, available at http://www.math.cornell.edu/matsumura/math4530/math4530web.html

MATH 4530 Topology. HW 4 Solutions


Please declare any collaborations with classmates; if you find solutions in books or online,
acknowledge your sources in either case, write your answers in your own words.
Please attempt all questions and justify your answers.

(1) Let C and D be compact subspaces of a topological space X. Show that A := C D is compact.
Solution: Let {U A} be an open covering of A where U s are open sets in X. Then
{U C} and {U D} are open coverings of C and D. Since C, D are compact, we find a
finite subcoverings {Vi C, i = 1, , n} and {W j D, j = 1, , m} where {Vi }, {W j } {U }.
Then {Vi A, W j A, i = 1, , n, j = 1, , m} is a finite subcovering of A.
(2) Prove the following:
(a) If f : X Y and g : Y Z are quotient maps, then g f : X Z is a quotient map.
(b) Let f : X Y and g : Y Z be continuous maps. If f and g f are quotient maps, then g
is a quotient map.
Solution:
(a) First note that since f and g are surjective, then so is g f . Since g and f are quotient
maps, we have:
U is open in Z g1 (U) is open in Y f 1 (g1 (U)) = (g f )1 (U) is open in X.
Thus g f : X Z is a quotient map.
(b) First note that since g f is surjective, then so is g. Since g is continuous we have for
any open U Z, g1 (U) is open in Y. Now given some V Z, such that g1 (V) is
open in Y, we want to show that V must be open in Z. Since f is a continuous map,
f 1 (g1 (V)) = (g f )1 (V) is open in X. Finally, since g f is a quotient map we must
have that V is open in Z as needed. Hence, g is a quotient map.
(3) Prove the following theorem: The image of a compact space under a continuous map is compact.
Solution: Let f : X Y be a continuous map and X a compact space. Let {V } be an
open covering of f (X). Then { f 1 (V )} is an open covering of X and so we have a finite
subcovering { f 1 (Vi ), i = 1, , n}. Then it is easy to see that {Vi } covers f (X). Indeed
f (X) = f (i f 1 (Vi )) i Vi .
(4) Let f (x1 , , xn ) be a polynomial in variables x1 , , xn . Show that the set of solutions to f = 0
is a closed set in Rn .
Solution: Since f : Rn R is continuous, the preimage f 1 (0) of a closed set {0} is closed.
(5) Let SL(2, R) be the set of all 2 2 matrices of determinant 1. Define a topology on SL(2, R) as a
subspace of R4 .
(a) Show that it is closed.
(b) Show that it is not compact. Hint: Show that it is not sequentially compact and apply Theorem
28.2 [M]. Or, apply Theorem 27.3 [M].
1

Solution:
(a) SL(2, R) is the set of solutions to the
equation ad bc = 1, so it is closed.
( polynomial
)
x y
(b) Let us identify a 2 2 matrices
with a 4-vector (x, y, z, w). Now consider
z w
(
)
1 n
Xn :=
which has determinant 1. Since the corresponding sequence {(1, n, 0, 1)} in R4
0 1
does not have any convergent subsequence, SL(2, n) is not sequentially compact and so by
Theorem 28.2 [M], it is not compact. Or the distance between (1, 0, 0, 1) and (1, n, 0, 1) can
be arbitrary large, so SL2 (R) is not bounded. Thus by Theorem 27.3 [M], it is not compact.
(6) Let C := {z = x + iy | x, y R} be the set of all complex numbers. By identifying C  R2 , we have
a topology on C induced from the standard topology of R2 . Conisider
X := C2 {(0, 0)} = {(z, w) C2 | (z, w) , (0, 0)}.
Define an equivalence relation on X by (z1 , w1 ) (z2 , w2 ) if (z1 , w1 ) = (z2 , w2 ) for some
C {0}. The quotient space X/ is called the projective space CP1 . Show that CP1 is compact.
Hint: find a surjective continuous map S 3 CP1 and use the fact that S 3 is compact.
Solution: There is S 3 sitting in C2 {(0, 0)}, defined by |z|2 +|w|2 = 1. We have the following
commutative diagram of continuous maps:

S 3 JJ i / C2 {(0, 0)}
JJ
JJ
J

p JJJ
% 
CP1 .
p is surjective: let [z, w] CP1 . If |z|2 + |w|2 = 1, then |rz|2 + |rw|2 = r2 so that (rz, rw) S 3 .
Since [z, w] = [rz, rw], p is surjective. S 3 is compact by Theorem 27.3 [M]. CP1 is its
image under a continuous map, so by Theorem 26.5 [M], it is compact.
References
[M]
[S]
[L]

Munkres, Topology.
Basic Set Theory, http://www.math.cornell.edu/matsumura/math4530/basic set theory.pdf
Lecture notes, available at http://www.math.cornell.edu/matsumura/math4530/math4530web.html

MATH 4530 Topology. HW 5


Please declare any collaborations with classmates; if you find solutions in books or online,
acknowledge your sources in either case, write your answers in your own words.
Please attempt all questions and justify your answers.

Write the proofs in complete sentences.


(1) Show that any infinite set with the finite complement topology is connected.
(2) Let T and T 0 be topologies on a set X. If T is finer than T 0 , then the connectedness of which
topology will implies the one of the other? Justify your answer.
(3) Let p : X Y be a quotient map. Show that, if p1 (y) is connected for each y Y and Y is
connected, then X is connected.
(4) Let f : X Y be a continuous map. Show that if X is path-connected, then Im f is pathconnected.
(5) Show that there is no homeomorphism between (0, 1) and (0, 1] by using the connectedness. Hint:
if we remove a point from each of the spaces, what happens?
(6) Show that if X is connected, then any continuous map f : X Y where Y is a topological space
with discrete topology is a constant map, i.e. f (X) = {y} for some y Y.
(7) Consider the quotient space of R2 by the identification (x, y) (x + n, y + m) for all (n, m) Z2 .
Show that it is connected and compact.
(8) Recall that a square matrix M is orthogonal if MM t = I. This condition is equivalent to the set
~ i = h~v, w
of row vectors form an orthonormal basis and also to hM~v, M w
~ i. In particular, if M is
orthogonal, then det M = 1. Let O(n, R) be the set of orthogonal matrices of size n. Show that it
is not connected.
References
[M]
[S]
[L]

Munkres, Topology.
Basic Set Theory, http://www.math.cornell.edu/matsumura/math4530/basic set theory.pdf
Lecture notes, available at http://www.math.cornell.edu/matsumura/math4530/math4530web.html

MATH 4530 Topology. HW 5 solutions


Please declare any collaborations with classmates; if you find solutions in books or online,
acknowledge your sources in either case, write your answers in your own words.
Please attempt all questions and justify your answers.

Write the proofs in complete sentences.


(1) Show that any infinite set with the finite complement topology is connected.
Solution: If X = C1 t C2 where C1 , C2 are non-empty closed sets, since C1 and C2 must be
finite, so X is finite. This is a contradiction.
(2) Let T and T 0 be topologies on a set X. If T is finer than T 0 , then the connectedness of which
topology will implies the one of the other? Justify your answer.
Solution: We have a condituous map idX : (X, T ) (X, T 0 ). Since the image of a connected space is connected, the connectedness of T implies T 0 .
(3) Let p : X Y be a quotient map. Show that, if p1 (y) is connected for each y Y and Y is
connected, then X is connected.
Solution: Suppose that X is not connected. Say, X = U1 t U2 for some non-empty open
sets U1 and U2 . By Theorem 5.5, every p1 (y) is contained in either U1 or U2 . Let V1 =
{y Y | p1 (y) U1 } and V2 = {y Y | p1 (y) U2 }. Since U1 and U2 are non-empty,
disjoint and p is surjective, we have Y = V1 t V2 and V1 and V2 are non-empty. Also,
1 (V1 ) = U1 , p1 (V2 ) = U2 by surjectivity of p, so by the definition of quotient maps, V1
and V2 are open sets in Y. It follows that Y is not connected. Contradiction.
(4) Let f : X Y be a continuous map. Show that if X is path-connected, then Im f is pathconnected.
Solution: Let x, y Im f . Let x1 f 1 (x) and y1 f 1 (y). Since X is path connected,
there is a path p : [0, 1] X connecting x1 and y1 . Then f p is a path connecting x and y.
(5) Show that there is no homeomorphism between (0, 1) and (0, 1] by using the connectedness. Hint:
if we remove a point from each of the spaces, what happens?
Solution: Suppose that there is a homeomorphism f : (0, 1] (0, 1). If f : X Y is a
homeomorphism, then the restriction f |X{x} : X {x} Y { f (x)} is a homeomorphism
too. If we apply this to (0, 1] {1} = (0, 1), then we have a homeomorphism (0, 1) 
(0, 1) { f (1)}. LHS is connected but the right hand side is (0, f (1)) t ( f (1), 1) which is
disconnected. So we have a contradiction.
(6) Show that if X is connected, then any continuous map f : X Y where Y is a topological space
with discrete topology is a constant map, i.e. f (X) = {y} for some y Y.
Solution: If f (X) has more than one element, then since Y has the discrete topology, f (X)
is disconnected. Since the image of a connected space must be connected, we have a contradiction. Thus f (X) has only one element, i.e. f is a constant map.
1

(7) Consider the quotient space of R2 by the identification (x, y) (x + n, y + n) for all (n, m) Z2 .
Show that it is connected and compact.
Solution: Since R2 is conencted, the quotient space must be connencted since the quotient
space is the image of a quotient map from R2 . Consider E := [0, 1] [0, 1] R2 , then the
restriction of the quotient map p : R2 R2 / to E is surjective. Since [0, 1] [0, 1] is
compact, the image R2 / must be compact.
(8) Recall that a square matrix M is orthogonal if MM t = I. This condition is equivalent to the set
~ i = h~v, w
~ i. In particular, if M is
of row vectors form an orthonormal basis and also to hM~v, M w
orthogonal, then det M = 1. Let O(n, R) be the set of orthogonal matrices of size n. Show that it
is not connected.
Solution: The determinant of an orthogonal matrix is 1. Since M1 := {M : det M = 1}
and M1 := {M : det M = 1} are closed subsets of the set of matrice, we have
O(n, R) = (M1 O(n, R)) t (M1 O(n, R))
is a disjoint union of closed sets. Each of them is non-empty, since we have the identity
matrix In M1 O(n, R) and the identity matrix with 1 multiplied to exactly one of the
diagonal entry is in M1 O(n, R).
References
[M]
[S]
[L]

Munkres, Topology.
Basic Set Theory, http://www.math.cornell.edu/matsumura/math4530/basic set theory.pdf
Lecture notes, available at http://www.math.cornell.edu/matsumura/math4530/math4530web.html

MATH 4530 Topology. Prelim I


In class 75min: 2:55-4:10 Thu 9/30.

Problem 1 (45pts): Consider the following topological spaces:


(1) Z as a subspace of R with the finite complement topology
(2) [0, ] as a subspace of R.
(3) [0, ] {1} as a subspace of R.
2
= 1} as a subspace of Rn+1 .
(4) The unit n-sphere S n := {(x1 , , xn+1 ) Rn | x12 + + xn+1
2
n
n
2
(5) The unit n-disk D := {(x1 , , xn ) R | x1 + + xn 1} as a subspace of Rn .
(6) S 1 S 1 with the product topology, called the torus.
(7) RP2 defined as a quotient space of R3 {(0, 0, 0)} by the identification (x, y, z) (x, y, z) for
all R {0}.
(8) The quotient space of R2 by the identification (x, y) (x, y) for all R>0 .
(9) O(2, R) the set of all 2 2 matrices M with real values such that MM t = I2 , as a subspace of R4 .
(10) The quotient space of R2 by the identification (x, y) (x + n, y + n) for all n, m Z.
(11) The closure of {(x2 , sin(1/x)) R2 |x R>0 } in R2 .
Q1-1: Find all compact spaces. (11pts)

Q1-2: Find all connected spaces. (11pts)

Q1-3: Find all Hausdorff spaces. (11pts)

Q1-4: Find two spaces that are homeomorphic. (2pts)

Q1-5: Find all path connected spaces from (2) - (11). (10pts)

Problem 2 (20pts): True or false? If it is true, write the proof in complete sentences. If it is false, then
give a counterexample.
(1) If X is a Hausdorff space, then any quotient space of X is Hausdorff. (5pts)

(2) Let f : X Y be a continuous map between topological spaces. If a sequence (xn ) converges to
x in X, then ( f (xn )) converges to f (x) in Y. (5pts)

(3) Let f : X Y be a continuous and injective map. Suppose that X is compact and Y is Hausdorff.
If A is a closed subspace of X, then the restriction map f |A : A f (A) is a homeomorphism.
(5pts)

(4) Let B and B0 be bases of topologies T and T 0 . If T is finer than T 0 , then B B0 . (5pts)

Definitions:
(1) A collection T of subsets of a set X is a topology if , X T and an arbitrary union and a finite
intersection of subsets in T are also in T .
(2) A collection B of subsets of X is a basis of a topology if the members of B covers X and for every
B1 , B2 B and every x B1 B2 , there is B3 such that x B3 B1 B2 .
(3) The topology TB consists of subsets U in X such that every x U, there is B B such that
x B U.
(4) A subset A of a topological space X is closed if X A is open.
(5) The closure of A is the intersection of all closed sets containing A.
(6) Let A be a subset of a topological space X. x X is a cluster point of A in X if x A {x}.
(7) Let X, Y be topological space. A map f : X Y is continuous if f 1 (U) is open in X for all open
set U in Y. A map f : X Y is a homeomorphism if f is a continuous bijection and f 1 is also
continuous.
(8) A topological space X is Hausdorff if for every distinct points x, y, there are neighborhoods U x
and Uy of x and y such that U x Uy = .
(9) A topological space X is compact if every open covering has a finite subcovering.
(10) A topological space X is connected if X is not a disjoint union of two open sets.
(11) A topological space X is path-connected if every two points can be connected by a path f :
[0, 1] X.
(12) A quotient space of a topological space X is given by a space Y such that f : X Y is a surjective
continuous map and a subset U in Y is open if and only if 1 (U) is open in X.
(13) A sequence (xn ) in a topological space X converges to x X if for every neighborhood U x of x,
there is N such that xn U x for all n > N.
(14) A metric d on a set X is a map d : X Y R such that (i) d(x, y) 0 and the equality holds
iff x = y, (ii) d(x, y) = d(y, x), and (iii) d(x, z) d(x, y) + d(y, z). The metric topology is then the
topology generated by -balls.
(15) For topological spaces X and Y, the product topology on X Y is generated by U V for all open
sets U in X and V in Y.
Theorems:
(1) Let A be a subset of a topological space X. Then x A if and only if U x A , for all
neighborhoods U x of x.
(2) A compact subspace of a Hausdorff space is closed.
(3) A closed interval [a, b] in R is compact.
(4) A finite product of compact spaces is compact.
(5) The image of a compact space under a continuous map is compact.
(6) Every closed subspace of a compact space is compact.
(7) A subspace A of Rn is compact if and only if it is closed and bounded, i.e. the distance of any two
points in A is bounded.
(8) Let X be a metric space and Y a topological space. A map f : X Y is continuous if and only if
for every convergent sequence (xn ) to x in X, the sequence ( f (xn )) in Y converges to f (x).
(9) If X is compact, Y is Hausdorff, and f : X Y is a continuous bijection, then f is a homeomorphism.
(10) The image of a connected space under a continuous map is connected.
(11) A finite product of connected spaces is connected.
(12) If A is a connected subspace of a topological space, then the closure A is also a connected subspace.
(13) If a topological space X is path-connected, then it is connected.

MATH 4530 Topology. Prelim I Solutions


In class 75min: 2:55-4:10 Thu 9/30.

Problem 1: Consider the following topological spaces:


(1) Z as a subspace of R with the finite complement topology
(2) [0, ] as a subspace of R.
(3) [0, ] {1} as a subspace of R.
2
= 1} as a subspace of Rn+1 .
(4) The unit n-sphere S n := {(x1 , , xn+1 ) Rn | x12 + + xn+1
2
n
n
2
(5) The unit n-disk D := {(x1 , , xn+1 ) R | x1 + + xn+1 1} as a subspace of Rn+1 .
(6) S 1 S 1 with the product topology, called the torus.
(7) RP2 defined as a quotient space of R3 {(0, 0, 0)} by the identification (x, y, z) (x, y, z) for
all R {0}.
(8) The quotient space of R2 by the identification (x, y) (x, y) for all R>0 .
(9) O(2, R) the set of all 2 2 matrices M with real values such that MM t = I2 , as a subspace of R4 .
(10) The quotient space of R2 by the identification (x, y) (x + n, y + n) for all n, m Z.
(11) The closure of {(x2 , sin(1/x)) R2 |x R>0 } in R2 .
Q1-1: Find all compact spaces. (11pts)
Solution:
(1) Compact: Any infinite set with finite complement topology is compact. The proof is
as follows. Let X be an infinite set with the f.c. topology. Let {U } be a covering of
X. Then X U is a finite set, say {x1 , , xn }. Let Ui be one of the open sets that
contains xi . Then U U1 Un = X.
(2) Compact: This is the most basic key fact of compactness.
(3) Not compact: Let Un := [0, 1) (1 + 1/n, ] for n Z>0 . Then {Un } is an open covering. If there is a finite subcovering, say, {Uni }i=1, ,m . Let a := max{n1 , , nm }.
Then m
i=1 U ni cannot contain 1 + 1/(a + 1). So it cant be a covering.
(4) Compact: Any closed, bounded subset in Rn is compact.
(5) Compact: Any closed, bounded subset in Rn is compact.
(6) Compact: S 1 is compact from (4). A finite product of compact spaces is compact.
(7) Compact: It is actually a quotient of S 2 R3 {(0, 0, 0)}. Since S 3 is compact from
(4), RP2 is the image of the quotient map of a compact space. Since the image of a
compact space under a continuous map is compact, it is compact.
(8) Compact: The quotient map factors through a surjective map S 3 {(0, 0, 0)}
R2 / . This map is not quotient map but it is still continuous (the restriction of a
continuous map on a subspace is continuous). Since S 3 {(0, 0, 0)} is bounded and
closed, it is compact. Thus R2 / is the image of a compact space, so it is compact.
(9) Compact: The row vectors ~v1 , ~v2 of orthogonal matrices are unit vectors. As a
vector in R4 , (~v1 , ~v2 ) has length 2. So O(2, R) is bounded, actually is contained in
S 3 . Consider the continuous map M 7 M M t . Then O(2, R) is the pre-image of I
a point in R4 , thus it is closed. Closed and bounded subsets in R4 are compact.
(10) Compact: The quotient map restricted to the compact supspace [0, 1] [0, 1] is
surjective continuous. Thus the quotient is compact.
(11) Not compact: It is not compact since it is not bounded.

Q1-2: Find all connected spaces. (11pts)


Solution:
(1) Connected: any infinite set with a f.c. topology is connected.
(2) Connected: it is the closure of (0, ) which is homeomorphic to R.
(3) Not connected: It is a disjoint union of non-empty open sets.
(4) Connected: since it is path-connected.
(5) Connected: since it is path-connected.
(6) Connected: since it is path-connected.
(7) Connected: since it is path-connected.
(8) Connected: since it is path-connected.
(9) Not connected: the determinant map M 7 det M sends orthogonal matrices to 1.
The preimage of 1 are non-empty closed sets and so O(n, R) is a disjoint union of
non-empty closed sets.
(10) Connected: since it is path-connected.
(11) Connected: since it is the closure of a connected space.
Q1-3: Find all Hausdorff spaces. (11pts)
Solution:
(1) Not Hausdorff: Every two open sets would intersect non-trivially in an infinite set
with f.c. topology.
(2) Hausdorff: Any subspace of a Hausdorff space is Hausdoff.
(3) Hausdorff: Any subspace of a Hausdorff space is Hausdoff.
(4) Hausdorff: Any subspace of a Hausdorff space is Hausdoff.
(5) Hausdorff: Any subspace of a Hausdorff space is Hausdoff.
(6) Hausdorff: The finite product of Hausdorff space is Hausdorff.
(7) Hausdorff: We are identifying all points on a line through origin. If we have two
distinct lines, there are open sets separating those two, if we take the origin out.
(8) Non-Hausdorff: Any open set around the origin intersects with any line minus origin, so we cant separate the image of the origin from other points in RP2 .
(9) Hausdorff: Any subspace of a Hausdorff space is Hausdoff.
(10) Hausdorff: Because it is homeomorphic to (6).
(11) Hausdorff: Any subspace of a Hausdorff space is Hausdoff.
Q1-4: Find two spaces that are homeomorphic. (2pts)
Solution: (6) and (10) are homeomorphic. First R/Z is homeomorphic to S 1 . The quotient
space in (10) is R/Z R/Z, so it is homeomorphic to S 1 S 1 .

Q1-5: Find all path connected spaces from (2) - (11). (10pts)
Solution:
(1)
(2) Path-connected: a path from x to y is given by tx + (1 t)y.
(3) Not path connected: because it is not connected.
(4) Path connected: There is a continuous surjection Rn+1 {(0, , 0)} S n defined
by ~x 7 ~x/|~x|. Since Rn+1 {(0, , 0)} is path connected, S n is path-connected
(image of a path connected is path connected).
(5) Path-connected: the path t~x + (1 t)~y stays inside of the disk.
(6) Path-connected: the finite product of path-connected spaces are path-connected.
(7) Path-connected: similar to (4).
(8) Path-connected: similar to (4).
(9) Not path-connected: because it is not connected.
(10) Path-connected: similar to (4).
(11) Not path-connected: {(x, sin(1/x)) | x R>0 } is not path connected. There is a
homeomorphism (x, sin(1/x)) 7 (x2 , sin(1/x)), so it is not path-connected.

Problem 2 (20pts): True or false? If it is true, write the proof in complete sentences. If it is false, then
give a counterexample.
(1) (5pts) If X is a Hausdorff space, then any quotient space of X is Hausdorff.
Solution: False. (8) in Problem 1 is a counterexample. R2 is Hausdorff, but the quotient is
not.
(2) (5pts) Let f : X Y be a continuous map between topological spaces. If a sequence (xn )
converges to x in X, then ( f (xn )) converges to f (x) in Y.
Solution: True. Let U be a neighborhood of f (x), then f 1 (U) is a neighborhood of x, so
there is N > 0 such that xn f 1 (U) for all n > N. Thus f (xn ) U for all n > N. Thus
f (xn ) converges to f (x).
(3) (5pts) Let f : X Y be a continuous and injective map. Suppose that X is compact and Y
is Hausdorff. If A is a closed subspace of X, then the restriction map f |A : A f (A) is a
homeomorphism.
Solution: True. Since A is a closed subspace of a compact space X, A is compact. Since
f (A) is a subspace of a Hausdorff space Y, f (A) is Hausdorff. Since f is injective, f |A :
A f (A) is bijective. Thus Theorem (9) implies f |A is a homeomorphism.
(4) (5pts) Let B and B0 be bases of topologies T and T 0 . If T is finer than T 0 , then B B0 .
Solution: False. The standard topology T of R is finer than the finite complement topology
T 0 of R. Let B be the open ball basis of T and B = T . T T 0 but B 2 B0 , i.e. an open
set in the finite complement topology is open in the standard topology but it is not an open
ball.

Definitions:
(1) A collection T of subsets of a set X is a topology if , X T and an arbitrary union and a finite
intersection of subsets in T are also in T .
(2) A collection B of subsets of X is a basis of a topology if the members of B covers X and for every
B1 , B2 B and every x B1 B2 , there is B3 such that x B3 B1 B2 .
(3) The topology TB consists of subsets U in X such that every x U, there is B B such that
x B U.
(4) A subset A of a topological space X is closed if X A is open.
(5) The closure of A is the intersection of all closed sets containing A.
(6) Let A be a subset of a topological space X. x X is a cluster point of A in X if x A {x}.
(7) Let X, Y be topological space. A map f : X Y is continuous if f 1 (U) is open in X for all open
set U in Y. A map f : X Y is a homeomorphism if f is a continuous bijection and f 1 is also
continuous.
(8) A topological space X is Hausdorff if for every distinct points x, y, there are neighborhoods U x
and Uy such that U x Uy = .
(9) A topological space X is compact if every open covering has a finite subcovering.
(10) A topological space X is connected if X is not a disjoint union of open sets.
(11) A topological space X is path-connected if every two points can be connected by a path f :
[0, 1] X.
(12) A quotient space of a topological space X is given by a space Y such that f : X Y is a surjective
continuous map and a subset U in Y is open if and only if 1 (U) is open in X.
(13) A sequence (xn ) in a topological space X converges to x X if for every neighborhood U x of x,
there is N such that xn U x for all n > N.
(14) A metric d on a set X is a map d : X Y R such that (i) d(x, y) 0 and the equality holds
iff x = y, (ii) d(x, y) = d(y, x), and (iii) d(x, z) d(x, y) + d(y, z). The metric topology is then the
topology generated by -balls.
(15) For topological spaces X and Y, the product topology on X Y is generated by U V for all open
sets U in X and V in Y.
Theorems:
(1) Let A be a subset of a topological space X. Then x A if and only if U x A , for all
neighborhoods U x of x.
(2) A compact subspace of a Hausdorff space is closed.
(3) A closed interval [a, b] in R is compact.
(4) A finite product of compact spaces is compact.
(5) The image of a compact space under a continuous map is compact.
(6) Every closed subspace of a compact space is compact.
(7) A subspace A of Rn is compact if and only if it is closed and bounded, i.e. the distance of any two
points in A is bounded.
(8) Let X be a metric space and Y a topological space. A map f : X Y is continuous if and only if
for every convergent sequence (xn ) to x in X, the sequence ( f (xn )) in Y converges to f (x).
(9) If X is compact, Y is Hausdorff, and f : X Y is a continuous bijection, then f is a homeomorphism.
(10) The image of a connected space under a continuous map is connected.
(11) A finite product of connected spaces is connected.
(12) If A is a connected subspace of a topological space, then the closure A is also a connected subspace.
(13) If a topological space X is path-connected, then it is connected.

MATH 4530 Topology. HW 6


Please declare any collaborations with classmates; if you find solutions in books or online,
acknowledge your sources in either case, write your answers in your own words.
Please attempt all questions and justify your answers.

Write the proofs in complete sentences.


(1) Let m be a positive integer.
(a) Find an explicit isomorphism from the quotient group (Z/mZ, +) to a subgroup of U(1).
(b) Let m = 9. Find all the invertible elements with respect to the multiplication induced from
(R , ) and show that they form a group. Is it a cyclic group?
(2) Let D3 be the group generated by two elements x, y with the relations xy = y2 x, x2 = 1, y3 = 1.
What is the number of the elements in the group D3 ? Write an isomorphism from S 3 to D3 where
S 3 is the permutation group on {1, 2, 3}. See Example 7.4 [L].
(3) Let g be an element of a topological group G. Show that the map g : G G, h 7 gh is a
homeomorphism. Also show that the map G G, h 7 h1 is a homeomorphism.
(4) Let R X X be an equivalence relation on a topological space X. Show that if R is closed in
X X, then the quotient space X/ is Hausdorff. Hint: first prove that, for subsets U, V X,
1 ((U)) 1 ((V)) = if and only if (U V) R = .
(5) Let G be a finite group with discrete topology. Show that, if G acts on a Hausdorff space X
continuously, then the quotient space X/G is Hausdorff. Hint: show that the relation is closed and
the apply the previous problem.
(6) (optional) Show that a topological group G is Hausdorff. Hint: for an open neighborhood U of the
identity 1G , find another open neighborhood V of 1G such that V = V 1 and V V U.
References
[M]
[S]
[L]

Munkres, Topology.
Basic Set Theory, http://www.math.cornell.edu/matsumura/math4530/basic set theory.pdf
Lecture notes, available at http://www.math.cornell.edu/matsumura/math4530/math4530web.html

MATH 4530 Topology. HW 6 solutions


Please declare any collaborations with classmates; if you find solutions in books or online,
acknowledge your sources in either case, write your answers in your own words.
Please attempt all questions and justify your answers.

Write the proofs in complete sentences.


(1) Let m be a positive integer.
(a) Find an explicit isomorphism from the quotient group (Z/mZ, +) to a subgroup of U(1).
(b) Let m = 9. Find all the invertible elements with respect to the multiplication induced from
(R , ) and show that they form a group. Is it a cyclic group?
Solution:
(a) Define f : (Z/mZ, +) U(1), by x 7 e2ix/m .
(b) The invertible elements are the ones relatively prime with 9.
Thus,
{[1], [2], [4], [5], [7], [8]} are all the invertible elements. This is a cyclic group,
generated by [2].
(2) Let D3 be the group generated by two elements x, y with the relations xy = y2 x, x2 = 1, y3 = 1.
What is the number of the elements in the group D3 ? Write an isomorphism from S 3 to D3 where
S 3 is the permutation group on {1, 2, 3}. See Example 7.4 [L].
Solution: There are 6 elements in D3 . They are 1, x, y, y2 , xy, xy2 . Let f : S 3 D3 be
defined by f ((1 2)) = x, f ((1 2 3)) = y.
(3) Let g be an element of a topological group G. Show that the map g : G G, h 7 gh is a
homeomorphism. Also show that the map G G, h 7 h1 is a homeomorphism.
Solution: Clearly, g is a bijection. By definition of a topological group, we know that g is
continuous. Now since g1 = (g )1 is multiplication on the left by g1 , by definition it is
continuous. Hence, g is a homeomorphism. Similar argument shows that the map h 7 h1
is a homeomorphism.
(4) Let R X X be an equivalence relation on a topological space X. Show that if R is closed in
X X, then the quotient space X/ is Hausdorff. Hint: first prove that, for subsets U, V X,
1 ((U)) 1 ((V)) = if and only if (U V) R = .
Solution: It seems I was thinking that R comes from a continuous group action. Assume
that to prove U xR is an open set.
(1) Let U R := 1 ((U)). If U R V R , , say, z U R V R , then there are x U, y V
such that x z and y z. But this means that x y. Thus (x, y) (U V) R. On the
other hand, if (x, y) (U V) R, then x, y U R V R obviously.
(2) Let x / y X. Then there are neighborhoods U x and Vy such that (U x Vy ) R =
since R is closed. Then by part (a) we have U xR UyR = . Now U xR is open: Let
z U xR . Then there is g G such that z = gx0 for some x0 U x . Since g : X X is a
homeomorphism, gU x is an open set and it contains z. Moreover it is contained in U xR .
Thus U xR is an open set. We find that (U x ) and (Uy ) are open sets in X/ . Since
U xR UyR = , (U x ) and (Uy ) are disjoint too. Thus X/ is Hausdorff.
1

(5) Let G be a finite group with discrete topology. Show that, if G acts on a Hausdorff space X
continuously, then the quotient space X/G is Hausdorff. Hint: show that the relation is closed and
the apply the previous problem.
Solution: Since X is Hausdorff, the diagonal X := {(x, x) XX} is closed (one of the past
homework problems). The relation is the image of the map G X X X, (g, x) 7 (x, gx)
and it is given by
gG (1G , g)X ,
where (1G , g) : X X X X, (x, y) 7 (x, gy) is a homeomophism by (3). Since G is
finite, the above union is a finite union. Since each (1G , g)X is closed, it shows that the
relation is closed.
(6) (optional) Show that a topological group G is Hausdorff. Hint: for an open neighborhood U of the
identity 1G , find another open neighborhood V of 1G such that V = V 1 and V V U.
Solution: See the Proposition 5.7, 5.8 of www.math.uh.edu/vern/grouprepn.pdf.
References
[M]
[S]
[L]

Munkres, Topology.
Basic Set Theory, http://www.math.cornell.edu/matsumura/math4530/basic set theory.pdf
Lecture notes, available at http://www.math.cornell.edu/matsumura/math4530/math4530web.html

MATH 4530 Topology. HW 7


Please declare any collaborations with classmates; if you find solutions in books or online,
acknowledge your sources in either case, write your answers in your own words.
Please attempt all questions and justify your answers.

Write the proofs in complete sentences.


(1) Show that if h, h0 : X Y are homotopic and k, k0 : Y Z are homotopic, then k h and k0 h0
are homotopic.
(2) Let Homot(X; Y) be the set of homotopy classes of continuous maps from X to Y.
(a) Let I := [0, 1]. Show that Homot(X; I) has a single element.
(b) Show that if Y is path-connected, then Homot(I; Y) has a single point.
(3) A space X is said to be contractible if the identity map iX : X X is homotopic to a constant
map.
(a) Show that a disk Dn is contractible.
(b) Show that a contractible space is path-connected.
(c) Show that if Y is contractible, then Homot(X; Y) has a single element.
(d) Show that if X is contractible and Y is path-connected, then Homot(X; Y) has a single element.
(4) Spaces X and Y is said to have the same homotopy type if there are maps f : X Y and
g : Y X such that f g is homotopic to 1Y and g f is homotopic to 1X .
(a) Show that if X is contractible, then X and a single point have the same homotopy type.
(b) Show that if X and Y are homeomorphic, then X and Y have the same homotopy type.
References
[M]
[S]
[L]

Munkres, Topology.
Basic Set Theory, http://www.math.cornell.edu/matsumura/math4530/basic set theory.pdf
Lecture notes, available at http://www.math.cornell.edu/matsumura/math4530/math4530web.html

MATH 4530 Topology. HW 7 solutions


Please declare any collaborations with classmates; if you find solutions in books or online,
acknowledge your sources in either case, write your answers in your own words.
Please attempt all questions and justify your answers.

Write the proofs in complete sentences.


(1) Show that if h, h0 : X Y are homotopic and k, k0 : Y Z are homotopic, then k h and k0 h0
are homotopic.
Solution 1: Let H : X I Y be the homotopy from h to h0 and K : Y I Z be
the homotopy from k to k0 . Define L : X I Z by L(x, t) = K(H(x, t), t). It is clearly
continuous since it is a composition of two continuous functions. We have L(x, 0) = k h(x)
and L(x, 1) = (k0 h0 )(x), hence L is a homotopy from k h to k0 h0 .
Homotopy between maps are not unique at all. It is just that we have so many ways to deform
a map to another continuously. So here is another homotopy for this problem
Solution 2: Let H : X I Y be the homotopy from h to h0 and K : Y I Z be the
homotopy from k to k0 . Define a map F : X [0, 1] Z as follows:

t [0, 1/2]
k(H(x, 2t))
F(x, t) :=
K(h0 (x), 2t 1) t [1/2, 1]

Check that maps on RHS are continuous maps from X [0, 1/2] and X [1/2, 1] to Z and
they agree on X {1/2} so that by the pasting lemma, F is a well-defined continuous map.
Now this F is a homotopy from k h and k0 h0 because
F(x, 0) = k(H(x, 0)) = k(h(x)),

F(x, 1) = K(h0 (x), 1) = k0 (h0 (x)).

(2) Let Homot(X; Y) be the set of homotopy classes of continuous maps from X to Y.
(a) Let I := [0, 1]. Show that Homot(X; I) has a single element.
(b) Show that if Y is path-connected, then Homot(I; Y) has a single point.
Solution:
(a) Let f, g Homot(X; I) and consider a straight-line homotopy between them defined by
H(x, t) = tg(x) + (1 t) f (x). Since any two elements are homotopic, Homot(X; I) has
just a single element.
(b) Let f, g : I Y be continuous maps. We need to show that f  g. For f , consider the
following map:
F : I [0, 1] Y, F(s, t) = f ((1 t)s).
It is a continuous since F is a composition of obvious continuous maps (Ill leave this
to you). Because F(s, 0) = f (s) and F(s, 1) = f (0), F is a homotopy from f to a
constant map e f (0) : I { f (0)} Y. Similarly we have a homotopy from g to eg(0) .
Now since Y is path-connected, we have a path k : [0, 1] Y from f (0) to g(0). k
induces a homotopy from e f (0) to eg(0) :
K : I [0, 1] Y, K(s, t) = k(t).
Thus f  e f (0)  eg(0)  g.
(3) A space X is said to be contractible if the identity map iX : X X is homotopic to a constant
map.
1

(a)
(b)
(c)
(d)

Show that a disk Dn is contractible.


Show that a contractible space is path-connected.
Show that if Y is contractible, then Homot(X; Y) has a single element.
Show that if X is contractible and Y is path-connected, then Homot(X; Y) has a single element.

Solution:
(a) Consider a straight-line homotopy from the points of the disk to the origin.
(b) Let x, y X be any two points. Since X is contractible, H : X I X, that is the
homotopy from the identity map to a constant map in X. Consider the map f : I X,
defined by f (s) = H(x, 2s) for 0 s 1/2 and f (s) = H(y, 2 2s) for 1/2 s 1.
(c) Let f, g Homot(X; Y). Since Y is contractible, H : Y I Y, that is the homotopy
from the identity map to a constant map in Y. Consider the homotopy K : X I Y,
defined by K(x, t) = H( f (x), 2t) for 0 t 1/2 and K(x, t) = H(g(x), 2 2t) for
1/2 t 1.
(d) Since X is contractible, idX is homotopic to a constant map c : X {x0 } X. By
Lemma 8.9 [L], if f : X Y, f idX = f is homotopic to c = f c where f c : X
{y0 := f (x0 )} Y is a constant map.
To prove the claim, we need to show that f  f 0 for any two maps f, f 0 : X Y.
From the first paragraph, f and f 0 are homotopic to constant maps c and c 0 . By the
same argument in the last part of the proof of Problem (2,b), we can show that c  c 0
since Y is path-connected. Thus f  c  c 0  f 0 .
(4) Spaces X and Y is said to have the same homotopy type if there are maps f : X Y and
g : Y X such that f g is homotopic to 1Y and g f is homotopic to 1X .
(a) Show that if X is contractible, then X and a single point have the same homotopy type.
(b) Show that if X and Y are homeomorphic, then X and Y have the same homotopy type.
Solution:
(a) Since X is contractible, H : XI X, that is the homotopy from the identity map, 1X
to a constant map 1c for some point c X. Let f : X c be a constant map f (x) = c
and g : c X be the inclusion map g(c) = c. Then clearly we have f g sends c to c.
By assumption, since X is contractible, we also have g f = 1c is homotopic to 1X .
(b) Take f to be the homeomorphism from X to Y and g = f 1 .
References
[M]
[S]
[L]

Munkres, Topology.
Basic Set Theory, http://www.math.cornell.edu/matsumura/math4530/basic set theory.pdf
Lecture notes, available at http://www.math.cornell.edu/matsumura/math4530/math4530web.html

MATH 4530 Topology. HW 8 solutions


Please declare any collaborations with classmates; if you find solutions in books or online,
acknowledge your sources in either case, write your answers in your own words.
Please attempt all questions and justify your answers.

Write the proofs in complete sentences.


(1) Let f, g : I X be paths from x0 to x1 and let k : X Y be a continuous map.
(1) Show that if F : f p g is a path homotopy, then k F : k f p k g is a path homotopy.
(2) Show that if f (1) = g(0), then k ( f g) = (k f ) (k g).
(2) Let Y be a discrete space. Show that if p : X Y X is the projection to the first component,
then p is a covering map.
(3) Let p : E B be a covering map.
(a) Show that if B is Hausdorff, then E is Hausdorff.
(b) Show that if B is compact and p1 (b) is finite for each b B, then E is compact.
(4) Let S 1 := U(1) := {ei } C. Show that pn : S 1 S 1 , z 7 zn is a covering map.
(5) Let h, k : X Y be continuous maps and suppose h(x0 ) = y0 and k(x0 ) = y0 . Show that if there
is a homotopy F : X [0, 1] Y from h to k such that F(x0 , t) = y0 for all t [0, 1], then
h = k : 1 (X, x0 ) 1 (Y, y0 ).
References
[M]
[S]
[L]

Munkres, Topology.
Basic Set Theory, http://www.math.cornell.edu/matsumura/math4530/basic set theory.pdf
Lecture notes, available at http://www.math.cornell.edu/matsumura/math4530/math4530web.html

MATH 4530 Topology. HW 8 solutions


Please declare any collaborations with classmates; if you find solutions in books or online,
acknowledge your sources in either case, write your answers in your own words.
Please attempt all questions and justify your answers.

Write the proofs in complete sentences.

(1) Let f, g : I X be paths from x0 to x1 and let k : X Y be a continuous map.


(1) Show that if F : f p g is a path homotopy, then k F : k f p k g is a path homotopy.
(2) Show that if f (1) = g(0), then k ( f g) = (k f ) (k g).
Solution:
(a) k F : I [0, 1] Y is a continuous map. Since k F(s, 0) = k f (s), k F(s, 1) = g(s),
k F(0, t) = k(x0 ) and k F(1, t) = k(x1 ), k F is a path-homotopy from k f to k g.
(b)

s [0, 1/2]
k f (2s)
(k f ) (k g)(s) =

k g(2s 1) s [1/2, 1]
If s [0, 1/2], k ( f g)(s) = k(( f g)(s)) = k( f (2s)) and if s [1/2, 1], k ( f g)(s) =
k(( f g)(s)) = k(g(2s 1)). Therefore k ( f g) = (k f ) (k g).
(2) Let Y be a discrete space. Show that if p : X Y X is the projection to the first component,
then p is a covering map.
Solution: Let b B and Ub a nbhd of b. Then p1 (Ub ) = Ub Y. Since Y is discrete,
Ub Y = tyY Ub {y}. Since p|Ub {y} Ub {y} Ub is a homeomorphism, p is a covering
map.
(3) Let p : E B be a covering map.
(a) Show that if B is Hausdorff, then E is Hausdorff.
(b) Show that if B is compact and p1 (b) is finite for each b B, then E is compact.
Solution:
(a) Let e1 and e2 be distinct points in E. If p(e1 ) = p(e2 ) = b, then let Ub be a nbhd of b
that is evenly covered by p where p1 (Ub ) = t V . Since each V is homeomorphic
to Ub under p, e1 and e2 must be contained in different V s, so e1 and e2 can be
separated by open sets. If p(e1 ) =: b1 is distinct from p(e2 ) =: b2 . Then there are
nbhds U1 and U2 of b1 and b2 such that U1 U2 = . Since p is a covering map, there
are nbhds V1 and V2 of b1 and b2 such that V1 and V2 are evenly covered by p. Let
Oi := Vi Ui which is a nbhd of bi such that O1 O2 = and Oi is evenly covered by
p so that p1 (Oi ) = ti Ei . If Ei is the open set containing ei , then since O1 O2 = ,
E1 E2 = . Thus e1 and e2 can be separated by open sets.
1

Solution:
(b)
First we prove that for any open set O E containing a fiber p1 (b), there is
a small nbhd Wb of b in B such that p1 (Wb ) O. The proof of this statement
is optional and you can use it to show the main statement. Let Ub be an evenly
covered nbhd of b, so that 1 (Ub ) = V1 t t Vm (p1 (b) is finite). Let
1

Wi := p(O Vi ). Let Wb := m
i=1 Wi . Then p (Wb ) = tWi where Wi
1
O Vi O. Thus p (Wb ) O.
Since p is a covering map, p is an open map (p336 [M] the last paragraph). Suppose that {U } covers E. For each b, we have a finite subcovers {Ub,1 , , Ub,nb } {U } of p1 (b). Let Wb be a nbhd of b in B such that
b
p1 (Wb ) Vb := ni=1
Ub,i (Use the statement above). Since {Wb , b B} covers B and B is compact, we have a finite subcover {Wbk , k = 1, , m}. Since
{p1 (Wbk ), k = 1, , m} covers E, {Vbk , k = 1, , m} covers E. Therefore
{Ubk ,ibk , ibk = 1, , nbk , k = 1, , m} is a desired finite subcovering of {U },
that covers E.
(4) Let S 1 := U(1) := {ei } C. Show that pn : S 1 S 1 , z 7 zn is a covering map.
Solution: Let U1 , , U4 be the open covering of S 1 from Theorem 9.17 [L]. We show
that U4 is evenly covered by pn . Other U1 , U2 , U3 are similar.
U4 = {ei : = (0, )}.
Then
p1 (U4 ) = V1 t t Vn
where Vk = {ei : = (2k/n, 2k/n+/n)}. Since pn |V k : V k U 4 is a bijective continuous
map, V k is compact and U4 is Hausdorff, pn |V k is a homeomorphism and so is pn |Vk .
(5) Let h, k : X Y be continuous maps and suppose h(x0 ) = y0 and k(x0 ) = y0 . Show that if there
is a homotopy F : X [0, 1] Y from h to k such that F(x0 , t) = y0 for all t [0, 1], then
h = k : 1 (X, x0 ) 1 (Y, y0 ).
Solution: Let [ f ] 1 (X, x0 ). We need to show that [h f ] = [k f ] in 1 (Y, y0 ). Define
t) = F( f (s), t). It is a composition of maps F ( f, id[0,1] ) so it is
F : I [0, 1] Y by F(s,
continuous. Now check
0) = F( f (s), 0) = h( f (s)), F(s,
1) = F( f (s), 1) = k( f (s)) homotopy conditions
F(s,

t) = F( f (1), t) = F(x0 , t) = y0 ,
F)(0,
t) = F( f (0), t) = F(x0 , t) = y0 , F(1,
Thus F is a path-homotopy from h f to k f .

path-homotopy conditions.

References
[M]
[S]
[L]

Munkres, Topology.
Basic Set Theory, http://www.math.cornell.edu/matsumura/math4530/basic set theory.pdf
Lecture notes, available at http://www.math.cornell.edu/matsumura/math4530/math4530web.html

MATH 4530 Topology. HW 9


Please declare any collaborations with classmates; if you find solutions in books or online,
acknowledge your sources in either case, write your answers in your own words.
Please attempt all questions and justify your answers.

Write the proofs in complete sentences.


(1) Let p : E B be a covering map. Let f, g are composable paths in B, i.e. f (1) = g(0). If f, g are
composable paths lifted from f, g, then show that f g is a lifting of f g.
(2) Show that the fundamental group of a torus S 1 S 1 is isomorphic to Z Z as groups (additive on
Z Z). (Hint; generalized the proof of 1 (S 1 , b0 )  Z.)
(3) A group G acts on a set X from right if there is an action map G X X, (g, x) 7 xg which
satisfies x = x1G and x(gh) = (xg)h. Show that there is a natural right action of 1 (B, b0 ) on
p1 (b0 ) if p : E B is a covering map. (Hint: use e0 in Section 10.2 [L]).
(4) Let B be a simply-connected space.Then any covering map p : E B with E path-connected, is
a homeomorphism.
(5) Show that the map p : S 1 S 1 , z 7 zn induces p : 1 (S 1 , b) 1 (S 1 , b), [ f ] 7 [ f ]n . In other
words, through the isomorphism in Section 10.4 [L], Z Z, m 7 nm.
R
[M]
[S]
[L]

Munkres, Topology.
Basic Set Theory, http://www.math.cornell.edu/matsumura/math4530/basic set theory.pdf
Lecture notes, available at http://www.math.cornell.edu/matsumura/math4530/math4530web.html

MATH 4530 Topology. HW 9


Please declare any collaborations with classmates; if you find solutions in books or online,
acknowledge your sources in either case, write your answers in your own words.
Please attempt all questions and justify your answers.

Write the proofs in complete sentences.


(1) Let p : E B be a covering map. Let f, g are composable paths in B, i.e. f (1) = g(0). If f, g are
composable paths lifted from f, g, then show that f g is a lifting of f g.
(2) Show that the fundamental group of a torus S 1 S 1 is isomorphic to Z Z as groups (additive on
Z Z). (Hint; generalized the proof of 1 (S 1 , b0 )  Z.)
Solution
Let S 1 := {ei , R} and p : R S 1 , 7 e2i . Let p := (p, p) : R R S1 S1 .
Let [ f ], [g] 1 (S 1 S 1 , (1, 1)). Let f, g be the lifts of f and g at (0, 0) along p and
f(1) =: (n1 , m1 ) and g (1) =: (n2 , m2 ). We need to show that if f g is the lift of f g at (0, 0)
along p, then fg
g(1) = (n1 + n2 , m1 + m2 ). Consider g 1 (s) := (n1 , m1 ) + g (s). Then g is the
lift of g at (n1 , m1 ) along p by the invariance of p under the shifting (p(~x + (n + m)) = p(~x)).
Since g 1 (0) = (n1 , m1 ), by (1), fg
g = f g 1 . Thus fg
g(1) = g 1 (1) = (n1 + n2 , m1 + m2 ).
(3) A group G acts on a set X from right if there is an action map G X X, (g, x) 7 xg which
satisfies x = x1G and x(gh) = (xg)h. Show that there is a natural right action of 1 (B, b0 ) on
p1 (b0 ) if p : E B is a covering map. (Hint: use e0 in Section 10.2 [L]).
(4) Let B be a simply-connected space.Then any covering map p : E B with E path-connected, is
a homeomorphism.
(5) Show that the map p : S 1 S 1 , z 7 zn induces p : 1 (S 1 , b) 1 (S 1 , b), [ f ] 7 [ f ]n . In other
words, through the isomorphism in Section 10.4 [L], Z Z, m 7 nm.
Solution Under the isomorphism in Theorem 10.8 [L], it is enough to prove p (1) = n,
since Z is generated by 1. More concretely, it is enough because f (m) = f (1 + + 1) =
f (1) + + f (1) = m f (1) = mn. Let S 1 = {e2i } and p : R S 1 , 7 e2i . Then it
is clear that p|
I : I S 1 is a loop at 1 and its lift at 0 R is the inclusion j : I , R.
Thus 0 ([ p I ]) = j(1) = 1. p (1) = p ([ p|
I ]) = [p p|
I ]. Then the lift of p p I at 0 is
p]
p|I : I R, s 7 ns (Check this!). Now p]
p|I (1) = n.

References
[M]
[S]
[L]

Munkres, Topology.
Basic Set Theory, http://www.math.cornell.edu/matsumura/math4530/basic set theory.pdf
Lecture notes, available at http://www.math.cornell.edu/matsumura/math4530/math4530web.html

MATH 4530 Topology. HW 9 solutions


Please declare any collaborations with classmates; if you find solutions in books or online,
acknowledge your sources in either case, write your answers in your own words.
Please attempt all questions and justify your answers.

Write the proofs in complete sentences.


(1) Let p : E B be a covering map. Let f, g are composable paths in B, i.e. f (1) = g(0). If f, g are
composable paths lifted from f, g, then show that f g is a lifting of f g.
Solution (Ex 54.3 [M]): We need to show that p ( f g ) = f g. If s [0, 1/2], then
p ( f g )(s) = p f(2s) = f (2s) = ( f g)(s). If s [1/2, 1], then p ( f g )(s) =
p g (2s 1) = g(2s 1) = ( f g)(s).
(2) Show that the fundamental group of a torus S 1 S 1 is isomorphic to Z Z as groups (additive on
Z Z). (Hint; generalized the proof of 1 (S 1 , b0 )  Z.)
Solution (Ex 54.7 [M]): Since R R is simply-connected, by Theorem 10.7 [L], e0 :
1 (S 1 S 1 , b0 ) ZZ, [ f ] 7 [ f(1)] is a bijective map where b0 = ((1, 0), (1, 0)) S 1 S 1
and e0 = (0, 0) Z Z. We need to show that this map is a group homomorphism.
(3) A group G acts on a set X from right if there is an action map G X X, (g, x) 7 xg which
satisfies x = x1G and x(gh) = (xg)h. Show that there is a natural right action of 1 (B, b0 ) on
p1 (b0 ) if p : E B is a covering map. (Hint: use e0 in Section 10.2 [L]).
Solution: Define an action map
1 (B, b0 ) p1 (b0 ) p1 (b0 ), ([ f ], e0 ) 7 e0 ([ f ]) = f(1).
If f is a constant path at b0 , then [ f ] = 1 is the identity. The lift of f at e0 is a constant path
at e0 , thus e0 ([ f ]) = e0 . This proves x1G = x axiom.
By Problem (1),
e0 ([ f ] [g]) = e0 ([ f g]) = fg
g(1) = ( f g )(1) = g (1)
where fg
g is the lift of f g at e0 and f is the lift of f at e0 and g is the lift of g at f(1).
(e0 [ f ])[g] = ( f(1))[g] = g (1).
(4) Let B be a simply-connected space.Then any covering map p : E B with E path-connected, is
a homeomorphism.
Solution (Ex 54.8 [M]): Since B is simply-connected, 1 (B, b0 ) is trivial. Since e0 :
1 (B, b0 ) p1 (b0 ) is surjective (E path-connected and Theorem 10.7 [L]), p1 (b0 ) = {e0 }.
This is true for any b0 , therefore p must be bijective. Since p is an open map, the inverse is
continuous too: p is a homeomorphism.
(5) Show that the map p : S 1 S 1 , z 7 zn induces p : 1 (S 1 , b) 1 (S 1 , b), [ f ] 7 [ f ]n . In other
words, through the isomorphism in Section 10.4 [L], Z Z, m 7 nm.
R
[M]
[S]
[L]

Munkres, Topology.
Basic Set Theory, http://www.math.cornell.edu/matsumura/math4530/basic set theory.pdf
Lecture notes, available at http://www.math.cornell.edu/matsumura/math4530/math4530web.html
1

MATH 4530 Topology. HW 10


Please declare any collaborations with classmates; if you find solutions in books or online,
acknowledge your sources in either case, write your answers in your own words.
Please attempt all questions and justify your answers.

Write the proofs in complete sentences.


(1) (How to show two spaces cant be homeomorphic to each other)
(a) Show that R1 is not homeomorphic to Rn , n > 1.
(b) Show that R2 is not homeomorphic to Rn , n > 2.
Hint: recall how you showed that (0, 1] and (0, 1) cant be homeomorphic to each other. That
might help.
Note: once we compute higher homotopy groups for S n , we can show that Rn and Rm are note
homeomorphic when n , m.
(2) Let X be the union of the two copies of S n , n 2 having a single point in common. What is the
fundamental group of X?
(3) Assume that the following statement holds: There is no retraction r : Bn+1 S n for each n.
(a) Prove that the identity map idS n : S n S n is not nullhomotopic, i.e. is not homotopic to a
constant map.
(b) Prove that the inclusion map j : S n , Rn+1 {~0} is not nullhomotopic.
(c) Explain why every continuous map f : Bn+1 Bn+1 must have a fixed point, i.e. f (x) = x
for some x Bn+1 .
Again, once we compute the higher homotopic groups of S n , we can prove the assumed statement
similarly to n = 1 case.
References
[M]
[S]
[L]

Munkres, Topology.
Basic Set Theory, http://www.math.cornell.edu/matsumura/math4530/basic set theory.pdf
Lecture notes, available at http://www.math.cornell.edu/matsumura/math4530/math4530web.html

MATH 4530 Topology. HW 10 solutions


Please declare any collaborations with classmates; if you find solutions in books or online,
acknowledge your sources in either case, write your answers in your own words.
Please attempt all questions and justify your answers.

Write the proofs in complete sentences.

(1) (How to show two spaces cant be homeomorphic to each other)


(a) Show that R1 and Rn , n > 1 are not homeomorphic.
(b) Show that R2 and Rn , n > 2 are note homeomorphic.
Hint: recall how you showed that (0, 1] and (0, 1) cant be homeomorphic to each other. That
might help.
Note: once we compute higher homotopy groups for S n , we can show that Rn and Rm are note
homeomorphic when n , m.
Solution
(a) Suppose that there is a homeomorphism f : R1 Rn . It induces a homeomorphism
f |R{0} : R {0} Rn { f (0)}. The domain is not connected but Rn { f (0)} is
connected for n > 1. Thus the assumption leans to a contradiction.
(b) Suppose that there is a homeomorphism f : R2 Rn . It induces a homeomorphism
f |R2 {~0} R2 {~0} Rn { f (~0)}. Since R2 {0} has a deformation retract to S 1 , the
fundamental group is isomorphic to Z. But the fundamental group of Rn { f (~0)}, n > 2
is trivial, since it has a deformation retract to S n1 which has trivial fundamental group.
Thus they cant be homeomorphism. The assumption leads to a contradiction.
(2) Let X be the union of the two copies of S n , n 2 having a single point in common. What is the
fundamental group of X?
Solution Let X = S 1 S 2 where S 1 and S 2 are copies of S n and S 1 S 2 = ~x. Let ~p and ~q
be points on S 1 and S 2 respectively and ~p, ~q , 0. Then U1 := X {~p} and U2 := X {~q}
are open sets in X. Since S n {~p} is homeomorphic to Rn by stereographic projection,
U1 , U2 are homeomorphic to the union of Rn and S n having a single point in common.
Since Rn has a deformation retract to any point, U1 and U2 have deformation retracts to
S n . Therefore the fundamental groups of U1 and U2 are trivial. The intersection U1 U2
is homeomorphic to Rn Rn where the intersection is the point x. The union of the pathconnected subspaces having a common point is path-connected. Thus applying Theorem
11.7 [L], the fundamental group of X must be trivial.
(3) Assume that the following statement holds: There is no retraction r : Bn+1 S n for each n.
(a) Prove that the identity map idS n : S n S n is not nullhomotopic, i.e. not homotopic to a
constant map.
(b) Prove that the inclusion map j : S n , Rn+1 {~0} is not nullhomotopic.
(c) Explain why every continuous map f : Bn+1 Bn+1 must have a fixed point, i.e. f (x) = x
for some x Bn+1 .
Again, once we compute the higher homotopic groups of S n , we can prove the assumed statement
similarly to n = 1 case.
1

Solution
(a) Suppose that idS n is nullhomotopic. Then there is a homotopic H : S n [0, 1] S n
such that H(b, 0) = b and H(b, 1) = b0 . Thus similarly to the argument in the proof
1 2 of Lemma 11.10, H factors through : S n [0, 1] Bn+1 , b 7 (1 t)b, which
is a quotient map similarly to the argument there again. Thus by *2 there, there is a
continuous map r : Bn+1 S n such that H = r . H|S 1 {0} = idS n and |S n is the
natural inclusion. Therefore r must be a retraction. It leads to a contradiction to the
assumption.
(b) By the same argument as in (a), the homotopy H from j to a constant map defines
a continuous map r : Bn+1 Rn+1 {~0} such that if l : S n , Bn+1 is the natural
inclusion, then r l = j. Let k : Rn+1 {~0} S n be the deformation retraction defined
in Example 10.16 [L]. Then k r l = idS n so that k r is a retraction from Bn+1 to S n .
Thus we have a contraction.
(c) If there is f : Bn+1 Bb+1 that has no fixed point, then we ca define a retraction
r : Bn+1 S n by
x 7 the intersection with S n and the half line from f (x) through x
Thus we have a contradiction, and therefore any map f must have a fixed point.
References
[M]
[S]
[L]

Munkres, Topology.
Basic Set Theory, http://www.math.cornell.edu/matsumura/math4530/basic set theory.pdf
Lecture notes, available at http://www.math.cornell.edu/matsumura/math4530/math4530web.html

MATH 4530 Topology. HW 11


Please declare any collaborations with classmates; if you find solutions in books or online,
acknowledge your sources in either case, write your answers in your own words.
Please attempt all questions and justify your answers.

Write the proofs in complete sentences.


(1) Apply Theorem 12.6 [L] (See also Remark 12.7 [L], Theorem 72.1 [M]) and compute the fundamental group of RP2 .
(2) Can we find examples to show that a simple loop in X may or may not separate X?:
(a) X = RP2 .
(b) X = a torus.
(3) What are the fundamental group of the following spaces? Explain.
(a) R3 {(t, t2 , 0)}
(b) RP2 {a point}.
(c) The space consisting of vertices and edges of Tetrahedron. (4 vertices and 6 edges).
References
[M]
[S]
[L]

Munkres, Topology.
Basic Set Theory, http://www.math.cornell.edu/matsumura/math4530/basic set theory.pdf
Lecture notes, available at http://www.math.cornell.edu/matsumura/math4530/math4530web.html

MATH 4530 Topology. Practice Problems For Final Part I


(1) Apply Theorem 12.6 [L] (See also Remark 12.7 [L], Theorem 72.1 [M]) and compute the fundamental group of RP2 ](Klein Bottle) where ] means the connected sum defined in Section 12.1
[L].
(2) Let h : S 1 S 1 R2 {~0} be a continuous map. Show that deg h = n(h, ~0). The deg h is defined
in Prelim II and n(h, ~0) is the winding number of h around ~0 defined in Section 13 [L].
(3) Consider the group G = hx, y | x2 , y2 , xy = yxi.
(a) Show that G  Z2 Z2 .
(b) Find a space B such that 1 (B, b)  G and its simply-connected covering space. Justify your
answer.
(4) Find a space X with the fundamental group which is isomorphic to hx, y, z | yz = zyi.
(5) Show that if a space X has the property that every continuous map f : X X has a fixed point,
then each retract Y of X has the property too.
(6) Find an example that Borsuk Lemma 13.15 (3) [L] doesnt hold if the map f is not injective.
(7) Let G be a topological group. Show that 1 (G, 1G ) is a commutative (abelian) group.
(8) Prove or disprove the following statement:
(a) If X is connected, then 1 (X, x1 ) is isomorphic to 1 (X, x2 ) for every x1 , x2 X.
(b) If A and B are deformation retract of C, then A are B are homotopy equivalent (i.e. have the
same homotopy type.)
(c) If p : E B is a covering map with p(e) = b and 1 (B, b) is abelian, then 1 (E, e) is abelian,
too.
(9) Let GL(2, R) denote the general linear group, the group of invertible 2 2 matrices with real
coefficients. Let O(2) denote the orthogonal group, the 2 2 orthogonal matrices. Finally, SO(2)
is the special orthogonal group, those matrices Q O(2) satisfying det(Q) = 1. Let I denote the
two-by-two identity matrix.
(a) Determine whether or not GL(2, R) is connected. Justify your response.
(b) The Gram-Schmidt orthogonalization process allows us to write a matrix A GL(2, R)
uniquely as a product A = QR where Q O(2) is orthogonal and R is upper triangular with
positive entries on the diagonal. Use this to produce a deformation retraction from GL(2, R)
to O(2).
(c) Show that SO(2), with matrix multiplication, is homeomorphic(as a topological space) to and
isomorphic (as a group) to S 1 = (R, +)/Z.
(d) Using the fact that O(2) = SO(2) {1, 1}, compute 1 (GL(2, R), I).
(10) Let A be a compact contractible subspace of S 2 . Show that A does not separate S 2 .
References
[M]
[S]
[L]

Munkres, Topology.
Basic Set Theory, http://www.math.cornell.edu/matsumura/math4530/basic set theory.pdf
Lecture notes, available at http://www.math.cornell.edu/matsumura/math4530/math4530web.html

MATH 4530 Topology. Practice Problems For Final Part II


(1) (One point compactification) Assume that X is a non-compact connected Hausdorff space in which
every point has a compact neighborhood. Define X 0 := X t {} as a set. You may use the fact that
the intersection of a family of compact sets in a Hausdorff space is compact and the fact that the
union of a finite collection of compact sets is compact.
(a) Define a topology on X 0 as follows: a subset U X 0 is open if (i) it is an open subset of X
if U X, and (ii) X 0 U is a compact subset in X if it is not a subset of X. Prove that this
actually defines a topology on X 0 .
(b) Show that X is a subspace of X 0 .
(c) Show that X 0 is compact.
(d) Show that X 0 is connected.
(e) Show that if X is R2 with the usual topology, then X 0 is homeomorphic to the 2-sphere S 2 .
(2) (a) Explain why (and how) a continuous map f : X Y with f (x) = y induces a group homomorphism 1 (X, x) 1 (Y, y).
(b) Use the fact that 1 (S 1 )  Z to prove Brouwers Fixed Point Theorem: for every continuous
map f : D2 D2 , there is a D2 such that f (a) = a.
(3) Let X be any topological space, Y a Hausdorff space, and f : X Y a continuous map. The graph
of f is defined as the subspace
G f := {(x, f (x)) X Y | x X}.

(4)

(5)

(6)

(7)
(8)

(a) Show that G f is a closed subspace.


(b) Find a counter example to part (a) in the case when Y is not Hausdorff.
(c) If f : X Y is a map and G f is closed, then f must be continuous?
Let X be a topological space, and A and B compact subspaces.
(a) Show that A B is compact.
(b) Show that if X is Hausdorff, then A B is compact.
(c) Give a counterexample to part (b) in the case when X is not Hausdorff.
(a) Let X be a Hausdorff space. Show that any connected subset A X contains one or infinitely
many elements.
(b) Let A be a countable subset of R2 . Prove that R2 A is path-connected.
Determine whether or not there is a retraction from X to A for the following spaces. If there is a
retraction, describe it explicitly, using pictures if you like.
(a) X is S 1 D2 and A is S 1 S 1 .
(b) X is S 1 S 1 and A = {(x, x) X | x S 1 }.
Prove that a surjective map from a compact space to a Hausdorff space is a quotient map.
Prove that S 1 := {e2i , R} C is homeomorphic to the quotient space obtained from [0, 1] by
identifying 0 and 1.
References

[M]
[S]
[L]

Munkres, Topology.
Basic Set Theory, http://www.math.cornell.edu/matsumura/math4530/basic set theory.pdf
Lecture notes, available at http://www.math.cornell.edu/matsumura/math4530/math4530web.html

MATH 4530 Topology. Practice Problems For Final Part II solutions

(1) (One point compactification) Assume that X is a non-compact connected Hausdorff space in which
every point has a compact neighborhood. Define X 0 := X t {} as a set. You may use the fact that
the intersection of a family of compact sets in a Hausdorff space is compact and the fact that the
union of a finite collection of compact sets is compact.
(a) Define a topology on X 0 as follows: a subset U X 0 is open if (i) it is an open subset of X
if U X, and (ii) X 0 U is a compact subset in X if it is not a subset of X. Prove that this
actually defines a topology on X 0 .
(b) Show that X is a subspace of X 0 .
(c) Show that X 0 is compact.
(d) Show that X 0 is connected.
(e) Show that if X is R2 with the usual topology, then X 0 is homeomorphic to the 2-sphere S 2 .
Solution:
(a)
T since is an open subset of X.
X 0 T since X 0 X 0 = is a compact subset in X and X 0 is not a subset of
S X.
Let {U j } satisfy condition (i), and
{V
}
satisfy
condition
(ii).
Clearly,
U
:=
U j still
k
S
satifies condition (i), and V := Vk still satifies condition (ii) since the intersection
of a family of compact sets in a Hausdorff space is compact. Note that X 0 \(U V) =
(X 0 \ U) (X 0 \ V), where X 0 \ U is closed and X 0 \ V is compact in the Hausdorff
space X and so the resulting set is compact in X (since it is a closed subset of a
compact set). So by condition (ii), U V T .
Let {U j : 1 j n}Tsatisfy condition (i), and {U j : n + 1 j T
m} satisfy condition
n
(ii). Clearly, V := i U j still satifies condition (i), and W := m
n+1 U j still satifies
condition (ii) since the union of finite collection of compact sets is compact. Since
X 0 W is compact in a Hausdorff space X, it is closed, and so W = X \ (X 0 W) is
open in X. Thus, we have V W T .
(b) Given U T . If U X, then X U = U is open in X by condition (i). If U 1 X, then
X 0 U is compact subset of X, so X 0 U is closed in X and so X U = X \ (X 0 U) is
open in X. So, X is a subspace of X 0 .
(c) Let {{Ui }iI {V j } jJ } be any open covering of X 0 , where each Ui satisfies condition (i) and
each V j satisfies condition (ii). Note that for each j, V j \ {} is an open set in X since it
is the complement of a compact (hence closed) set X 0 \ V j . Also note that J is non-empty
since one of the sets must cover . Let V be an arbitrary element of {V j } jJ . Then X \ V
is compact by condition (ii) and is covered by {Ui }iI {V j \ {}} jJ . Therefore X \ V
admits a finite subcover {Ui }ni=1 {V j \ {}}mj=1 . Now clearly {V {Ui }ni=1 {V j }mj=1 } is a
finite subcover for X 0 .
(d) Assume X 0 is not connected, i.e. open sets U, V such that X 0 = U V and U V = .
WLOG, say U, then V is compact in X by condition (ii), and thus V is closed. Since
V X, by condition (i), V is open in X, contradicting the fact that X is connected. Hence,
X 0 is connected.
(e) Consider the bijective map p : Rt{} S 2 defined by sending to the north pole of the
sphere and the rest of the map defined by stereographic projection. By part (c), R t {} is
compact. S 2 is a subspace of a Hausdorff space, so it is Hausdorff. A continuous bijection
from a compact space to a Hausdorff space is a homeomorphism.
(2) (a) Explain why (and how) a continuous map f : X Y with f (x) = y induces a group homomorphism 1 (X, x) 1 (Y, y).
1

(b) Use the fact that 1 (S 1 )  Z to prove Brouwers Fixed Point Theorem: for every continuous
map f : D2 D2 , there is a D2 such that f (a) = a.
Solution:
(a) If : I X is a loop at x, then f : I Y is a loop at y. If : I X is a loop at x such
that  p , then f is a loop at y and f  p f . This path-homotopy is given by
f H : I [0, 1] Y if H : I [0, 1] X is a path-homotopy from to .
(b) Suppose that there is no such fixed point, i.e. f (x) , x for all x D2 . Then for each x,
consider the half line from f (x) to x. This line intersects with S 1 . Let this point be denoted
by r(x) So define a map
r : B2 S 1 , x 7 r(x).
This map is well-defined because there is no fixed point. This map is continuous. This
map r is a retraction from D2 to S 1 and thus the inclusion i : S 1 D2 induces an injection
on fundamental groups. It is impossible since 1 (S 1 ) = Z and 1 (D2 ) = {1}.
(3) Let X be any topological space, Y a Hausdorff space, and f : X Y a continuous map. The graph
of f is defined as the subspace

G f := {(x, f (x)) X Y | x X}.

(a) Show that G f is a closed subspace.


(b) Find a counter example to part (a) in the case when Y is not Hausdorff.
(c) If f : X Y is a map and G f is closed, then f must be continuous?
Solution:
(a) Let (x, y) < G f , i.e f (x) , y in Y. Since Y is Hausdorff, there are open nbhds Uy and U f (x)
such that Uy U f (x) = . Then consider U := f 1 (U f (x) ) Uy which is an open nbd of
(x, y). We can show that U G f = which proves that G f is closed: let (a, b) U. Then
f (a) U f (x) . Since b Uy , b , f (a).
(b) Let X be any topological space that is not Hausdorff and let f : X X be the identity
map on X. Then by HW3 question 1, we know that the diagonal = G f is not closed.
(c) I havent gotten the complete answer yet.
(4) Let X be a topological space, and A and B compact subspaces.
(a) Show that A B is compact.
(b) Show that if X is Hausdorff, then A B is compact.
(c) Give a counterexample to part (b) in the case when X is not Hausdorff.
Solution
(a) Let K = A B. Let {U K} be an open covering of K where U s are open sets in
X. Then {U A} and {U B} are open coverings of A and B. Since A, B are compact,
we find a finite subcoverings {Vi A, i = 1, , n} and {W j B, j = 1, , m} where
{Vi }, {W j } {U }. Then {Vi K, W j K, i = 1, , n, j = 1, , m} is a finite subcovering
of K.
(b) Let L = A B and K = A B. Since X is Hausdorff and A, B compact, by Thm 26.3, A
and B are closed. Thus, A B is closed. Since A B is a closed subset of a compact set
A, it must be compact by Thm 26.2.

Solution
(c) Let X be the Cartesian product of the real line with usual topology and the set {0, 1} with
trivial topology. Let A = {[a, b] 0} {(a, b) 1} and B = {(a, b) 0} {[a, b] 1}. Note
that A is compact: given an open cover of A, say {Ui } = {Ui0 {0, 1}}, there is a finite
subcover for [a, b] consisting of finitely many Ui0 . Similarly, B is compact. But clearly
A B = (a, b) {0, 1} is not compact since a {0, 1} is a limit point.
(5) (a) Let X be a Hausdorff space. Show that any connected subset A X contains one or infinitely
many elements.
(b) Let A be a countable subset of R2 . Prove that R2 A is path-connected.
Solution
(a) Say A is connected, with 1 < |A| < , say A = {a1 , a2 , . . . , an }. Since X is Hausdorff, i,
T
S
Ui , Vi such that a1 Ui , ai Vi and Ui Vi = . Let U = i Ui and V = i Vi . Then
U and V separate A, contradicting the assumption. Thus, A has one or infinitely many
elements.
(b) Consider points x, y R2 A. Note that there are uncountably many disjoint paths in the
plane from x to y. Since A is countable, there exists a path that does not intersect A. Since
x, y were arbitrary, R2 A is path-connected.
(6) Determine whether or not there is a retraction from X to A for the following spaces. If there is a
retraction, describe it explicitly, using pictures if you like.
(a) X is S 1 D2 and A is S 1 S 1 .
(b) X is S 1 S 1 and A = {(x, x) X | x S 1 }.
Solution:
(a) If there is a retraction from X to A, then i : 1 (A) 1 (X) is injective. 1 (X) is Z because
1 (S 1 D2 ) 1 1 (S 1 ) 1 (D2 ) 2 1 (S 1 ) {1} (1 is by Thm 11.14 [L] and 2 is by
Lemma 9.2 [L]). On the other hand, 1 (S 1 S 1 ) is Z Z. So it is impossible.
(b) r : S 1 S 1 A defined by (x, y) (x, x) is a retraction.
(7) Prove that a surjective map from a compact space to a Hausdorff space is a quotient map.
Solution: A surjective map from a compact space E to a Hausdorff space X is a closed map and
so a quotient map: a closed set A E is compact since E is compact, the image of a compact
subspace is compact, a compact subspace in a Hausdorff space is closed.
(8) Prove that S 1 := {e2i , R} C is homeomorphic to the quotient space obtained from [0, 1] by
identifying 0 and 1.
Solution: Consider the map p : [0, 1] S 1 , 7 e2i . It factors through the quotient map
q : [0, 1] [0, 1]/ and there is a continuous bijection [0, 1]/ S 1 . Since [0, 1] is compact,
the image of q, [0, 1]/ is compact. S 1 is a subspace of a Hausdorff space, so it is Hausdorff.
A continuous bijection from a compact space to a Hausdorff space is a homeomorphism.

References
[M]
[S]

Munkres, Topology.
Basic Set Theory, http://www.math.cornell.edu/matsumura/math4530/basic set theory.pdf

[L]

Lecture notes, available at http://www.math.cornell.edu/matsumura/math4530/math4530web.html

MATH 4530 Topology. Practice Problems For Final solutions


Write the proofs in complete sentences.
(1) Apply Theorem 12.6 [L] (See also Remark 12.7 [L], Theorem 72.1 [M]) and compute the fundamental group of RP2 ](Klein Bottle) where ] means the connected sum defined in Section 12.1
[L].
Solution
(2) Let h : S 1 S 1 R2 {~0} be a continuous map. Show that deg h = n(h, ~0). The deg h is defined
in Prelim II and n(h, ~0) is the winding number of h around ~0 defined in Section 13 [L].
Solution Let S 1 = {e2i } C {0}. It is natural to define n(h, ~0) := n(h p|I , ~0) (Definition
13.3 [L]) where p : R S 1 is the standard covering. Let n := n(h, ~0), i.e. if g is a lift of
g := h p|I , then g (1) g (0) = n. On the other hand, for a generator = [p|I ] of 1 (S 1 , 1)
and if m := deg h, we have h = [h p|I ] = [( p|I )m ] for a path from 1 to h(1)
in S 1 . Since h p|I is path homotopic to ( p|I )m , their lifts to R at the same point
are path homotopic too. Therefore the ending points are the same. Thus if is a lift of
(0)

( p|I )m = p|m
, then (1)
= n. A lift of p|m
is given by q : I R, t 7 mt.
I
I

If is a lift of , then := (q + (0))

( + m) is a lift of p|m
. (1)
= (1)

+m
I

(0)

and (0)
= (1),

thus (1)
= m = n.
(3) Consider the group G = hx, y | x2 , y2 , xy = yxi.
(a) Show that G  Z2 Z2 .
(b) Find a space B such that 1 (B, b)  G and its simply-connected covering space. Justify your
answer.
Solution
(a) Consider : G Z2 Z2 , defined by extending x 7 (1, 0), y 7 (0, 1) to a homomorphism.
(b) We know that 1 (RP2 ) = Z2 and that 1 (X Y)  1 (X) 1 (Y). So if you think of
RP2 as a square with sides appropriately identified, you can think of B as four squares
glued together accordingly. Since the map identifying antipodal points S 2 RP2 is
a covering map and S 2 is simply-connected, S 2 S 2 is a simply-connected covering
space of RP2 RP2 .
(4) Find a space X with the fundamental group which is isomorphic to hx, y, z | yz = zyi.
Solution First think about just y and z. We want them to be the generators that commute, so
that gives us a torus. Now we want another generator x that does not commute with either
x or y. We can achieve this by attaching a loop to some point of the torus. So we get the
space that is a torus and a circle sharing a point.
(5) Show that if a space X has the property that every continuous map f : X X has a fixed point,
then each retract Y of X has the property too.
Solution Let r : X Y be the retraction map. Let g : Y Y be any continuous map.
Define g0 : X X to be the extension of g, say g0 = g r. Clearly g0 is continuous, so there
is a fixed point, say g0 (x0 ) = x0 . Let y0 = r(x0 ), then y0 = r(x0 ) = r(g0 (x0 )) = g0 (x0 ) =
g(r(x0 )) = g(y0 ), where the middle equality is true since g0 (x0 ) Y. Hence, y0 is the fixed
point in Y under g.
1

(6) Find an example that Borsuk Lemma 13.15 (3) [L] doesnt hold if the map f is not injective.
Solution WLOG assume a and b are the north and the south pole of S 2 and the center of the
sphere is at the origin. Consider a map g : [0, 2] S 2 given by g(0) = g(1) = g(2) = 1, 0, 0,
from g([0, 1]) is a clockwise loop around the equator and g([1, 2]) is a counter-clockwise
loop around the equator. Clearly, g is nulhomotopic, but a and b lie in different components
of S 2 g([0, 2]).
(7) Let G be a topological group. Show that 1 (G, 1G ) is a commutative (abelian) group.
Solution
(8) Prove or disprove the following statement:
(a) If X is connected, then 1 (X, x1 ) is isomorphic to 1 (X, x2 ) for every x1 , x2 X.
(b) If A and B are deformation retract of C, then A are B are homotopy equivalent (i.e. have the
same homotopy type.)
(c) If p : E B is a covering map with p(e) = b and 1 (B, b) is abelian, then 1 (E, e) is abelian,
too.
Solution
(a) False. Need path-connectedness to make the sentence true. The counter example would
be as follows. The toplogists sine curve is the space which is connected but not pathconnected. Each path connected component is simply connected so add half circle
going from (0, 1) to (0, 1) counterclockwise. Then the fundamental group of one
component is Z but the other one is trivial.
(b) True.
(c) True. By Thm 54.6, we know that p : 1 (E, e0 ) 1 (B, b0 ) is a monomorphism.
(9) Let GL(2, R) denote the general linear group, the group of invertible 2 2 matrices with real
coefficients. Let O(2) denote the orthogonal group, the 2 2 orthogonal matrices. Finally, SO(2)
is the special orthogonal group, those matrices Q O(2) satisfying det(Q) = 1. Let I denote the
two-by-two identity matrix.
(a) Determine whether or not GL(2, R) is connected. Justify your response.
(b) The Gram-Schmidt orthogonalization process allows us to write a matrix A GL(2, R)
uniquely as a product A = QR where Q O(2) is orthogonal and R is upper triangular with
positive entries on the diagonal. Use this to produce a deformation retraction from GL(2, R)
to O(2).
(c) Show that SO(2), with matrix multiplication, is homeomorphic(as a topological space) to and
isomorphic (as a group) to S 1 = (R, +)/Z.
(d) Using the fact that O(2) = SO(2) {1, 1}, compute 1 (GL(2, R), I).
Solution:
(a) Consider the continuous map det : GL(2, R) R {0}, A 7 det A. This map is
surjective. If GL(2, R) is connected, then R {0} must be connected since the image of
a continuous map from a connected space is connected. But R {0} is not connected.
Thus GL(2, R) is not connected.

Solution
(b)
(i) Let T + be the set of all upper triangular matrices in GL(2, R) with positive diagonal entries. Consider the deformation retract H : R [0, 1] R, (x, t) 7 tx of R
to a point {0}. Consider the homeomorphism ln : R>0 R, x 7 ln x. Use ln to
identify R>0 and R, and get a deformation retract H 0 : R>0 [0, 1] R>0 of R>0
0
to a point({1}. Use
!) H and H to give the deformation retract F : T + [0, 1] T +
1 0
of T + to
.
0 1
(ii) Consider the map m : O(2) T + GL(2, R), (Q, R) 7 QR, which is a continuous bijective map. It is an open map, since if U V O(2) T + , then
m(U V) = UV = vV Uv which is a union of open sets. Thus it is a homeomorphism.
(iii) Now the map (idO(2) , F) : O(2) T + [0, 1] O(2) T + is a deformation retract
of O(2) T + to O(2). By the homeomorphism m, we have found the desired
deformation retract.
(c)
(
!
)
cos t sin t
SO(2) =
, 0 t < 2 .
sin t cos t
Define a map f from S 1 = {ei , 0 < 2} to SO(2) by
ei 7

!
cos sin
.
sin cos

It is a continuous map since it can


! be obtained from the continuous map g :
cos t sin t
[0, 2] SO(2), t 7
by factoring through the standard quotient map
sin t cos t
p : [0, 2] S 1 . The bijectivity can be checked directly. Since it is a map from
compact to Hausdorff, it is a homeomorphism. It is a group homomorphism because
!
!
cos t cos s sin t sin s cos t sin s + sin t cos s
cos(t + s) sin(t + s)
f (eit ) f (eis ) =
=
cos t sin s + sin t cos s cos t cos s sin t sin s
sin(t + s) cos(t + s)
(d) (GL(2, R), I) = (O(2), I) = (SO(2), I) = (S 1 , 1) = Z.
(10) Let A be a compact contractible subspace of S 2 . Show that A does not separate S 2 .
Solution Consider any two points, a, b S 2 \ A. Define f : A S 2 ab to be the identity
map. Then by Borsuk lemma, a and b lie in the same component of S 2 \ f (A) = S 2 \ A.
Since a, b were chosen randomly, all points of S 2 \ A lie in the same component and so A
does not separate S 2 .
References
[M]
[S]
[L]

Munkres, Topology.
Basic Set Theory, http://www.math.cornell.edu/matsumura/math4530/basic set theory.pdf
Lecture notes, available at http://www.math.cornell.edu/matsumura/math4530/math4530web.html

MATH 4530 Topology. Prelim II


TAKE HOME (DUE Nov 18th in class)
This is an exam. Please do it on your own. You can discuss it with the lecturer.
Write the solutions in complete sentences and also in your own words.
Degree of maps from S 1 to S 1
We want to define a homotopy invariant for continuous maps S 1 S 1 to classify them. Follow the
instructions and answer the questions (1) (8).
Let S 1 := {e2i } C. Let x0 S 1 and let be a path from 1 S 1 to x0 . Let be a generator of
1 (S 1 , 1).
(1) Show that ()

is a generator of 1 (S 1 , x0 ). See Appendix 1.


(2) Show that ()

depends only on x0 (not on paths ).


By (2), it is OK to write x0 := ()

for any path from 1 to x0 . Now let h : S 1 S 1 be a continuous


1
map. Let x0 S and let x1 := h(x0 ). Define degree of h to be an integer d such that
h ( x0 ) = ( x1 )d .
It is well-defined because, by (1), x1 is a generator of 1 (S 1 , x1 ).
(3) Show that d is independent of the choice of x0 .
(4) Show that d is independent of the choice of .
By (3) and (4), we have defined the degree of a map h, which is independent of all choices. Now we
consider the properties of this degree:
(5) Show that if h, k : S 1 S 1 are homotopic, they have the same degree.
(6) Show that deg h k = deg h deg k.
(7) Compute the degree of the map h(z) = zn where n Z.
(8) (Optional) Show that if h, k : S 1 S 1 have the same degree, then they are homotopic.
(5) says the degree is a homotopy invariant, i.e. if h, k have the dierent degrees, they can not be homotopic
to each other. Together with (7) and (8), it classifies all homotopy equivalence classes of maps S 1 S 1 .
(6) says associating degrees have a certain algebraic structure.
Appendix
*1 An infinite cyclic group G is a group isomorphic to (Z, +). We say g G is a generator, if
G = {gn | n Z}. If g is a generator, then g1 is also a generator and any generator is either g or
g1 .
1

MATH 4530 Topology. Prelim II, Solutions


TAKE HOME

1. Degree
Let S 1 := {e2i } C. We define the degree of a continuous map S 1 S 1 as follows. Let x0 S 1 and
let be a path from 1 S 1 to x0 .
(1) Show that if is a generator of 1 (S 1 , 1), then ()

is a generator of 1 (S 1 , x0 ). See Appendix 1.


Solution: Let [ f ] 1 (S 1 , x0 ). Then 1 ([ f ]) = [] [ f ] []
1 (S 1 , 1). Since is a
1
generator for 1 (S , 1), there is some n such that [] [ f ] []
= n which implies
[ f ] = []
n [] = ([]
[])n = ()
n.
(2) Show that ()

depends only on x0 but not on paths .


Solution: Take another path from 1 to x0 . Then is a loop at 1. Now
[] []
[] = ().

[] = []
[ ]
[] []

()
= []
[ ]
1 [] = []
By (2), it is OK to write x0 := [] for a path from 1 to x0 . Now let h : S 1 S 1 be a continuous map.
Let x0 S 1 and let x1 := h(x0 ). Define degree of h to be an integer d such that
h ( x0 ) = ( x1 )d .
It is well-defined because, by (1), x1 is a generator of 1 (S 1 , x1 ) respectively.
(3) Show that d is independent of the choice of x0 .
Solution: Let y0 S 1 and let be a path from x0 to y0 . Let y1 := h(y0 ), then 0 := h is
a path from x1 to y1 .
y1 = [ 0 ] x1 [0 ]
x0 []) = [ 0 ] h ( x0 ) [0 ] = [ 0 ] ( x1 )d [0 ] = ([ 0 ] x1 [0 ])d = (y1 )d .
h (y0 ) = h ([]
(4) Show that d is independent of the choice of .
Solution: By *1, a generator is either or 1 . We we use 1 , then
(1 ) x0 = ( x0 )1 ,

(1 ) x1 = ( x1 )1 .

Thus h (( x0 )1 ) = (( x1 )d )1 = (( x1 )1 )d .
By (3) and (4), we have defined the degree of a map h, which is independent of all choices. Now we
consider the properties of this degree:
(5) Show that if h, k : S 1 S 1 are homotopic, they have the same degree.
Solution: It follows from Theorem 11.1 Lecture Notes.
(6) Show that deg h k = deg h deg k.
Solution: It follows from (8) and (7).
1

(7) Compute the degree of the map h(z) = zn where n Z.


Solution: HW 9 (5).
(8) (Optional) Show that if h, k : S 1 S 1 have the same degree, then they are homotopic.
Solution:
Consider h : 1 (S 1 , 1) 1 (S 1 , h(1)) and k : 1 (S 1 , 1) 1 (S 1 , k(1)). By the
n
n .
assumption, there is n such that h () = h(1)
and k () = k(1)
Let := [p|I ] where p : R S 1 , t 7 e2it is the standard map and I := [0, 1]. Consider
the following lifting diagram,
]I ,
hp|

p|I

/ S1

5R
]I
kp|

h, k


/ S1

h]
p|I and k]
p|I are the lifts of h p|I and k p|I at h0 p1 (h(1)) and k0 p1 (k(1))
respectively. Let h1 := h]
p|I (1) and k1 := k]
p|I (1).
n and k () = [k p| ] = n , we have h h = k k = n.
Since h () = [h p|I ] = h(1)

I
1
0
1
0
k(1)
]
]
I wanted to find a path-homotopy between h p|I and k p|I , but because the starting
points and ending points are different, I can find it. So I will shift one of them. Define
]
h]
p| (s) := h]
p| (s) h + k .
I

Now it is a path from k0 to k1 . You can check it by evaluating at 0 and 1. Thus there is
]
p|I to k]
p|I , because R is a contractible space.
a path-homotopy F from h]
]
2i(k
h
)
0
0

p|I is the lift of h p|I at k0 . Then we check


Consider h(x) := h(x) e
. Then h]
p|I to k p|I . Now since p F is a path-homotopy,
that p F is a path homotopy from h
it factors through (p, id), giving a homotopy F from h to k.
Now the homotopy from h to h is easy to find:
G(x, t) := h(x)e2it(k0 h0 ) .
By constructing F and G, we have shown that h is homotopic to k.
(5) says the degree is a homotopy invariant, i.e. if h, k have the different degrees, they can not be homotopic
to each other. Together with (7) and (8), it classifies all homotopy equivalence classes of maps S 1 S 1 .
(6) says associating degrees have a certain algebraic structure.
Appendix
*1 An infinite cyclic group G is a group isomorphic to (Z, +). We say g G is a generator, if
G = {gn | n Z}. If g is a generator, then g1 is also a generator and any generator is either g or
g1 .

Fall 2010 MATH 4530 Topology. FINAL EXAM


2hr 30min: 9:00-11:30 Tue 12/14 2010.
Closed Book, 6 problems, total 105 points (5 points bonus!).
(1) (24 pts) True or False. Dont need to explain. Just say True or False. (Correct answer +4, Wrong
answer +0, No answer +1).
(a) One point compactification of R is homeomorphic to a circle.
(b) Let X and Y be any topological spaces. Then if f : X Y is a continuous map, then the
graph G f = {(x, f (x)) | x X} is a closed subspace in X Y.
(c) The connectedness of a topological space implies the path-connectedness.
(d) There is a path-connected topological group G such that 1 (G, g) is not abelian (non-commutatve)
for some element g G.
(e) There is no continuous map f : D2 D2 such that f (x) , x for all x D2 where D2 is the
2-disk.
(f) A covering map is a quotient map.

(2) (16 pts) Find the fundamental group of the following spaces. Dont need to explain. Just say what
they are. (Correct answer +4, Wrong answer +0, No answer +1)
(a) 1 (GL(2, R), I2 ) where GL(2, R) is the set of all 2 2 matrices with real numbers whose
determinant is not zero and I2 is the identity matrix.
(b) 1 (R3 { 3 points distinct from origin }, origin ).
(c) The fundamental group of RP2 ]T where RP2 is the real projective space of dimension 2 and
T is a torus.
(d) The fundamental group of the eight figure.

(3) (10pts) Recall the unique lifting theorems of paths and path-homotopy:
Theorem: Let p : E B is a covering map. Let f, g : [0, 1] B denote paths from b0 to b1 .
(i) There is a unique lifting f of f at each point e0 p1 (b0 ).
(ii) For every path-homotopy F from a path f to a path g in B, there is a unique lifting F of F
such that F(0, 0) = e0 for each e0 p1 (b0 ), and F is a path-homotopy from the lifting f of
f at e0 to the lifting g of g at e0 .
Prove that for every covering map p : E B, the induced group homomorphism p :
1 (E, e0 ) 1 (B, b0 ) is injective.

(4) (15pts) Let X be a topological space, and A and B compact subspaces of X.


(a) (8pts) Show that A B is compact.

(b) (7pts) Show that if X is Hausdorff, then A B is compact.

(5) (a) (10pts) Prove that the continuous bijection from a compact space to a Hausdorff space is a
homeomorphism.

(b) (10pts) Define the real projective space RP2 to be the quotient of R3 {(0, 0, 0)} by the action
of R {0} where R {0} sends (x, y, z) to (x, y, z). Prove that RP2 is homeomorphic
to the quotient of the 2-sphere S 2 R3 through the identification of the antipodal points (i.e.
(x, y, z) (x, y, z)). Hint: Use the statement in (a).

(6) (a) (10pts) Show that the Klein bottle is homeomorphic to RP2 ]RP2 by using polygon quotient
pictures.

(b) (10pts) Write a polygon quotient picture of a torus T , and find a triangulation of T in the
polygon quotient picture. Show that the Euler characteristic of T is 0.

You might also like